Blackboard Mock Exam 2018 Flashcards Preview

Y3 z Symptom differentials/cases > Blackboard Mock Exam 2018 > Flashcards

Flashcards in Blackboard Mock Exam 2018 Deck (201)
Loading flashcards...
1
Q

A 65 year old obese man presents with gradual worsening dysphagia for solids, which had initially been intermittent. He has had GORD for many years but is poorly compliant with medication.

What is the diagnosis?
A.	Oesophageal cancer
B.	Benign oesophageal stricture
C.	Globus hystericus
D.	Candidal oesophagitis
E.	Parkinson’s disease
F.	Scleroderma
G.	Diffuse oesophageal spasm
H.	Eosinophilic oesophagitis
I.	Upper oesophageal web
J.	Oesophageal diverticulum
K.	Stroke
L.	Achalasia
A

B. Benign oesophageal stricture

Healing of oesophageal damage inflicted in GORD involves the deposition of collagen. This causes contraction of the distal oesophagus which causes the formation of strictures. This is often associated with dysphagia for solids. Other complications of GORD include oesophageal ulcer, haemorrhage or perforation, Barrett’s oesophagus and oesophageal adenocarcinoma.

2
Q

A 30 year old woman presents with aspiration pnuemonia. She has a long history of intermittent mild dysphagia for both liquids & solids and often suffers from severe retrosternal chest pain. Occasionally she gets food stuck but overcomes this by drinking vast amounts of water.

What is the diagnosis?
A.	Oesophageal cancer
B.	Benign oesophageal stricture
C.	Globus hystericus
D.	Candidal oesophagitis
E.	Parkinson’s disease
F.	Scleroderma
G.	Diffuse oesophageal spasm
H.	Eosinophilic oesophagitis
I.	Upper oesophageal web
J.	Oesophageal diverticulum
K.	Stroke
L.	Achalasia
A

L. Achalasia

This is achalasia which is a motility disorder with loss of peristalsis in the distal oesophagus and failure of the LOS to relax in response to swallowing. This presents commonly with dysphagia to both liquids and solids, regurgitation and retrosternal chest pain, which can be slowly progressive over time. In structural obstruction such as cancer, dysphagia to liquids is uncommon unless the disease is very advanced. Retrosternal pressure experienced can be precipitated by drinking liquids but is eased by continuing to drink, and the pain may be relieved by cold water. This may wake the individual from sleep. A UGI endoscopy is needed to exclude malignancy as a cause of dysphagia. The diagnosis is established on manometry or barium studies. Treatment is symptomatic.

3
Q

A 75 year old male smoker presents with a 3 month history of dysphagia for solids. He has lost 8kg in weight over the last 5 months. O/E he has lymphadenopathy.

What is the diagnosis?
A.	Oesophageal cancer
B.	Benign oesophageal stricture
C.	Globus hystericus
D.	Candidal oesophagitis
E.	Parkinson’s disease
F.	Scleroderma
G.	Diffuse oesophageal spasm
H.	Eosinophilic oesophagitis
I.	Upper oesophageal web
J.	Oesophageal diverticulum
K.	Stroke
L.	Achalasia
A

A. Oesophageal cancer

Dysphagia (normally in a progressive pattern) coupled with weight loss points to malignancy. Dysphagia occurs when there is obstruction of more than 2/3 of the lumen and presence indicates locally advanced disease. There may additionally be odynophagia. Lymphadenopathy is a sign of metastatic disease here. Men are twice as likely to develop oesophageal cancer. GORD, Barrett’s oesophagus, FH, tobacco and alcohol are all risk factors. The two main types are squamous cell carcinoma and adenocarcinoma. Tumours in the upper 2/3 of the oesophagus are SCC whereas those that lie in the lower 1/3 are adenocarcinomas. The main test to order is an OGD with biopsy. Treatment is either surgical resection or with chemo or radiotherapy alongside endoscopic ablation with or without stenting and brachytherapy.

4
Q

A 33 year old presented with retrosternal discomfort on swallowing but without any real difficulty swallowing. O/E he was found to have creamy plaques in his mouth and later admitted to having AIDS.

What is the diagnosis?
A.	Oesophageal cancer
B.	Benign oesophageal stricture
C.	Globus hystericus
D.	Candidal oesophagitis
E.	Parkinson’s disease
F.	Scleroderma
G.	Diffuse oesophageal spasm
H.	Eosinophilic oesophagitis
I.	Upper oesophageal web
J.	Oesophageal diverticulum
K.	Stroke
L.	Achalasia
A

D. Candidal oesophagitis

This is really odynophagia rather than true dysphagia. Whenever a patient complains of ‘difficulty swallowing’, you should always take a good history and explain what the patient actually means by this. Does the patient mean that there is true difficulty swallowing, or just that it is painful to swallow, or is there a feeling of a lump in the throat (globus). The white plaques here give alongside discomfort away the diagnosis. The patient also has AIDS and is therefore immunocompromised. Candidiasis here is an opportunistic infection in an immunocompromised host and is a mucosal infection caused in most cases by Candida albicans (and occasionally by other species). Patients may present like this, or with oral or vaginal infection. Oral thrush may be seen (pseudomembranous candidiasis) with altered taste sensation or indeed dysphagia, as well as odynophagia in oesophageal disease. In vaginal infection, there is erythema with a white discharge and pruritis. This patient should be started on an empirical trial of antifungals. A systemic azole can be used such as fluconazole or itraconazole.

5
Q

A 45 year old female with a history of psychological problems presented with difficulty swallowing which had been getting progressively worse over the last 6 months. She described a sensation of a lump in the throat but after examinations and an endoscopy, no cause could be found.

What is the diagnosis?
A.	Oesophageal cancer
B.	Benign oesophageal stricture
C.	Globus hystericus
D.	Candidal oesophagitis
E.	Parkinson’s disease
F.	Scleroderma
G.	Diffuse oesophageal spasm
H.	Eosinophilic oesophagitis
I.	Upper oesophageal web
J.	Oesophageal diverticulum
K.	Stroke
L.	Achalasia
A

C. Globus hystericus

Globus hystericus is a sensation of fullness or a lump in the neck or difficulty swallowing which is not a true case of dysphagia. Swallowing can be performed normally and there is no real lump or obstruction in the throat. In some cases the cause is unknown and is believed to be psychogenic in cause and is associated with anxiety disorders. In other cases throat inflammation can cause this sensation. The normal examination and endoscopy here in a patient with a psychiatric history is diagnostic.

6
Q

An adult male with hypogonadism, small testicles and gynaecomastia is found to be infertile.

What is the diagnosis?
A.	DiGeorge syndrome 
B.	Down's syndrome
C.	Edwards syndrome
D.	Klinefelter's syndrome
E.	William’s syndrome
F.	Tuberous sclerosis
G.	Turner's syndrome 
H.	Fragile X syndrome
I.	Prader-Willi syndrome
J.	Patau's syndrome 
K.	Angelman syndrome
A

D. Klinefelter’s syndrome

Klinefelter’s syndrome is the presence of an extra X chromosome in a male to give 47, XXY. Hypogonadism is a principle feature of this condition and there is reduced fertility. Hypogonadism itself does not mean ‘small testicles’ but XXY men do also have small testicles. They will also often have low testosterone levels but high LH and FSH levels due to primary hypogonadism. The only reliable method of diagnosis is with karyotype analysis and the degree to which XXY males are affected varies from person to person. Gynaecomastia is to some extent present in around a third of individuals affected by this condition. 1 in 10 will choose cosmetic surgery to fix this.

7
Q

A child with severe learning difficulties, cleft lip & palate, polydactyly and multiple heart defects. Karyotype analysis shows trisomy 18.

What is the diagnosis?
A.	DiGeorge syndrome 
B.	Down's syndrome
C.	Edwards syndrome
D.	Klinefelter's syndrome
E.	William’s syndrome
F.	Tuberous sclerosis
G.	Turner's syndrome 
H.	Fragile X syndrome
I.	Prader-Willi syndrome
J.	Patau's syndrome 
K.	Angelman syndrome
A

C. Edward’s syndrome

Edwards syndrome is trisomy 18 and is phenotypically similar to Patau’s. Most affected are females and most die before birth. The incidence, as with most trisomies, increases with advanced maternal age. The rate of survival is low due to cardiac abnormalities, renal malformations and other visceral disorders. Signs and symptoms include those mentioned and a whole host of other signs of this phenotype such as a small head, small jaw, widely spaced eyes and ptosis. The cardiac defects seen include VSD, ASD and PDA (all the lovely 3 letter acronyms). Classic EMQ signs include webbing of the second and third toes and the Rocker bottom feet characterised by calcaneal prominence and a convex rounded bottom to the foot, which is associated with both trisomy 13 and 18.

8
Q

A male child is found to have moderate learning difficulties and behavioural problems. There is a family history of learning difficulties. On examination he has large testicles, epicanthic folds and large ears. DNA testing reveals trinucleotide repeat expansion (CGG).

What is the diagnosis?
A.	DiGeorge syndrome 
B.	Down's syndrome
C.	Edwards syndrome
D.	Klinefelter's syndrome
E.	William’s syndrome
F.	Tuberous sclerosis
G.	Turner's syndrome 
H.	Fragile X syndrome
I.	Prader-Willi syndrome
J.	Patau's syndrome 
K.	Angelman syndrome
A

H. Fragile X syndrome

This is fragile X syndrome. History includes learning difficulties, which can range from mild to severe, social communication difficulties (patients may be autistic), hyperactivity and attention deficit and motor co-ordination difficulties. There may be a FH of learning difficulties too. Examination may reveal macrocephaly, low muscle tone, long face, high arched palate, prominent jaw, big testicles (macro-orchidism), large ears and strabismus. DNA testing is diagnostic and reveals a ragile site on Xp27.3 (FRM1 gene position). This is characterised by trinucleotide repeat expansion (CGG) to more than 200 copies.

9
Q

A child with moderate learning difficulties, round face, small head, slanting eyes and a single palmar crease.

What is the diagnosis?
A.	DiGeorge syndrome 
B.	Down's syndrome
C.	Edwards syndrome
D.	Klinefelter's syndrome
E.	William’s syndrome
F.	Tuberous sclerosis
G.	Turner's syndrome 
H.	Fragile X syndrome
I.	Prader-Willi syndrome
J.	Patau's syndrome 
K.	Angelman syndrome
A

B. Down’s syndrome

This is the one you need to be really aware of. Down’s syndrome is trisomy 21 and the diagnosis is one which is made antenatally or perinatally. You will never have a patient with Down’s who gets diagnosed as a child unless you are in a country which is very deprived of any medical personnel and your patient was born in a rural farm away from civilisation. The patient may have a history of delayed development, congenital cardiac anomalies, epilepsy as a child, atlanto-occipital instability, GI or hearing problems and there may also be associated autism. Examination may display dysmorphism, oblique palpebral fissures, epicanthic folds, low nasal bridge and low set ears, characteristic central iris Brushfield spots, short curved 5th finger, single palmar crease and may also have cardiac murmurs. Karyotype analysis will reveal trisomy 21, robertsonian translocation, or mosaicism.

10
Q

Adult female with short stature, amenorrhoea, webbed neck and widely spaced nipples. She is found infertile although there is no cognitive impairment.

What is the diagnosis?
A.	DiGeorge syndrome 
B.	Down's syndrome
C.	Edwards syndrome
D.	Klinefelter's syndrome
E.	William’s syndrome
F.	Tuberous sclerosis
G.	Turner's syndrome 
H.	Fragile X syndrome
I.	Prader-Willi syndrome
J.	Patau's syndrome 
K.	Angelman syndrome
A

G. Turner’s syndrome

Turner’s syndrome is characterised on diagnostic karyotype analysis by 45 XO (complete or partial absence of the second sex chromosome occuring in 1 in 2500 liver female births). This may be diagnosed antenatally by amniocentesis, which is an invasive test performed after 15 weeks gestation. The patient will be phenotypically female and may give a history of amenorrhoea, infertility, visual problems and hearing loss. Specific learning difficulties are normal but there is no cognitive impairment and intelligence is normal. Examination may reveal short stature, low-set ears, webbed neck, low hairline and cubitus valgus. Obvious stigmata though such as webbed neck only affects 20-30% of patients.
I wouldn’t worry too much about most of these syndromes. A brief overview of the rest for those who are interested: Prader-Willi is characterised by 15q11-13 deletion and patients can have a short stature, small hands and feet, and dysmorphism. The classic association is with hyperphagia and obesity from 3 years of age. Angelman’s is characterised by 15q11-12 deletion and there is severe general developmental delay and cognitive impairment. William’s is characterised by deletion of 7q11.23 and has classic phenotypic features. DiGeorge classically presents with a triad of cardiac abnormalities, hypoplastic thymus and hypocalcaemia but manifestations are highly variable. Tuberous sclerosis is characterised by seizures in infants and children, variable cognitive impairment and examination may reveal ash leaf patches of hypomelanotic macules. There is either mutation of TS1 or TS2 on DNA analysis.

11
Q

A 31 year old woman is brought into A&E by her boyfriend who claimed she may have had too many ‘sleeping pills’ after he tried to break up with her. Prior to this she had drank two bottles of wine and consumed three chocolate cakes. She is ataxic with slurred speech with a GCS of 10. Her medical file shows she is taking medication for panic attacks.

What is the cause?
A.	Alcohol intoxication
B.	Carbon monoxide
C.	Anthrax
D.	Opiate analgesics
E.	Methanol
F.	Ecstasy
G.	Benzodiazepines
H.	Insulin
I.	Tricyclic antidepressants
J.	Ethylene glycol
K.	Sympathomimetics
L.	Salicylates
M.	Volatile solvents
A

G. Benzodiazepines

This woman here who is clearly distraught after her breakup has overdosed on benzodiazepines. The clue here is given when it says she is taking medication for panic attacks at the moment. BZDs are the most commonly prescribedmedication for anxiety disorders, sedation and sleep. Patients may present like this and may be intentional or accidental in nature, and may be in combination with other CNS depressants such as alcohol and opioids in older people. Occasionally overdose is due to medication error. The key feature of overdose is excessive sedation and anterograde amnesia. Vital signs are unremarkable. Larger doses can lead to coma and respiratory depression. Treatment is symptomatic and may include assisted ventilation and haemodynamic support and death is uncommon and often due to mixed overdoses with other depressants such as alcohol. Flumazenil is a BZD antagonist that can be used in first time or infrequent users to reverse CNS depression but it is contraindicated in those who are long-term of frequent users (like this patient) due to the risk of provoking seizures, which outweights the benefits.

12
Q

A 29 year old man presents to A&E with agitation, tremor, dilated pupils, tachycardia, arrhythmias, convulsions after ingesting an overdose of an unknown substance.

What is the cause?
A.	Alcohol intoxication
B.	Carbon monoxide
C.	Anthrax
D.	Opiate analgesics
E.	Methanol
F.	Ecstasy
G.	Benzodiazepines
H.	Insulin
I.	Tricyclic antidepressants
J.	Ethylene glycol
K. Sympathomimetics
L.	Salicylates
M.	Volatile solvents
A

K. Sympathomimetics

The symptoms described here are those of sympathetic activation and the overdose here is of sympathomimetics. This group of drugs mimic the effects of transmitter substances of the sympathetic nervous system such as adrenaline, dopamine and noradrenaline.

13
Q

A 23 year old man who has taken an overdose of an unknown drug after getting dumped by his pregnant girlfriend (he is not the father) was admitted to A&E. He is slightly tachycardic, complains of tinnitus and has high blood pressure at first but 30 minutes later, starts seizing and is intubated. There is a wide anion-gap metabolic acidosis.

What is the cause?
A.	Alcohol intoxication
B.	Carbon monoxide
C.	Anthrax
D.	Opiate analgesics
E.	Methanol
F.	Ecstasy
G.	Benzodiazepines
H.	Insulin
I.	Tricyclic antidepressants
J.	Ethylene glycol
K.	Sympathomimetics
L.	Salicylates
M.	Volatile solvents
A

L. Salicylates

This is salicylate overdose which is potentially fatal and can present either acutely or indolently with more chronic exposure. It is a relatively common overdose so really with anyone presenting with an unknown overdose it should be considered along with paracetamol. The unexplained acid-base disturbance should make you suspicious of this diagnosis. Tinnitis is common in the early stages of acute salicylate poisoning and reflects CNS toxicity. There may also be deafness and both are reversible. Seizures are common especially in patients with salicylate levels >80mg/dL. An ABG is also indicated in this patient which during the course of salicylate poisoning would initiually show a respiratory alkalosis and later a concomitant metabolic acidosis, potentially with a wide anion gap. GIT decontamination should be considered as an adjunct on arrival to A&E and activated charcoal can be given. The mainstay of treatment is alkaline diuresis induced by an infusion of sodium bicarbonate. In cases of severe poisoning, it is still started as a bridge to haemodialysis.

14
Q

Inebriation, coma, reduced reflexes, tachycardia, pulmonary oedema, shock, metabolic acidosis.

What is the cause?
A.	Alcohol intoxication
B.	Carbon monoxide
C.	Anthrax
D.	Opiate analgesics
E.	Methanol
F.	Ecstasy
G.	Benzodiazepines
H.	Insulin
I.	Tricyclic antidepressants
J.	Ethylene glycol
K.	Sympathomimetics
L.	Salicylates
M.	Volatile solvents
A

B. Carbon monoxide

This man who works in a petrol refinery is suffering from carbon monoxide poisoning and his co-worker who has found him lying on the ground is starting to suffer from similar symptoms due also to CO exposure. CO is a colourless and odourless gas so patients may not initially be aware of the poisoning which can cause hypoxia, cell damage and death (in approximately 1/3). CO can come from fire or non-fire sources and early symptoms are non-specific and include the symptoms seen here: headache, dizziness and nausea. Increasing exposure leads to cardiovascular effects like myocardial ischaemia, infarction and possible arrest. Neurological symptoms include the confusion seen here and may lead eventually to coma and syncope. Diagnosis is based on carboxyhaemoglobin levels and the clinical picture here is very suggestive. It is worth noting that in severe CO poisoning, pulse oximetry readings may be falsely raised. Key in treatment is the use of high-flow oxygen, hyperbaric oxygen and supportive care. Hyperbaric treatment entails complications such as barotrauma, oxygen toxicity and pulmonary oedema.

15
Q
A pregnant 17 year old woman who has been recently abandoned by her boyfriend is brought into hospital with altered mental status by her mother, who reported that she had a brief seizure beforehand. She is tachycardic with low BP and appears flushed. She responds to pain only and her skin feels warm and dry. Pupils are poorly reactive to light and dilated.
What is the cause?
A.	Alcohol intoxication
B.	Carbon monoxide
C.	Anthrax
D.	Opiate analgesics
E.	Methanol
F.	Ecstasy
G.	Benzodiazepines
H.	Insulin
I.	Tricyclic antidepressants
J.	Ethylene glycol
K.	Sympathomimetics
L.	Salicylates
M.	Volatile solvents
A

I. Tricyclic antidepressants

This is an overdose of tricyclic antidepressants which are a class of drugs with a narrow therapeutic index and therefore become potent toxins in moderate doses to both the CNS and cardiovascular system. This patient has reason to be depressed and is therefore on these pills. There is a sudden deterioration of mental status here and the diagnosis here is clinical. The main aim in treatment is to provide respiratory and cardiovascular support until the medicine has been fully metabolised and eliminated. The warm, dry and flushed skin is part of the anticholinergic effects (physostigmine should NOT be used to reverse this as it has been in rare cases been associated with asystole – would you rather have a patient who is flushed or flatlined?). Other anticholinergic effects include dilated pupils, urinary retention, decreased or absent bowel sounds and changes in mental status. Hypotension is common and is due to alpha 1 antagonism. Classic ECG changes are of sinus tachycardia progressing to wide complex tachycardia and ventricular arrhythmias (with increasing severity and intoxication). Condution problems and hypotension is improved with hypertonic sodium bicarbonate and if arrhythmias are present, treatment of these involves correcting the acidosis, hypoxia and electrolyte abnormalities. Anti-arrhythmics are generally avoided. If hypotension is refractory then a vasopressor can be used. BZDs can be used for any seizures.

16
Q

A 66 year old woman admitted with diarrhoea and weight loss. O/E she has a fine tremor and has rapid atrial fibrillation. FBC, ESR and CRP are normal.

What is the diagnosis?
A.	Clostridium difficile
B.	Amoebic dysentery
C.	Crohn's disease
D.	Ulcerative colitis
E.	Malabsorption
F.	Thyrotoxicosis
G.	Cancer of the colon
H.	Cancer of the rectum
I.	Bacterial gastroenteritis
J.	Diverticular disease
K.	Drug induced
L.	Irritable bowel syndrome
A

F. Thyrotoxicosis

This woman has symptoms and signs of hyperthyroidism (diarrhoea, weight loss, AF and a fine tremor). In countries where sufficient iodine intake is not an issue, Graves’ disease is the most common cause of hyperthyroidism. Peripheral manifestations such as ophthalmopathy, pretibial myxoedema and hyperthyroid acropachy do not occur with other causes of hyperthyroidism. Acropachy manifests as clubbing with soft tissue swelling. Pretibial myxoedema is almost always associated with ophthalmopathy. Treatment aims to normalise thyroid function and is achieved by radioactive iodine, antithyroid medications or with surgery. They are all effective and relatively safe options. Symptomatic therapy is given with beta blockers such as propranolol.

17
Q

A 30 year old city banker comes to the GP with symptoms of abdominal bloating, intermittent constipation and diarrhoea with occasional nausea. This started about 6 months ago but she has not lost any weight or found any blood in her stools.

What is the diagnosis?
A.	Clostridium difficile
B.	Amoebic dysentery
C.	Crohn's disease
D.	Ulcerative colitis
E.	Malabsorption
F.	Thyrotoxicosis
G.	Cancer of the colon
H.	Cancer of the rectum
I.	Bacterial gastroenteritis
J.	Diverticular disease
K.	Drug induced
L.	Irritable bowel syndrome
A

L. Irritable bowel syndrome

The intermittent diarrhoea and constipation, with bloating, without symptoms suggestive of IBD make IBS a more likely diagnosis. IBS is a chronic condition with abdominal pain associated with bowel dysfunction and is a diagnosis of exclusion. The pain or discomfort may be relieved by defecation. Examination is usually unremarkable and the diagnosis is based on the patient’s history in line with the Rome Criteria. If the patient presents with any worrying symptoms, then these will warrant a more thorough investigation. Treatment depends on the patient’s predominant symptoms. Antispasmodics relieve abdominal pain or discomfort but do not affect bowel habit. Examples include peppermint oil and dicycloverine. Laxatives can also be used such as lactulose. Lifestyle and dietary modifications combined with reassurance remain the 1st line intervention for functional bowel disease. IBS is linked with stressful jobs such as working as a banker in the city and there is a female/male ratio of 2:1.

18
Q

An 18 year old student attends A&E at 2am with acute onset of vomiting, diarrhoea and abdominal cramps. There is some blood in the stool and he has a high fever. He hasn’t been abroad recently. His FBC had a normal Hb but raised neutrophils.

What is the diagnosis?
A.	Clostridium difficile
B.	Amoebic dysentery
C.	Crohn's disease
D.	Ulcerative colitis
E.	Malabsorption
F.	Thyrotoxicosis
G.	Cancer of the colon
H.	Cancer of the rectum
I.	Bacterial gastroenteritis
J.	Diverticular disease
K.	Drug induced
L.	Irritable bowel syndrome
A

I. Bacterial gastroenteritis

This acute presentation in a previous fit and healthy individual and the raised neutrophils on FBCand a fever indicate an infective cause for his GI symptoms. It is self-limiting and diagnosis is on isolating the organism from a stool culture (if needed). Blood in the stool allows you to narrow down the list of potential causative organisms. Treatment is supportive with fluid and electrolyte replacement and antibiotics are generally used only for patients with risk factors for severe disease or those with extra-GI complications. It is worth noting that viral gastroenteritis often presents with mainly UGI symptoms like N&V more so than diarrhoea.

19
Q

A 76 year old woman admitted with a chest infection develops non-bloody diarrhoea on the ward. She was on cefuroxime and erythromycin for her chest. She appears unwell and there is a fever. CRP is elevated.

What is the diagnosis?
A.	Clostridium difficile
B.	Amoebic dysentery
C.	Crohn's disease
D.	Ulcerative colitis
E.	Malabsorption
F.	Thyrotoxicosis
G.	Cancer of the colon
H.	Cancer of the rectum
I.	Bacterial gastroenteritis
J.	Diverticular disease
K.	Drug induced
L.	Irritable bowel syndrome
A

A. Clostridium difficile

This is infection with clostridium difficile with the major risk factor here of antibiotic exposure due to the recent chest infection. The most common ones implicated are ampicillin, second and third generation cephalosporins, clindamycin and fluoroquinolones, especially if used in the preceding 3 months (though most manifestations occur on days 4 through to 9 of antibiotic therapy). Diarrhoea may range from a few loose stools to severe diarrhoea, though absence could be related to toxic megacolon to paralytic ileus. Abdominal pain is also common as is fever. C. difficile produces 2 exotoxins which are responsible for its pathogenicity. These are called toxin A and toxin B (A is thought to be more important than B) which lead to an inflammatory response in the large bowel, increased vascular permeability and the formation of pseudomembranes. Colonic pseudomembranes look like raised yellow and white plaques against an inflamed mucosa and are composed of neutrophils, fibrin, mucin and cellular debris. The diagnostic standard is with cytotoxic tissue culture assay. Treatment involves discontinuing the implicated antibiotic and beginning oral metronidazole or vancomycin. 5-20% will have a recurrence on discontinuing treatment and will need a second course.

20
Q

A 30 year old female presents with a 3 month history of bloody diarrhoea and vague lower abdominal cramps. She gave up smoking a few months ago. The doctor feels that this could have contributed to her condition.

What is the diagnosis?
A.	Clostridium difficile
B.	Amoebic dysentery
C.	Crohn's disease
D.	Ulcerative colitis
E.	Malabsorption
F.	Thyrotoxicosis
G.	Cancer of the colon
H.	Cancer of the rectum
I.	Bacterial gastroenteritis
J.	Diverticular disease
K.	Drug induced
L.	Irritable bowel syndrome
A

D. Ulcerative colitis

While this could be Crohn’s disease, bloody diarrhoea is more commonly a presentation of UC than Crohn’s. UC is characterised by diffuse mucosal inflammation running a relapsing and remitting course. Bloody diarrhoea is commonly experienced by patients who may also complain of other symptoms such as (lower) abdominal pain, faecal urgency and the host of extra-intestinal manifestations associated with UC. These include erythema nodosum, pyoderma gangrenosum, sacroiliitis, ankylosing spondylitis, PSC, aphthous ulcers, episcleritis, peripheral arthropathy and anterior uveitis. Another clue in this question which makes you pick UC instead of Crohn’s is the fact the patient has given up smoking. While I remain convinced this link as a risk factor is a weak one, you should try to think like an EMQ when answering EMQs (generally the information is there for a reason). There is a weak risk of UC development in non-smokers and those who were a former smoker (though it is an established link). This is based on a review paper published by some German medics in an exciting journal named ‘Inflammatory Bowel Diseases’. Should be you interested you can check it out: Inflammatory Bowel Diseases. 10(6):848-859, November 2004 (just read the abstract if you want)
Diagnosis of UC requires endoscopy with biopsy and a negative stool culture to rule out infectious gastroenteritis. Flare ups are usually linked to pathogens so a stool culture will always be needed in these cases. Toxic megacolon is a complication which is associated with a risk of perforation. UC is also linked with bowel adenocarinoma and PSC. Treatment involves mesalazine (5-ASA) used to induce and maintain remission.

21
Q

An 18 year old girl has felt unwell with myalgia and general malaise for a week develops sharp chest pains which are worse when she lies flat. The pain is constant and unrelated to exertion. There have also been fevers.

What is the diagnosis?
A.	Atrial septal defect
B.	Dilated cardiomyopathy
C.	Infective endocarditits
D.	Mitral regurgitation
E.	 Mitral stenosis
F.	Pulmonary fibrosis
G.	Pericarditis
H.	Conduction system disease
I.	Pericardial effusion
J.	Aortic valve disease
K.	Tuberculosis
L.	Hypertensive cardiomyopathy
A

G. Pericarditis

This patient has presented with pericarditis – most likely viral following a viral infection (as suggested by the prodrome and fever). Symptoms include a sharp and severe chest pain retrosternally which is worse on inspiration and when supine, relieved by sitting forwards. The classical finding on examination is a friction rub which is said to sound like ‘walking on snow’ – a monophasic, biphasic or triphasic friction rib is pathognomic with close to 100% specificity. There may be diffuse ST elevations on ECG, an effusion on echocardiography and blood results suggesting inflammation. Complications include tamponade and constrictive pericarditis. Prior viral infection is a risk factor with the most common pericardial infection being viral. Bacterial purulent pericarditis also occurs. The inflammation is due either to direct viral attack or immune mediated damage. Other risk factors include male gender, post-MI (both ‘early’ and Dressler’s), post-pericardiotomy syndrome, neoplasm from local tumour invasion, uraemia and autoimmune conditions such as RA and SLE.

22
Q

During the month following his acute MI, a 56 year old man has become progressively more breathless. O/E he has a loud pan-systolic murmur

What is the diagnosis?
A.	Atrial septal defect
B.	Dilated cardiomyopathy
C.	Infective endocarditits
D.	Mitral regurgitation
E.	 Mitral stenosis
F.	Pulmonary fibrosis
G.	Pericarditis
H.	Conduction system disease
I.	Pericardial effusion
J.	Aortic valve disease
K.	Tuberculosis
L.	Hypertensive cardiomyopathy
A

D. MR can occur as a complication of MI which may cause structural damage to the mitral valve apparatus. MR is loudest at the apex and radiates to the axilla and tends to be around grade 4. It is associated with a systolic thrill at the apex. TTE is the investigation of choice for diagnosis. Chronic MR is associated with a laterally displaced apex beat with LV dilatation. This case of acute MR in the setting of an acute MI is very serious can lead to high LA pressure and pulmonary oedema secondary to reduced LA compliance. Occasionally no murmur is heard. Note that while a VSD also gives a pansystolic murmur, which is generally easily heard, and is loudest at the left parasternal region, with no axillary radiation.

23
Q

A 24 year old Asian male chef is referred by his GP after a 2 month history of cough, fever, night sweats and weight loss. The GP has prescribed anti-biotics with no improvement. He is admitted with SOB and haemoptysis. His CXR shows a normal heart size.

What is the diagnosis?
A.	Atrial septal defect
B.	Dilated cardiomyopathy
C.	Infective endocarditits
D.	Mitral regurgitation
E.	 Mitral stenosis
F.	Pulmonary fibrosis
G.	Pericarditis
H.	Conduction system disease
I.	Pericardial effusion
J.	Aortic valve disease
K.	Tuberculosis
L.	Hypertensive cardiomyopathy
A

K. Tuberculosis

It is important to have a high level of suspicion when evaluating patients with risk factors who present with suggestive symptoms. Night sweats, fever, weight loss, malaise, cough, haemoptysis and erythema nodosum are all suggestive. In the first half of the 20th century, tuberculosis accounted for over 90% of cases of erythema nodosum. Other key risk factors for pulmonary TB include exposure to infection and returning from or being born in a high-risk region such as Asia, Africa and Latin America (highly possible in this case). If TB is suspected, the patient should be placed in isolation and a CXR obtained with 3 sputum samples cultured for AFB being the gold standard of diagnosis. Culture takes several weeks so sputum smears will be done before culture results are known. Interferon-gamma release assays (IGRAs) are now used by some hospitals to rapidly determine a patient’s TB status. All patients who have TB should be tested for HIV within 2 months of diagnosis. Specific anti-TB medication is required, not standard antibiotics for community-acquired pneumonia.
The CXR in pulmonary TB is almost always abnormal in patients who are not immunocompromised. Patients with, for instance, advanced HIV, may have a normal CXR. Primary disease common presents as middle and lower zone infiltrates and ipsilateral adenopathy, atelectasis from airway compression and pleural effusion can be seen. Post-primary classically involves apical changes, spreading to other lobes/segments as the disease progresses. The CXR may be normal in normal individuals but it is rare and it is likely only part of this patient’s CXR is reported here so as not to completely give the game away in this EMQ. Or whoever was interpreted it should not have been allowed to pass finals, which is also possible.

24
Q

A 42 year old alcoholic is admitted with SOB. He has no murmurs but the apex is laterally displaced and there are crackles at the lung bases with raised JVP. There is also hepatomegaly, clubbing and multiple spider naevi.

What is the diagnosis?
A.	Atrial septal defect
B.	Dilated cardiomyopathy
C.	Infective endocarditits
D.	Mitral regurgitation
E.	 Mitral stenosis
F.	Pulmonary fibrosis
G.	Pericarditis
H.	Conduction system disease
I.	Pericardial effusion
J.	Aortic valve disease
K.	Tuberculosis
L.	Hypertensive cardiomyopathy
A

B. Dilated cardiomyopathy

It is worth noting that ventricular hypertrophy due to hypertension causes concentric hypertrophy i.e. the wall of the ventricle gets thicker inwards. Hence the apex beat is not displaced unlike in DCM. DCM is characterised by LV dilation and systolic dysfunction without significant coronary artery disease or abnormal loading conditions. RV dilation is often also present. 25-35% are familial (there may be FH of sudden death). Causes are extensive and include post-myocarditis, alcohol, chemotherapy agents, haemochromatosis, AI conditions and acromegaly. This case is alcohol related DCM with a history of alcohol excess, signs of chronic liver disease on examination and signs of systolic dusfunction on examination (crackles at lung bases, JVP distension and there may also be peripheral oedema). ECG may show non-specific ST-T changes, CXR can show an enlarged cardiac shadow and echo also give consistent results (wall thickness, LV dilation). LFTs, serum albumin and clotting profile may all be abnormal here too, and GGT would especially be expected to be elevated due to alcohol abuse.

25
Q

A 55 year old man with known carcinoma of the lungs, develops SOB over a few days. He has a large cardiac silhouette on his CXR but no pulmonary oedema.

What is the diagnosis?
A.	Atrial septal defect
B.	Dilated cardiomyopathy
C.	Infective endocarditits
D.	Mitral regurgitation
E.	 Mitral stenosis
F.	Pulmonary fibrosis
G.	Pericarditis
H.	Conduction system disease
I.	Pericardial effusion
J.	Aortic valve disease
K.	Tuberculosis
L.	Hypertensive cardiomyopathy
A

I. Pericardial effusion

This is a malignant effusion (one of the most likely to lead to tamponade) caused by lung cancer. Other prevalent malignant causes include breats cancer, lymphomas and leukaemias. This may also be the first sign of metastatic disease. The history of lung cancer here should make you suspicious. Other causes of a pericardial effusion include hypothyroidism (high protein content and accumulate very slowly due to capillary leak), cardiac causes such as CHF and dissection of the proximal aorta, trauma, radiation-related, uraemia, immune-mediated such as SLE, Dressler’s, amyloidosis and Wegener’s, infectious or idiopathic (which is generally assumed to be viral). Symptoms may coexist with those of pericarditis sometimes. ECG and CXR are indicated here. Most patients also get an echocardiogram whic is the preferred test to establish the diagnosis. On ECG there may be diffuse ST elevation and PR depression with epicardial inflammation. If the effusion is large enough there may be electrical alternans, which is beat-to-beat variation of the ventricular axis (find an image of this to cement it in your memory). The cardiac shadow on CXR is said to be ‘water-bottle shaped’. Pericardiocentesis may be necessary depending on the clinical case.

26
Q

A 40 year old female who had been taking ibuprofen for pain relief when she gets headaches, presents to A&E with a history of weight loss and melaena with pain in her epigastric region. The pain gets worse with eating.

What is the diagnosis?
A.	Meckel's diverticulum
B.	Duodenal ulcer
C.	Ulcerative colitis
D.	Oesophageal varices
E.	Oesophageal malignancy
F.	Mallory-Weiss tear
G.	Gastric ulcer
H.	Crohn's disease
I.	Oesophagitis
A

G. Gastric ulcer

The patient has a bleeding peptic ulcer (the black tarry stools from the UGI bleed). Epigastric pain and tenderness related to eating a meal is typical of a peptic ulcer. 80% are duodenal and 20% are gastric. Ulcers may cause iron deficiency anaemia and associated symptoms may feature. Key risk factors are NSAID use, like in this patient, H. pylori infection, smoking and a family history of PUD. Zollinger-Ellison syndrome should be considered if there are multiple ulcers or ulcers refractory to treatment.
Gastric ulcers classically cause pain which is exacerbated by eating and immediately relieved on vomiting. There is usually also weight loss due to a fear of food and its association with pain. Duodenal ulcers are classically made worse by hunger and are relieved by eating and the patient may wake at night with the pain. As a result, weight gain is typically a feature. In reality, it is difficult to differentiate the site of the ulcer based on these features.
The most specific and sensitive test is an upper GI endoscopy which is initially ordered if the patient has ‘red flag’ symptoms, is >55 years of age or fails to respond to treatment. Duodenal ulcers rarely undergo malignant transformation so do not require a compulsory biopsy but gastric ulcers require biopsies to rule this out. In patients who are 55 or younger without ‘red flags’, testing for Helicobacter pylori (breath testing with radiolabelled urea or stool antigen testing) is necessary. Management is aimed at correcting the underlying cause such as discontinuing NSAIDs. H. pylori eradication should be started if the organism is present with triple therapy. Otherwise, a PPI is indicated.

27
Q

A 25 year old Jewish man presents to A&E with some abdominal discomfort, weight loss with associated loss of appetite. His history revealed loose and bloody stools. Examination reveals tenderness in the RLQ. He is booked in for endoscopy.

What is the diagnosis?
A.	Meckel's diverticulum
B.	Duodenal ulcer
C.	Ulcerative colitis
D.	Oesophageal varices
E.	Oesophageal malignancy
F.	Mallory-Weiss tear
G.	Gastric ulcer
H.	Crohn's disease
I.	Oesophagitis
A

H. Crohn’s disease

This patient gives a history of IBD. This could well be UC where the mainstay of treatment is with 5-ASA. A colonoscopy is required to assess the extent of disease and for a definitive diagnosis. Biopsy in CD will show transmural granulomatous inflammation. CD can affect the whole GIT but favours the TI (RLQ pain) and proximal colon and is macroscopically characterised by skip lesions. UC on the other hand is characterised by the presence of crypt abscesses, which is pathognomic. CD risk is increased 3-4 fold by smoking whereas smoking seems protective in UC. The mainstay of treatment in CD is with steroids and azathioprine to revent relapses and for those suffering side effects of steroid treatment. TNF-alpha inhibitors also have a role. Surgery in CD is only indicated in a small number of patients who bleed, for bowel perforation and cases of complete obstruction. The aim is to rest distal disease by temporarily diverting faecal flow.

28
Q

A 36 year old gentleman presents with a 36 hour history of diarrhoea and vomiting following a takeaway meal the night before. In the last few hours he has increasing amounts of bright red blood in his vomit.

What is the diagnosis?
A.	Meckel's diverticulum
B.	Duodenal ulcer
C.	Ulcerative colitis
D.	Oesophageal varices
E.	Oesophageal malignancy
F.	Mallory-Weiss tear
G.	Gastric ulcer
H.	Crohn's disease
I.	Oesophagitis
A

F. Mallory-Weiss tear

This occurs after a rise in abdominal pressure which induces a tear in the oesophageal mucosa, causing subsequent GI bleeding. It commonly presents with haematemesis after an episode of retching/vomiting/coughing/straining. Hence, risk factors include anything which can cause vomiting like heavy alcohol use, which is commonly the case in EMQs. Also, other conditions would include food poisoning like in this case, bowel obstruction, hyperemesis gravidarum, bulimia, the chronic cough of COPD, meningitis etc… you name it really. Classically, MWT presents with a small self limiting episode of haematemesis. Definitive diagnosis is made by OGD. Treatment is supportive because most cases are self limiting and emergency treatment is not offered unless the patient is showing signs of clinical instability. If the patient is actively bleeding, treatment will be with therapeutic endoscopy in most cases, and very very few cases will require more intervention such as angiography with embolisation.

29
Q

An 80 year old man presents with a 6 month history of increasing weakness and 8kg weight loss. He also has some vague abdominal pain and a few episodes of black stools. He is a long term smoker.

What is the diagnosis?
A.	Meckel's diverticulum
B.	Duodenal ulcer
C.	Ulcerative colitis
D.	Oesophageal varices
E.	Oesophageal malignancy
F.	Mallory-Weiss tear
G.	Gastric ulcer
H.	Crohn's disease
I.	Oesophagitis
A

E. Oesophageal malignancy

Such levels of extreme weight loss over a short period of time with GI symptoms here points to GI malignancy which is bleeding. The only option on the list is oesophageal. EMQs normally mention dysphagia, which occurs when there is obstruction of more than 2/3 of the lumen and presence indicates locally advanced disease however this is absent here. There may additionally be odynophagia. Men are twice as likely to develop oesophageal cancer. GORD, Barrett’s oesophagus, FH, tobacco and alcohol are all risk factors. The two main types are squamous cell carcinoma and adenocarcinoma. Tumours in the upper 2/3 of the oesophagus are SCC whereas those that lie in the lower 1/3 are adenocarcinomas. The main test to order is an OGD with biopsy. Treatment is either surgical resection or with chemo or radiotherapy alongside endoscopic ablation with or without stenting and brachytherapy.

30
Q

A 38 year old man presents with a 2 month history of intermitted pain in the upper abdomen which he describes as dull in nature. It sometimes wakes him up at night and is relieved by food and particularly when he has a glass of milk. He has had a similar episode before where he remembers the doctor prescribed him some pills, which helped. Examination reveals mild epigastric tenderness.

What is the diagnosis?
A.	Meckel's diverticulum
B.	Duodenal ulcer
C.	Ulcerative colitis
D.	Oesophageal varices
E.	Oesophageal malignancy
F.	Mallory-Weiss tear
G.	Gastric ulcer
H.	Crohn's disease
I.	Oesophagitis
A

B. Duodenal ulcer

The patient has a duodenal ulcer. Epigastric pain and tenderness related to eating a meal is typical of a peptic ulcer. 80% are duodenal and 20% are gastric. Ulcers may cause iron deficiency anaemia and associated symptoms may feature. Key risk factors are NSAID use, like in this patient, H. pylori infection, smoking and a family history of PUD. Zollinger-Ellison syndrome should be considered if there are multiple ulcers or ulcers refractory to treatment.
Gastric ulcers classically cause pain which is exacerbated by eating and immediately relieved on vomiting. There is usually also weight loss due to a fear of food and its association with pain. Duodenal ulcers are classically made worse by hunger and are relieved by eating and the patient may wake at night with the pain. As a result, weight gain is typically a feature. In reality, it is difficult to differentiate the site of the ulcer based on these features.
The most specific and sensitive test is an upper GI endoscopy which is initially ordered if the patient has ‘red flag’ symptoms, is >55 years of age or fails to respond to treatment. Duodenal ulcers rarely undergo malignant transformation so do not require a compulsory biopsy but gastric ulcers require biopsies to rule this out. In patients who are 55 or younger without ‘red flags’, testing for Helicobacter pylori (breath testing with radiolabelled urea or stool antigen testing) is necessary. Management is aimed at correcting the underlying cause such as discontinuing NSAIDs. H. pylori eradication should be started if the organism is present with triple therapy. Otherwise, a PPI is indicated.

31
Q

A 30 year old Afro-Caribbean patient comes to A&E complaining of coughing up blood and a rash on his cheeks, nose and shins. The CXR shows hilar lymphadenopathy bilaterally.

What is the diagnosis?
A.	Tuberculosis
B.	Bronchiectasis
C.	Bronchial carcinoma
D.	Primary pulmonary hypertension
E.	Pulmonary embolus
F.	Streptococcus pneumoniae
G.	Trauma
H.	Left ventricular failure
I.	Mycoplasma pneumoniae
J.	 Idiopathic pulmonary fibrosis
K.	Sarcoidosis
L.	COPD
A

K. Sarcoidosis

Sarcoidosis is a chronic multisystem disease with an unknown aetiology but pulmonary involvement usually dominates. Hypercalcaemia occurs in these granulomatous conditions (also including TB and leprosy) as a result of tissue being able to 1-alpha-hydroxylate 25(OH) D leading to vitamin D (the 1,25-dihydroxyvitamin D3) excess. Erythema nodosum, tender erythematous nodules and lupus pernio, indurated plaques with discoloration on the face, are typical skin manifestations of sarcoidosis which this patient has. CXR will typically show bilateral hilar lymphadenopathy and CXR findings are used in the staging of disease. Additionally, serum calcium and ACE levels may be raised. A transbronchial biopsy is essential for diagnosis in most cases and shows the presence of non-caseating granulomas. Black people have a higher lifetime risk of sarcoidosis, as do those of Scandinavian origin. The mainstay of treatment for severe disease involves systemic corticosteroids.

32
Q

A 49 year old lady presents to A&E with severe haemoptysis. She has a history of continuous production of foul-smelling khaki-coloured sputum. She had whooping cough as a child.

What is the diagnosis?
A.	Tuberculosis
B.	Bronchiectasis
C.	Bronchial carcinoma
D.	Primary pulmonary hypertension
E.	Pulmonary embolus
F.	Streptococcus pneumoniae
G.	Trauma
H.	Left ventricular failure
I.	Mycoplasma pneumoniae
J.	 Idiopathic pulmonary fibrosis
K.	Sarcoidosis
L.	COPD
A

B. Bronchiecstasis

Bronchiectasis is permanent bronchi dilatation due to bronchial wall damage and loss of elasticity. It is often as a consequence of recurrent/severe infections (whooping cough in this case caused by Bordetella pertussis) and most present with chronic productive mucopurulent cough. The most common identifiable cause is CF. Many patients will have crackles on auscultation of the lungs and high-pitched inspiratory squeaks and pops are also commonly heard. SOB is present in the majority of patients, especially with exertion (and is a good correlate of the severity on CT) and more than half will have recurrent episodes of fever, which is also a sign of an acute exacerbation. Daily sputum production is present in two thirds of patients and bloody sputum in half. The most common identifiable cause is CF and clubbing is a sign of bronchiectasis. Chest CT is the diagnostic test. Diagnosis is aided by sputum analysis.

33
Q

A 45 year old male from India, presents with a 3 week history of tiredness, loss of weight and haemoptysis. His left lung base is stony-dull to percussion.

What is the diagnosis?
A.	Tuberculosis
B.	Bronchiectasis
C.	Bronchial carcinoma
D.	Primary pulmonary hypertension
E.	Pulmonary embolus
F.	Streptococcus pneumoniae
G.	Trauma
H.	Left ventricular failure
I.	Mycoplasma pneumoniae
J.	 Idiopathic pulmonary fibrosis
K.	Sarcoidosis
L.	COPD
A

A. Tuberculosis

It is important to have a high level of suspicion when evaluating patients with risk factors who present with suggestive symptoms. Night sweats, fever, weight loss, malaise, cough, haemoptysis and erythema nodosum are all suggestive. In the first half of the 20th century, tuberculosis accounted for over 90% of cases of erythema nodosum. Other key risk factors for pulmonary TB include exposure to infection and returning from or being born in a high-risk region such as Asia, Africa and Latin America (India in this case). The examination findings here are consistent with a parapneumonic effusion. If TB is suspected, the patient should be placed in isolation and a CXR obtained with 3 sputum samples cultured for AFB being the gold standard of diagnosis. Culture takes several weeks so sputum smears will be done before culture results are known. Interferon-gamma release assays (IGRAs) are now used by some hospitals to rapidly determine a patient’s TB status. All patients who have TB should be tested for HIV within 2 months of diagnosis.

34
Q

A 60 year old patient presents with SOB and haemoptysis. O/E the patient is tachycardic, tachypnoeic with swollen ankles and bilateral basal crepitations

What is the diagnosis?
A.	Tuberculosis
B.	Bronchiectasis
C.	Bronchial carcinoma
D.	Primary pulmonary hypertension
E.	Pulmonary embolus
F.	Streptococcus pneumoniae
G.	Trauma
H.	Left ventricular failure
I.	Mycoplasma pneumoniae
J.	 Idiopathic pulmonary fibrosis
K.	Sarcoidosis
L.	COPD
A

H. Left ventricular failure

LVF causes congestion in the pulmonary circulation so the symptoms are respiratory with evidence here of pulmonary oedema. As seen in this patient, there is SOB and there may also be orthopnoea. This is why you can ask patients in a cardiac history how many pillows they sleep with. PND can also occur as well as ‘cardiac asthma’. RVF leads to a backlog of blood and congestion of the systemic capillaries. This causes peripheral oedema and ascites and hepatomegaly may develop. Nocturia may be a symptom as fluid returns from the legs when the patient lies down flat. This patient does have peripheral oedema too so technically has CCF (congestive cardiac failure), but the best option on the list is LVF.

35
Q

A 69 year old lady presents with a sudden onset of fever and coughing up a purulent, rusty coloured sputum. Examination of her chest showed signs of consolidation.

What is the diagnosis?
A.	Tuberculosis
B.	Bronchiectasis
C.	Bronchial carcinoma
D.	Primary pulmonary hypertension
E.	Pulmonary embolus
F.	Streptococcus pneumoniae
G.	Trauma
H.	Left ventricular failure
I.	Mycoplasma pneumoniae
J.	 Idiopathic pulmonary fibrosis
K.	Sarcoidosis
L.	COPD
A

F. Streptococcus pneumoniae

The rusty coloured sputum is hinting at a pneumococcal pneumonia.The patient has presented with common symptoms of fever and a productive cough. Examination findings are also consistent – have a think about what would actually be found while performing a respiratory examination on this patient. There may also be SOB, chills, rigors and pleuritic chest pain. The most specific and sensitive test is a CXR (PA and lateral) and initial treatment of a CAP is empirical with antibiotics. Often diagnosis is made solely on history and examination findings. Management is guided by the patient’s CURB-65 score.

36
Q

A 21 year old male has a 3 day history of hoarseness. He has pain in his throat which is worse on talking and eating. O/E his throat appears normal.

What is the diagnosis?
A.	Laryngitis
B.	Acromegaly
C.	Vocal cord nodules
D.	Sjogren's syndrome
E.	Hypothyroidism
F.	Wegener's syndrome
G.	Carcinoma of the larynx
H.	Angioedema
I.	Laryngeal nerve palsy
J.	Foreign body
A

A. Laryngitis

Laryngitis, as the name would suggest, is inflammation of the larynx, which can lead to oedema of the true vocal folds. It has both infectious and noninfectious causes such as vocal strain. Symptoms of acute disease are most commonly hoarseness, generally over a period of less than a week, usually preceded by viral URTI and usually self limiting. The pain on swallowing and sore throat is common of URTIs. An exudate or cervical lymphadenitis would suggest bacterial infection instead. Treatment begins, as always, with ABC and airway assessment. Chronic laryngitis presents with hoarseness lasting more than 3 weeks and this needs investigating due to the fact that symptoms may be similar to cancer of larynx. Antibiotics are given in bacterial cases or otherwise voice rest and hydration is sufficient.

37
Q

A 25 year old man suddenly developed hoarseness, wheeze and stridor whilst eating peanuts in a bar. Looking in his mouth you notice a swollen tongue.

What is the diagnosis?
A.	Laryngitis
B.	Acromegaly
C.	Vocal cord nodules
D.	Sjogren's syndrome
E.	Hypothyroidism
F.	Wegener's syndrome
G.	Carcinoma of the larynx
H.	Angioedema
I.	Laryngeal nerve palsy
J.	Foreign body
A

H. Angioedema

Angio-oedema is swelling involving the deeper layers of the subdermis (occuring in association with urticaria in around 40% of cases). In this case it involves the face/neck and is dangerous – the risk being airway compromise and this requires rapid treatment with adrenaline. The food trigger here is obvious – this person has just consumed peanuts and this is allergic in nature, causing an IgE mediated reaction. Common triggers aside from nuts include eggs and shellfish although any food can be implicated. As mentioned, airway management and adrenaline is crucial here and you would also give antihistamines, IV corticosteroids and tell the patient to avoid the trigger.

38
Q

A 55 year old woman develops hoarseness 2 days after a partial thyroidectomy for thyrotoxicosis.

What is the diagnosis?
A.	Laryngitis
B.	Acromegaly
C.	Vocal cord nodules
D.	Sjogren's syndrome
E.	Hypothyroidism
F.	Wegener's syndrome
G.	Carcinoma of the larynx
H.	Angioedema
I.	Laryngeal nerve palsy
J.	Foreign body
A

I. Laryngeal nerve palsy

This patient has just had neck surgery and the hoarseness here results from damage to the recurrent laryngeal nerve, which is a branch of the vagus nerve which supplies motor function and sensation to the larynx. This nerve runs posterior to the thyroid and results in hoarseness when damaged. Bilateral damage is even worse and the patient could have difficulty breathing and the complete inability to speak. The right recurrent laryngeal is more prone to damage as it is located relatively more medial than the left.

39
Q

A 58 year old male smoker has a 2 month history of progressive persistent hoarseness and pain in his left ear on swallowing. He has enlarged left cervical lymph nodes.

What is the diagnosis?
A.	Laryngitis
B.	Acromegaly
C.	Vocal cord nodules
D.	Sjogren's syndrome
E.	Hypothyroidism
F.	Wegener's syndrome
G.	Carcinoma of the larynx
H.	Angioedema
I.	Laryngeal nerve palsy
J.	Foreign body
A

G. Carcinoma of the larynx

Laryngeal cancer is frequently associated with two big risk factors: smoking and alcohol use. Major signs are persistent (>3 weeks) of hoarseness, dysphonia, sore throat, dysphagia, referred otalgia (seen here) and neck masses/adenopathy. These patients need to be evaluated by a specialist and as mentioned this can resemble chronic laryngitis in presentation. An MDT approach is taken to try and preserve the organ as much as possible, with salvage surgical resection offered in advanced disease.

40
Q

A 40 year old woman develops a progressively hoarse voice over 6 months. She has also gained 8kg in weight and complains of constipation.

What is the diagnosis?
A.	Laryngitis
B.	Acromegaly
C.	Vocal cord nodules
D.	Sjogren's syndrome
E.	Hypothyroidism
F.	Wegener's syndrome
G.	Carcinoma of the larynx
H.	Angioedema
I.	Laryngeal nerve palsy
J.	Foreign body
A

E. Hypothyroidism

This woman is in a hypothyroid state. The hoarseness may be a symptom of a goitre (enlarged thyroid). Worldwide, the most common cause is iodine deficiency. Other causes include Hashimoto’s or secondary and tertiary hypothyroidism. It can also result from viral de Quervain’s thyroiditis or postpartum thyroiditis. Symptoms include those mentioned as well as depression, bradycardia, sluggish reflexes, cold intolerance and muscle cramps. Diagnosis is based on measurement of TSH and thyroid hormones. Treatment is by replacement of T4 with or without T3 in combination. If the patient has normal T3 and T4 but mildly elevated TSH, this is described as subclinical hypothyroidism.

41
Q

A 43 year old man with a long history of excess alcohol consumption presents with haematemesis. O/E his is clubbed and has spider naevi.

What is the most suitable investigation?
A.	Serum sodium
B.	U&E
C.	Bronchoscopy
D.	Echocardiogram
E.	TFT
F.	Lung(pulmonary) function tests
G.	Abdominal ultra sound scan (USS)
H.	Colonoscopy
I.	FBC
J.	Liver function tests
K.	Stool culture
L.	Sputum culture
M.	CXR
A

G. Abdominal ultra sound scan (USS)

This is hepatic cirrhosis leading to clubbing. The haematemesis is secondary to oesophageal varices and he has signs of chronic liver disease. LFTs are non-specific however ultrasound can show signs of advanced cirrhosis. There may be liver surface nodularity (remember cirrhosis entails nodular regeneration), small liver and possible left/caudate lobe hypertrophy. Signs of portal hypertension may also be picked up such as ascites, splenomegaly and increased diameter of the hepatic portal vein. The most specific and sensitive test is a liver biopsy, which is not given as an option here. Additionally abdominal CT or MRI can also be done similar to USS, with similar findings.

42
Q

A 47 year old demolition expert presents with SOB. O/E he has some clubbing and signs of pleural effusion.

What is the most suitable investigation?
A.	Serum sodium
B.	U&E
C.	Bronchoscopy
D.	Echocardiogram
E.	TFT
F.	Lung(pulmonary) function tests
G.	Abdominal ultra sound scan (USS)
H.	Colonoscopy
I.	FBC
J.	Liver function tests
K.	Stool culture
L.	Sputum culture
M.	CXR
A

M. CXR

Possible mesothelioma. Possible pulmonary fibrosis. Either way, CXR is the way to go.

43
Q

A 19 year old woman who had cardiac surgery as a child, presents with decreasing exercise tolerance. O/E she is cyanosed and has clubbed nails.

What is the most suitable investigation?
A.	Serum sodium
B.	U&E
C.	Bronchoscopy
D.	Echocardiogram
E.	TFT
F.	Lung(pulmonary) function tests
G.	Abdominal ultra sound scan (USS)
H.	Colonoscopy
I.	FBC
J.	Liver function tests
K.	Stool culture
L.	Sputum culture
M.	CXR
A

D. Echocardiogram

The cause here is cardiac and likely Eisenmenger’s syndrome. This is clubbing due to congenital cyanotic heart disease and you would want to perform an echocardiogram here to look for structural heart defects.

44
Q

A 35 year old woman with a history of recurrent lower abdominal pain, bloody diarrhoea and passing mucus PR. O/E she has lower abdominal tenderness and clubbing.

What is the most suitable investigation?
A.	Serum sodium
B.	U&E
C.	Bronchoscopy
D.	Echocardiogram
E.	TFT
F.	Lung(pulmonary) function tests
G.	Abdominal ultra sound scan (USS)
H.	Colonoscopy
I.	FBC
J.	Liver function tests
K.	Stool culture
L.	Sputum culture
M.	CXR
A

H. Colonoscopy

This is clubbing due to inflammatory bowel disease (Crohn’s or UC) and diagnosis of this would be made on colonoscopy. Biopsy in CD will show transmural granulomatous inflammation. CD can affect the whole GIT but favours the TI (RLQ pain) and proximal colon and is macroscopically characterised by skip lesions. UC on the other hand is characterised by the presence of crypt abscesses, which is pathognomic.

45
Q

A 49 year old heavy smoker presents with SOB and weight loss. O/E she is clubbed and her CXR shows a perihilar shadow.

What is the most suitable investigation?
A.	Serum sodium
B.	U&E
C.	Bronchoscopy
D.	Echocardiogram
E.	TFT
F.	Lung(pulmonary) function tests
G.	Abdominal ultra sound scan (USS)
H.	Colonoscopy
I.	FBC
J.	Liver function tests
K.	Stool culture
L.	Sputum culture
M.	CXR
A

C. Bronchoscopy

This is likely lung cancer and diagnosis relies on pathological confirmation from a tissue sample, often obtained from bronchoscopy. A CXR will of course be done in the first instance as the initial investigation but the best test on the list for diagnosis is a bronchoscopy.

46
Q

A 50 year old woman has developed weight loss and passes loose pale stools. She has mouth ulcers and is anaemic. She is taking thyroxine for myxoedema.

What is the most suitable investigation?
A.	ERCP
B.	Faecal elastase-1
C.	Abdominal ultrasound
D.	Liver function tests
E.	HIV test
F.	Skin biopsy
G.	Immunoglobulin
H.	Endomysial antibodies
I.	Sweat test
J.	Hydrogen breath test
K.	Abdominal X-ray
L.	Thyroid function
M.	Colonoscopy
A

H. Endomysial antibodies

This is a common condition in the US and Europe. Coeliac disease most commonly presents with IDA, although it can also lead to a macrocytic anaemia with mainly folate deficiency (though B12 is also affected but hepatic stores last several years). The mouth ulcers are a sign of this. There are also GI symptoms resulting from malabsorption. It is an autoimmune condition (the presence of another autoimmune condition here is a risk factor) triggered by gluten peptides found in wheat, rye and barley. The ultimate best test is duodenal biopsy and histology to show intra-epithelial lymphocytes, villous atrophy and crypt hyperplasia. Macroscopic changes may be present but endoscopy is generally unhelpful. The test of choice before performing such an invasive confirmatory test is to look for elevated anti-gliadin antibodies. Anti-tissue transglutaminase is less accurate and endomysial antibody is more expensive and has lower sensitivity, though is the only option on this list specific for coeliac.
It is worth knowing about the Schilling test as it is frequently examined. However, it is no longer routinely done in clinical practice. In this test, IM vitamin B12 is given to saturate stores. Then oral radiolabelled B12 is given and urine is collected over 24 hours. The amount excreted is lower in B12 malabsorption. If this is not corrected by IF the problem is with the ileum and not inadequate IF.

47
Q

A 35 year old man presents with weight loss, diarrhoea and pain on swallowing. O/E he has oral candidiasis & molluscum contagiosum.

What is the most suitable investigation?
A.	ERCP
B.	Faecal elastase-1
C.	Abdominal ultrasound
D.	Liver function tests
E.	HIV test
F.	Skin biopsy
G.	Immunoglobulin
H.	Endomysial antibodies
I.	Sweat test
J.	Hydrogen breath test
K.	Abdominal X-ray
L.	Thyroid function
M.	Colonoscopy
A

E. HIV test

Molluscum contagiosum is an umbilicated, pearl like, smooth papules and at least a third of patients will also develop symptoms of local erythema, swelling or pruritis. HIV is a strong risk factor for MC. It is transmitted skin-to-skin or fomite in children and sexually transmitted in adulthood. Oral candidiasis is also an opportunistic infection associated with the immunocompromised state of HIV. Weight loss is also seen in HIV and may result from malnutrition, TB or HIV wasting syndrome. Diarrhoea is also common. An HIV test is needed in this patient and contact tracing will be necessary.

48
Q

A 10 year old girl with a history of recurrent chest infections has developed pale floating stools and weight loss.

What is the most suitable investigation?
A.	ERCP
B.	Faecal elastase-1
C.	Abdominal ultrasound
D.	Liver function tests
E.	HIV test
F.	Skin biopsy
G.	Immunoglobulin
H.	Endomysial antibodies
I.	Sweat test
J.	Hydrogen breath test
K.	Abdominal X-ray
L.	Thyroid function
M.	Colonoscopy
A

I. Sweat test

CF is autosomal recessive and the mean age of death is around 40. There is currently no cure for this condition. The reccurent chest infections and greasy stools (fat malabsorption due to pancreatic insufficiency) should make you think of CF. A persistent cough which is productive would also raise suspicions. Additionally, you may find nasal polyps and hepatomegaly and/or splenomegaly and a congenital absence of the vas deferens in males. There may also be failure to thrive in infants. The most conclusive diagnostic test is the sweat test which is positive if sweat chloride is >60mmol/L. Serum IRT from a heel prick blood spot allows screening of newborns. CF is a genetic condition with abnormal salt and water transport due to mutations in the CFTR (an apical anion channel). Heterozygotes generally do not demonstrate disease.

49
Q

A 45 year old man has recurrent epigastric pain, weight loss and steatorrhoea. He has a previous history of alcoholism.

What is the most suitable investigation?
A.	ERCP
B.	Faecal elastase-1
C.	Abdominal ultrasound
D.	Liver function tests
E.	HIV test
F.	Skin biopsy
G.	Immunoglobulin
H.	Endomysial antibodies
I.	Sweat test
J.	Hydrogen breath test
K.	Abdominal X-ray
L.	Thyroid function
M.	Colonoscopy
A

A. ERCP

This is chronic pancreatitis which is most commonly associated with chronic alcohol abuse. Features include the epigastric pain here, which classically radiates to the back, and steatorrhoea from malabsorption (pale, foul-smelling and difficult to flush stools). There may additionally be DM due to pancreatic failure and the patient may be malnourished. The diagnosis is based on findings and imaging – your options are USS which is less sensitive, or CT, which is more sensitive but involves radiation exposure. AXR is not a sensitive enough test. However, this question is looking for the best test which is ERCP, commonly considered the most accurate test with high sensitivity and specificity. It is limited in use though due to cost and the risk to the patient. Characteristically ERCP would show beading of the main pancreatic duct as well as irregularities in the side branches. Faecal elastase-1 is inaccurate for diagnosing mild to moderate pancreatic insufficiency, and anyway has unacceptably low sensitivity.
There is no real definitive treatment, which is mainly symptomatic and the underlying and precipitating factors are treated – in this case, this man’s alcohol excess. Complications of chronic pancreatic imflammation include the development of pseudocysts, calficiation, DM and malabsorption.

50
Q

A 20 year old man has a 4 month history of diarrhoea & weight loss. He is pale, has mouth ulcers and clubbing.

What is the most suitable investigation?
A.	ERCP
B.	Faecal elastase-1
C.	Abdominal ultrasound
D.	Liver function tests
E.	HIV test
F.	Skin biopsy
G.	Immunoglobulin
H.	Endomysial antibodies
I.	Sweat test
J.	Hydrogen breath test
K.	Abdominal X-ray
L.	Thyroid function
M.	Colonoscopy
A

M. Colonoscopy

This is a history of inflammatory bowel disease. A colonoscopy is required to assess the extent of disease and for a definitive diagnosis. Biopsy in CD will show transmural granulomatous inflammation. CD can affect the whole GIT but favours the TI (RLQ pain) and proximal colon and is macroscopically characterised by skip lesions. UC on the other hand is characterised by the presence of crypt abscesses, which is pathognomic.

51
Q

A 50 year old housewife presents with pruritis and jaundice with pale stools, dark urine and steatorrhea, pigmentation and xanthelasma. Examination reveals splenomegaly. Anti-mitochondrial antibodies are present.

What is the diagnosis?
A.	Hepatitis
B.	Gall stones
C.	Gilbert's syndrome
D.	Primary sclerosing cholangitis
E.	Haemolytic anaemia
F.	Dubin-Johnson syndrome
G.	Carcinoma of the pancreas
H.	Primary biliary cirrhosis
A

H. Primary biliary cirrhosis

Primary biliary cirrhosis (PBC) is a chronic condition where the intrahepatic small bile ducts are progressively damaged (and eventually lost) occuring on a background of portal tract inflammation. Fibrosis develops, ultimately leading to cirrhosis (which is defined as fibrosis with nodular regeneration). It is widely believed to be autoimmune in aetiology as almost all patients have AMA (present here). The pointers in this question which would raise your suspicion, is xanthelasma around the eyes, pruritis in the absence of an obvious dermatological cause, fatigue and the features of liver disease typical of cirrhosis (obstructive jaundice) and splenomegaly as a feature of portal hypertension.

52
Q

A 25 year old man presents to you with an incidental finding of a raised bilirubin (31 umol). No other signs of liver disease are present. Further investigations show a raised unconjugated bilirubin. When asked he tells you that other family members have suffered from jaundice.

What is the diagnosis?
A.	Hepatitis
B.	Gall stones
C.	Gilbert's syndrome
D.	Primary sclerosing cholangitis
E.	Haemolytic anaemia
F.	Dubin-Johnson syndrome
G.	Carcinoma of the pancreas
H.	Primary biliary cirrhosis
A

C. Gilbert’s syndrome

Gilbert’s occurs in an asymptomatic patient, often as an incidental finding or mild jaundice occuring in adolescence/young adult age. There is elevated unconjugated BR with other liver tests being normal. The blood smear is also normal with normal reticulocyte count, and normal Hb indicating that this is not due to haemolysis. It is a common syndrome and is not really a disease, more a physiological variant. No treatment is needed and this condition is due to decreased UDPGT activity leading to decreased conjugation of unconjugated bilirubin, leading to elevated levels. Positive FH is common as this condition is most likely transmitted in an autosomal recessive pattern.

53
Q

A 65 year old ex-smoker is deeply jaundiced. He has epigastric pain radiating to his back. A dilated gall bladder is palpable and there is hepatomegaly. He has lost about 5kg in weight.

What is the diagnosis?
A.	Hepatitis
B.	Gall stones
C.	Gilbert's syndrome
D.	Primary sclerosing cholangitis
E.	Haemolytic anaemia
F.	Dubin-Johnson syndrome
G.	Carcinoma of the pancreas
H.	Primary biliary cirrhosis
A

G. Carcinoma of the pancreas

Pancreatic cancer (of the head) typically presents with painless obstructive jaundice and weight loss and generally presents late. There is however epigastric pain in this case, which is a possible presentation. Whipple’s procedure or Traverso-Longmire procedure (pancreaticoduodenectomy) offers the only hope of a cure but only a small minority are elegible for these procedures. The first tests to order are an abdominal USS and LFTs. Note Courvoisier’s law: Jaundice and a palpable painless gallbladder is unlikely to be caused by gallstones. The tumour marker for pancreatic cancer is CA19-9 which is useful in preoperative staging.

54
Q

A 22 year old man comes to see you on his return from a holiday in Spain. He has a 3-4 day history of fever, malaise, nausea, vomiting and abdominal discomfort. He is noticeably jaundiced with dark urine and pale stools. There is also tender hepatomegaly on examination. He wonders if this is related to his meal of shellfish from a street vendor.

What is the diagnosis?
A.	Hepatitis
B.	Gall stones
C.	Gilbert's syndrome
D.	Primary sclerosing cholangitis
E.	Haemolytic anaemia
F.	Dubin-Johnson syndrome
G.	Carcinoma of the pancreas
H.	Primary biliary cirrhosis
A

A. Hepatitis A

This is likely hepatitis A which is primarily transmitted via the faecal-oral route. After the virus is consumed and absorbed, it replicates in the liver and is excreted in the bile (to be re-transmitted). Transmission usually precedes symptoms by about 2 weeks and patients are non-infectious 1 week after onset of jaundice. The history can reveal risk factors such as living in an endemic area, contact with an infected person, homosexual sex or a known food-borne outbreak. This is classically, in EMQs, associated with shellfish which is harvested from sewage contaminated water. If the patient has other liver diseases such as HBV or HCV or cirrhosis then there is a higher risk of fulminant HAV infection. The clinical course of HAV consists of a pre-icteric phase, lasting 5-7 days, consisting characteristically of N&V, abdominal pain, fever, malaise and headache. Rarer symptoms may be present such as arthralgias and even severe thrombocytopenia and signs that may be found include splenomegaly, RUQ tenderness and tender hepatomegaly as well as bradycardia. The icteric phase is characterised by dark urine, pale stools, jaundice and pruritis. When jaundice comes on, the pre-icteric phase symptoms usually diminish, and jaundice typically peaks at 2 weeks. However, a fulminant course runs in <1% of patients with worsenining jaundice and encephalopathy. Serum transaminases may reach in excess of 10,000 units, although there is little correlation between the level and disease severity. ALT is typically higher than AST.

55
Q

A 6 year old presents with mild jaundice and some pain and swelling of his fingers. O/E you note splenomegaly.

What is the diagnosis?
A.	Hepatitis
B.	Gall stones
C.	Gilbert's syndrome
D.	Primary sclerosing cholangitis
E.	Haemolytic anaemia
F.	Dubin-Johnson syndrome
G.	Carcinoma of the pancreas
H.	Primary biliary cirrhosis
A

E. Haemolytic anaemia

Africans have higher incidence of sickle cell anaemia. This is a presentation of bone pain here with dactylitis, consistent with hand-foot syndrome which can be what young infants and children present with (it is often a child’s first presentation of disease). The jaundice here is due to haemolysis and so while this is sickle cell anaemia, the options are trying to get you to think a bit about the best fit here which would be haemolytic anaemia. About 8% of black people carry the gene and the prevalence is high in sub-Saharan Africa. The condition is autosomal recessive and therefore occurs in 1 in 4 pregnancies where both parents carry the sickle gene. Sickling occurs when RBCs containing HbS become distorted into a crescent shape. Patients with sickle cell anaemia have no HbA at all. If both parents carry the sickle cell gene, there is a 1 in 4 chance of giving birth to a child with sickle cell anaemia. Sickle cell disease also includes other conditions such as HbS from one parent with another abnormal Hb or beta thalassaemia from the other parent such as HbS-Beta thal and HbSC. Treatment goals here include fluid replacement therapy, pain management and symptomatic control.

56
Q

A 25 year old female with recent onset of depression takes 50 paracetamol capsules, each containing 500mg of active drug. In several days her liver is most likely to show what?

A.	Extensive cirrhosis
B.	 Enlarged right lobe
C.	 Pancreatic carcinoma
D.	Hypervascularity
E.	Hepatitis B
F.	Cholecystitis
G.	 Arterio-venous malformations
H.	Hepatocellular carcinoma
I.	Extensive necrosis
J.	Hepatitis C
K.	 Pancreatic pseudocyst
L.	Portal chronic inflammation
A

I. Extensive necrosis

Paracetamol OD can occur after a single large OD or repeated ODs. Often, the patient is asymptomatic at initial presentation but if untreated may cause liver injury over the 2-4 days after ingestion, including fulminant liver failure. Massive hepatic necrosis will occur and will be the cause of fulminant liver failure. Paracetamol is the most frequent intentional OD drug in this country. The risk of liver damage is increased after taking drugs which induce CYP 450. Inducers include St John’s wort, barbiturates, phenytoin, tetracycline, chronic alcohol use and carbamazepine. A serum paracetamol level is important to order as early as possible, but at the earliest 4 hours post-ingestion.Treatment if indicated is with N-acetylcysteine with the level based on a paracetamol treatment graph.

57
Q

A 40 year old male has a long history of alcoholism. His liver is firm on palpation. You finally convince him to stop drinking and join Alcoholics Anonymous. Despite abstinence he remains at risk for developing which disease?

A.	Extensive cirrhosis
B.	 Enlarged right lobe
C.	 Pancreatic carcinoma
D.	Hypervascularity
E.	Hepatitis B
F.	Cholecystitis
G.	 Arterio-venous malformations
H.	Hepatocellular carcinoma
I.	Extensive necrosis
J.	Hepatitis C
K.	 Pancreatic pseudocyst
L.	Portal chronic inflammation
A

H. Hepatocellular carcinoma

Patients with cirrhosis, especially those with alcoholic liver disease, are at a high risk of developing HCC. Cirrhosis is irreversible so despite stopping drinking, he is still at risk of HCC (hepatoma). Patients with cirrhosis should be screened for HCC with serum AFP and USS at 6 month intervals.

58
Q

A 40-year-old female, rather overweight, has episodes of right upper quadrant pain and fever.

What is the diagnosis?
A.	Extensive cirrhosis
B.	 Enlarged right lobe
C.	 Pancreatic carcinoma
D.	Hypervascularity
E.	Hepatitis B
F.	Cholecystitis
G.	 Arterio-venous malformations
H.	Hepatocellular carcinoma
I.	Extensive necrosis
J.	Hepatitis C
K.	 Pancreatic pseudocyst
L.	Portal chronic inflammation
A

F. Cholecystitis

Cholecystitis is acute GB inflammation caused by an obstruction at the cystic duct. It occurs as a major complication of gallstones and classically presents with RUQ pain and fever. Gallstones in EMQs classically involves the Fs (Fat, Forty, Female, Fertile, Fair). USS is the definitive initial investigation. HIDA scanning and MRI may help if the diagnosis remains unclear. Treatment is with cholecystectomy.

59
Q

A 60 year old alcoholic is hospitalised after an episode of haematemesis. He is about to undergo endoscopy. What would his liver biopsy likely low evidence of?

A.	Extensive cirrhosis
B.	 Enlarged right lobe
C.	 Pancreatic carcinoma
D.	Hypervascularity
E.	Hepatitis B
F.	Cholecystitis
G.	 Arterio-venous malformations
H.	Hepatocellular carcinoma
I.	Extensive necrosis
J.	Hepatitis C
K.	 Pancreatic pseudocyst
L.	Portal chronic inflammation
A

A. Extensive cirrhosis

This is a case of haematemesis secondary to oesophageal varices. Oesophageal varices are a direct result of portal hypertension, which occurs as a progressive complication of cirrhosis, which is what liver biopsy will show. Diagnosis and surveillance by endoscopy is an important part of management of this condition and in terms of prophylaxis against variceal bleeding before it has occured, non-selective beta blockers and/or endoscopic ligation can be used.

60
Q

On liver biopsy a moderate chronic inflammation is observed. Special stains identify antigens from a double stranded DNA virus within the cytoplasm of hepatocytes.

What is the diagnosis?
A.	Extensive cirrhosis
B.	 Enlarged right lobe
C.	 Pancreatic carcinoma
D.	Hypervascularity
E.	Hepatitis B
F.	Cholecystitis
G.	 Arterio-venous malformations
H.	Hepatocellular carcinoma
I.	Extensive necrosis
J.	Hepatitis C
K.	 Pancreatic pseudocyst
L.	Portal chronic inflammation
A

E. Hepatitis B

Hepatitis B is a DNA virus which is transmitted percutaneously and permucosally. It is also a STI. HCV is an RNA virus and RNA-PCR will be positive. A brief bit about hepatitis B markers: HBsAb appears several weeks after HBsAg disappears and in most patients suggests a resolved infection and life-long immunity (it is also detectable and titres are measured in those immunised with the HBV vaccine). HBsAg on the other hand appears 2-10 weeks after exposure to HBV and usually, in self-limiting acute cases, becomes undetectable after 4-6 months of infection. Persistence for >6 months implies chronic infection. Core antibody (IgM) appears within weeks of acute infection and remains detectable for 4-8 months and can be the only way to diagnose acute infection during the period when surface antigen disappears but before surface antibody has appeared. Chronic infection is indicated by IgG core antibody. The best single test to screen household contacts of infected individuals to determine the need to vaccinate is still HBcAb. E antigen is a soluble viral protein in serum which is part of the early acute infection and disappears soon after peak ALT levels. Presence >3 months indicates chronic infection is likely. E antigen being present in those with surface antigen indicates greater infectivity and a high level of viral activity and replication.

61
Q

A 50 year old man took an overdose of his antidepressants one hour ago. He has a dry mouth and dilated pupils but is not drowsy.

What is the best treatment?
A.	N-acetlycysteine
B.	IV-ethanol
C.	Gastric lavage
D.	IV-glucagon
E.	Activated charcoal
F.	Forced emesis
G.	Haemodialysis
H.	IV-naloxone
I.	Forced alkaline diuresis
J.	Oral methionine
K.	Hyperbaric oxygen
A

E. Activated charcoal

This is an overdose of tricyclic antidepressants which are a class of drugs with a narrow therapeutic index and therefore become potent toxins in moderate doses to both the CNS and cardiovascular system. The main aim in treatment is to provide respiratory and cardiovascular support until the medicine has been fully metabolised and eliminated. GI decontamination should be considered in those presenting with early overdose (under 2 hours after ingestion) provided that the airway can be protected. There is no shown clear benefit to repeated doses of activated charcoal.
The warm, dry skin is part of the anticholinergic effects (physostigmine should NOT be used to reverse this as it has been in rare cases been associated with asystole – would you rather have a patient who is flushed or flatlined?). Other anticholinergic effects include dilated pupils, urinary retention, decreased or absent bowel sounds and changes in mental status. Hypotension is common and is due to alpha 1 antagonism. Classic ECG changes are of sinus tachycardia progressing to wide complex tachycardia and ventricular arrhythmias (with increasing severity and intoxication). Condution problems and hypotension is improved with hypertonic sodium bicarbonate and if arrhythmias are present, treatment of these involves correcting the acidosis, hypoxia and electrolyte abnormalities. Anti-arrhythmics are generally avoided. If hypotension is refractory then a vasopressor can be used. BZDs can be used for any seizures.
62
Q

A 30 year old woman with toothache has taken 50 paracetamol 500mg tablets in the last 24 hours. She feels nauseated and still has toothache but is otherwise well.

What is the best treatment?
A.	N-acetlycysteine
B.	IV-ethanol
C.	Gastric lavage
D.	IV-glucagon
E.	Activated charcoal
F.	Forced emesis
G.	Haemodialysis
H.	IV-naloxone
I.	Forced alkaline diuresis
J.	Oral methionine
K.	Hyperbaric oxygen
A

A. N-acetylcysteine

Paracetamol OD can occur after a single large OD or repeated ODs. Often, the patient is asymptomatic at initial presentation but if untreated may cause liver injury over the 2-4 days after ingestion, including fulminant liver failure. Paracetamol is the most frequent intentional OD drug in this country. The risk of liver damage is increased after taking drugs which induce CYP 450. Inducers include St John’s wort, barbiturates, phenytoin, tetracycline, chronic alcohol use and carbamazepine. A serum paracetamol level is important to order as early as possible, but at the earliest 4 hours post-ingestion.Treatment if indicated is with N-acetylcysteine with the level based on a paracetamol treatment graph.

63
Q

A 20 year old heroin addict arrives in casualty, unconscious and cyanosed. His resp rate is 6/min and he has pin-point pupils.

What is the best treatment?
A.	N-acetlycysteine
B.	IV-ethanol
C.	Gastric lavage
D.	IV-glucagon
E.	Activated charcoal
F.	Forced emesis
G.	Haemodialysis
H.	IV-naloxone
I.	Forced alkaline diuresis
J.	Oral methionine
K.	Hyperbaric oxygen
A

H. IV-naloxone

Signs of opiate OD include CNS depression, miosis (pinpoint pupils) and apnoea. Naloxone is indicated both therapeutically and diagnostically. If there is a response, then it is diagnostic. Another diagnosis should be sought if the patient is unresponsive. IV is the preferred route of administration although naloxone can be given IM or SC if IV access cannot be established. Ventilatory support is key with 100% oxygen.

64
Q

A 40 year old woman with a history of hypertension was brought in to casualty two hours ago having taken a whole bottle of her medication in an attempt to commit suicide. She suddenly collapses with a pulse of 30bpm and a BP of 70/30.

What is the best treatment?
A.	N-acetlycysteine
B.	IV-ethanol
C.	Gastric lavage
D.	IV-glucagon
E.	Activated charcoal
F.	Forced emesis
G.	Haemodialysis
H.	IV-naloxone
I.	Forced alkaline diuresis
J.	Oral methionine
K.	Hyperbaric oxygen
A

D. IV-glucagon

Glucagon stimulates adenyl cyclase which acts to increase intracellular cAMP and to therefore increase cytosolic calcium and cardiac contractility. Hypotension and bradyarrhythmias are the most common initial findings of beta blocker toxicity. If IV glucogon is not available then high-dose insulin can be used instead with co-administration of dextrose to maintain blood glucose levels.

65
Q

A 45 year old homeless man complains of headache, abdominal pain, nausea and dizziness. He admits to having drunk anti-freeze on the previous night. He is hyperventilating and slightly drowsy.

What is the best treatment?
A.	N-acetlycysteine
B.	IV-ethanol
C.	Gastric lavage
D.	IV-glucagon
E.	Activated charcoal
F.	Forced emesis
G.	Haemodialysis
H.	IV-naloxone
I.	Forced alkaline diuresis
J.	Oral methionine
K.	Hyperbaric oxygen
A

B. IV-ethanol

Antifreeze is ethylene glycol. It is a sweet-tasting, odourless and colourless liquid and the substance itself is non-toxic and initially causes inebriation. Toxicity appears within 12-24 hours and is due to metabolic acidosis and the formation of calcium oxalate from one of the metabolites. Oxalate deposits in the lungs, myocardium and kidneys leading to organ damage and renal failure, and hypocalcaemia may also occur due to the consumption of circulating calcium. Ethylene glycol is not absorbed by activated charcoal and gastric decontamination is pointless regardless of time since consumption. The first line treatment is fomepizole (4-methylpyrazole) which is a competitive inhibitor of alcohol dehydrogenase, an enzyme involved in catalysing the initial steps in metabolism of ethylene glycol and methanol into toxic metabolites. However, this is not on the list, and oral ethanol (loading dose or infusion) can be used in this case (have a think about why ethanol would work as an antidote, if you think back to how ethanol is metabolised). Dialysis may well be required.

66
Q

A 23 year old woman presents with a 24 hour history of right iliac fossa pain. There is tenderness & guarding in the right iliac fossa. There are no menstrual symptoms. Abdominal & pelvic ultrasound is normal.

What is the most suitable investigation?
A.	ECG
B.	 PR exam
C.	Diagnostic laparotomy
D.	CT scan
E.	 AXR
F.	Endoscopy
G.	 CXR
H.	Laxatives
I.	Oral antibiotics
J.	Ultrasound scan
K.	Palliative care
L.	Acute pancreatitis
A

C. Diagnostic laparotomy

An abdominal and pelvic CT scan would normally be ordered in situations like this, with possible appendicitis, but it is assumed here that the doctors were thinking of pregnancy as a possibility so a sonogram was done instead, which turns out to be inconclusive. Now, in this situation, you could do an abdominal MRI (especially in early pregnancy) or go ahead with a CT scan anyway, but a diagnostic laparotomy is the best option here to diagnose and treat at the same time. The main differential here is either obstetric, such as a ruptured ectopic pregnancy, or acute appendicitis. You would have imagined they would have done the usual important tests like FBC and a urinary pregnancy test… but these results are not available, nor are they an option, but a prudent doctor would have ordered them in the diagnostic work up. However, given USS does not show a mass in the fallopian tubes, this may push you away from an ectopic pregnancy… however, USS is operator dependent, this is a female of childbearing age (this age is getting lower and lower in the UK) and the doctor has failed to obtain either serum or urine HCG levels or asked about any missed menstrual periods.
This patient, should be made NBM with maintenance IV fluids like lactated Ringer’s, and have a laparotomy which can be both diagnostic and therapeutic. You can take the appendix out if this is the problem or deal with the ectopic, if that is the problem, or deal with whatever it could be.. say for instance, a rare Meckel’s diverticulitis.

67
Q

A 30 year old man presents with severe left loin pain, colicky in nature. It radiates towards the left groin.

What is the most suitable investigation?
A.	ECG
B.	 PR exam
C.	Diagnostic laparotomy
D.	CT scan
E.	 AXR
F.	Endoscopy
G.	 CXR
H.	Laxatives
I.	Oral antibiotics
J.	Ultrasound scan
K.	Palliative care
L.	Acute pancreatitis
A

D. CT scan

This patient has renal colic which classically presents with severe flank pain radiating to the groin. Infection may complicate renal calculi. Microscopic haematuria is present in up to 90% of cases. Up to 85% of stones are visible on a plain KUB although urate stones are radiolucent. If the stone is radio-opaque, calcification will be seen within the urinary tract. In pregnancy though, a renal USS is first line. The IVP has now been replaced by the CT scan which is the new diagnostic standard. A non-contrast helical (or spiral) CT is preferred due to high sensitivity and specificity and acurately determines presence, site and size of stones. Stones are analysed after they are extracted or when they are expelled to check their composition.
In reality the first test to order is urinalysis – it is quick and simple. You’d also always exclude ectopic pregnancy with a urine pregnancy test if this were a female of childbearing age, look for hypercalcaemia and hyperuricaemia and perform a FBC to look for infection.

68
Q

A 45 year old man presents with sudden onset epigastric pain, constant in nature. He has had several previous episodes. He drinks half a bottle of whisky per day.

What is the most suitable investigation?
A.	ECG
B.	 PR exam
C.	Diagnostic laparotomy
D.	CT scan
E.	 AXR
F.	Endoscopy
G.	 CXR
H.	Laxatives
I.	Oral antibiotics
J.	Ultrasound scan
K.	Palliative care
L.	Acute pancreatitis
A

L. Acute pancreatitis

This patient has acute pancreatitis – the cause here being alcohol. He is describing mid-epigastric pain. This pain classically radiates around to the back, which in itself is almost diagnostic. Complicated haemorrhagic pancreatitis may exhibit Cullen’s sign, Grey-Turner’s sign and Fox’s sign. Make sure you know what these are and you are familiar with the causes of acute pancreatitis (GET SMASHED). Those caused by hypocalcaemia may also display Chvostek’s sign and Trousseau’s sign.
Key to diagnosis is serum amylase or lipase levels which are massively elevated. Prognostic criteria are outlined in Ranson’s criteria applied on admission and after 48 hours, or the modified Glasgow score which you can find in your Oxford Handbook. An abdominal CT is however the most sensitive and specific study and findings may include enlargement of the pancreas with irregular contours, necrosis, pseudocysts and peripancreatic fat obliteration.

69
Q

A 77 year old woman presents with a 3 day history of constant left iliac fossa pain. She has a temp of 38oC and O/E is tender with guarding in the left iliac fossa. CT scan demonstrates an inflamed sigmoid colon with numerous diverticulae.

What is the most suitable investigation?
A.	ECG
B.	 PR exam
C.	Diagnostic laparotomy
D.	CT scan
E.	 AXR
F.	Endoscopy
G.	 CXR
H.	Laxatives
I.	Oral antibiotics
J.	Ultrasound scan
K.	Palliative care
L.	Acute pancreatitis
A

I. Oral antibiotics

This patient obviously has diverticulitis and does not need further investigation. Symptomatic diverticulitis presents with fever, high WCC and LLQ pain. Risk factors for diverticular disease include low dietary fibre and advanced age. Oral antibiotic therapy and analgesia is indicated. If there is no improvement in 72 hours after oral antibiotics then IV antibiotics are indicated. Make sure you understand the differences in the terms: diverticulosis, diverticulitis and diverticular disease.

70
Q

A 48 year old woman presents with a 1 day history of constant right upper quadrant pain radiating round the right side of her chest. She says her urine may be darker than usual. Her GP started her on oral antibiotics. Amylase has already been ordered.

What is the most suitable investigation?
A.	ECG
B.	 PR exam
C.	Diagnostic laparotomy
D.	CT scan
E.	 AXR
F.	Endoscopy
G.	 CXR
H.	Laxatives
I.	Oral antibiotics
J.	Ultrasound scan
K.	Palliative care
L.	Acute pancreatitis
A

I. Ultrasound scan

Abdominal ultrasound is ordered when a patient presents with biliary pain and is the single best test for cholelithiasis (though has a low sensitivity for choledocholithiasis). Note that cholelithiasis refers to stones in the gallbladder and choledocholithiasis refers to stones in the bile duct. If stones are found then this would give weight to a diagnosis of acute cholecystitis. There are symptoms here of obstructive jaundice due to gallstone obstruction of bile outflow. Serum amylase would also be ordered in any patient presenting with pain located in the epigastric region, to rule out acute pancreatitis. This has been done here. In this patient you would also order LFTs, FBC looking for evidence of inflammation. MRCP, ERCP and EUS can be considered if necessary.

71
Q

A 30 year old male is brought into A&E having been found unrousable by his partner on the floor. He is breathing very occasionally and has small pupils. He has track marks in his antecubital fossa.

What is the best treatment?
A.	IV adrenaline
B.	 IV dextrose
C.	Urgent CT scan
D.	 IV carbamazepine
E.	 Phenytoin infusion
F.	IM benzylpenicillin
G.	Elevate legs
H.	IM adrenaline
I.	IV lorazepam
J.	IV diazepam
K.	IV naloxone
L.	 IV propofol
M.	 IV midazolam
N.	CPR
A

K. IV naloxone

Signs of opiate OD include CNS depression, miosis (pinpoint pupils) and apnoea. Naloxone is indicated both therapeutically and diagnostically. If there is a response, then it is diagnostic. Another diagnosis should be sought if the patient is unresponsive. IV is the preferred route of administration although naloxone can be given IM or SC if IV access cannot be established. Ventilatory support is key with 100% oxygen.

72
Q

A 50 year old male collapses in hospital while you are taking a history from the patient next to him. After 10 seconds, he is rapidly jerking his head with tonic stiffening arms quickly followed by clonic jerking. He becomes incontinent of urine and unresponsive.

What is the best treatment?
A.	IV adrenaline
B.	 IV dextrose
C.	Urgent CT scan
D.	 IV carbamazepine
E.	 Phenytoin infusion
F.	IM benzylpenicillin
G.	Elevate legs
H.	IM adrenaline
I.	IV lorazepam
J.	IV diazepam
K.	IV naloxone
L.	 IV propofol
M.	 IV midazolam
N.	CPR
A

I. IV lorazepam

This is a tonic-clonic, generalised seizure. It is characterised by LOC and widespread motor tonic contractions followed by clonic jerking movements. There will characteristically be a suppressed level of arousal following the event. This may either reflect a primary generalised episode or a focal seizure with secondary generalisation. The main aim of acute treatment is to terminate the seizure and to protect the airway. Management always starts with basic life-support (like every acute emergency) and your ABCs. IV access needs to be established (bloods sent to the lab too and serum glucose measured to test for reversable causes of seizure activity – thiamine should also be given to the patient if there is any concern about deficiency and hypoglycaemia, for instance in alcohol abuse). The following are needed: ECG, pulse oximetry, ABG. IV lorazepam is the preferred initial therapy, though rectal diazepam can be used if there is no IV access. If BZDs fail to stop the seizure then phenytoin or fosphenytoin can be tried.
After the episode, MRI and EEG are essential in diagnosing an epilepsy syndrome. During the episode of generalised tonic-clonic activity, the EEG will show bilateral synchrony in the epileptiform activity. If this is a one-off seizure in which a provoking factor, such as electrolyte disturbance or hypoglycaemia, has been identified then there is no need for therapy for epilepsy. In unprovoked cases, this depends on history, examination, EEG and MRI. Treatment may not be needed the first time but after a second unprovoked instance, therapy is generally recommended.

73
Q

An 18 year old man attends his GP to have a routine blood test. He has no significant past medical history. While having the blood test he becomes pale, clammy and says he feels sick. He subsequently becomes unresponsive.

What is the best treatment?
A.	IV adrenaline
B.	 IV dextrose
C.	Urgent CT scan
D.	 IV carbamazepine
E.	 Phenytoin infusion
F.	IM benzylpenicillin
G.	Elevate legs
H.	IM adrenaline
I.	IV lorazepam
J.	IV diazepam
K.	IV naloxone
L.	 IV propofol
M.	 IV midazolam
N.	CPR
A

G. Elevate legs

This is a case of vasovagal syncope. Elevation of the legs will help move blood towards the head. The other options on the list are not necessary.

74
Q

A 13-year-old boy collapses at the playground. He has recurrent seizures for over 30 minutes. It seems he has not regained consciousness in between the seizures.

What is the best treatment?
A.	IV adrenaline
B.	 IV dextrose
C.	Urgent CT scan
D.	 IV carbamazepine
E.	 Phenytoin infusion
F.	IM benzylpenicillin
G.	Elevate legs
H.	IM adrenaline
I.	IV lorazepam
J.	IV diazepam
K.	IV naloxone
L.	 IV propofol
M.	 IV midazolam
N.	CPR
A

I. IV lorazepam

This patient is in status epilepticus defined by 30 minutes or more of continuous seizure activity, or repetitive seizures with no intervening recovery of consciousness. SE can be either generalised convulsive or non-convulsive (simple or complex partial). In children, seizures of a shorter duration may also be considered to be status epilepticus. The initial treatment is as 2) and should start with BLS measures, continuous monitoring and benzodiazepines as first line therapy – with IV lorazepam. Securing the patient’s airwar may prove difficult due to the convulsions and neuromuscular blockade with a short-acting drug such as vecuronium may be necessary.
Unresponsive cases can get anticonvulsants or phenobarbitone. It is worth noting that phenytoin infusions may lead to venous irritation and tissue damage if the undiluted drug is given through a small bore cannula. If SE persists, the next step to take is to intubate and start general anaesthesia. The best initial agents to use are midazolam and propofol though other options include pentobarbital and thiopental, the former being an active metabolite of the latter. GA should be tapered off after a minimum of 12 hours, and if the seziure recurs then the infusion should be restarted for another 12-24 hours.

75
Q

A 22-year-old student collapses in her room. She is febrile and you notice a rash over her body.

What is the best treatment?
A.	IV adrenaline
B.	 IV dextrose
C.	Urgent CT scan
D.	 IV carbamazepine
E.	 Phenytoin infusion
F.	IM benzylpenicillin
G.	Elevate legs
H.	IM adrenaline
I.	IV lorazepam
J.	IV diazepam
K.	IV naloxone
L.	 IV propofol
M.	 IV midazolam
N.	CPR
A

F. IM benzylpenicillin

This is suspected bacterial meningitis and in the community setting the patient should receive immediate IM benzylpenicillin before being transferred to hospital as an emergency case.

76
Q

For each disease/syndrome, choose the most likely mode of inheritance.

Duchenne muscular dystrophy

A.	Autosomal recessive
B.	None/unknown
C.	X-linked recessive
D.	 X-linked dominant
E.	 Mitochondrial inheritance
F.	Chromosomal abnormality
G.	Autosomal dominant
A

C. X-linked recessive

Here is a list of the important conditions you should be aware of:
Autosomal dominant: Achrondroplasia, APKD, dystrophia myotonica, familial hypercholesterolaemia, Huntington’s, Marfan’s, NF, tuberous sclerosis, osteogenesis imperfecta
Autosomal recessive: CF, hereditary haemochromatosis, sickle cell disease, Wilson’s, Friedreich’s ataxia, CAH
X-linked recessive: DMD, Fragile X syndrome, Haemophilia A and B, G6PDH deficiency, Alport’s
You should be aware that Down’s is trisomy 21, Edwards’ is trisomy 18 and Patau’s is trisomy 13.

77
Q

For each disease/syndrome, choose the most likely mode of inheritance.

Cystic fibrosis

A.	Autosomal recessive
B.	None/unknown
C.	X-linked recessive
D.	 X-linked dominant
E.	 Mitochondrial inheritance
F.	Chromosomal abnormality
G.	Autosomal dominant
A

A. Autosomal recessive

Here is a list of the important conditions you should be aware of:
Autosomal dominant: Achrondroplasia, APKD, dystrophia myotonica, familial hypercholesterolaemia, Huntington’s, Marfan’s, NF, tuberous sclerosis, osteogenesis imperfecta
Autosomal recessive: CF, hereditary haemochromatosis, sickle cell disease, Wilson’s, Friedreich’s ataxia, CAH
X-linked recessive: DMD, Fragile X syndrome, Haemophilia A and B, G6PDH deficiency, Alport’s
You should be aware that Down’s is trisomy 21, Edwards’ is trisomy 18 and Patau’s is trisomy 13.

78
Q

For each disease/syndrome, choose the most likely mode of inheritance.

Achondroplasia

A.	Autosomal recessive
B.	None/unknown
C.	X-linked recessive
D.	 X-linked dominant
E.	 Mitochondrial inheritance
F.	Chromosomal abnormality
G.	Autosomal dominant
A

G. Autosomal dominant

Here is a list of the important conditions you should be aware of:
Autosomal dominant: Achrondroplasia, APKD, dystrophia myotonica, familial hypercholesterolaemia, Huntington’s, Marfan’s, NF, tuberous sclerosis, osteogenesis imperfecta
Autosomal recessive: CF, hereditary haemochromatosis, sickle cell disease, Wilson’s, Friedreich’s ataxia, CAH
X-linked recessive: DMD, Fragile X syndrome, Haemophilia A and B, G6PDH deficiency, Alport’s
You should be aware that Down’s is trisomy 21, Edwards’ is trisomy 18 and Patau’s is trisomy 13.

79
Q

For each disease/syndrome, choose the most likely mode of inheritance.

Down’s syndrome

A.	Autosomal recessive
B.	None/unknown
C.	X-linked recessive
D.	 X-linked dominant
E.	 Mitochondrial inheritance
F.	Chromosomal abnormality
G.	Autosomal dominant
A

F. Chromosomal abnormality

Here is a list of the important conditions you should be aware of:
Autosomal dominant: Achrondroplasia, APKD, dystrophia myotonica, familial hypercholesterolaemia, Huntington’s, Marfan’s, NF, tuberous sclerosis, osteogenesis imperfecta
Autosomal recessive: CF, hereditary haemochromatosis, sickle cell disease, Wilson’s, Friedreich’s ataxia, CAH
X-linked recessive: DMD, Fragile X syndrome, Haemophilia A and B, G6PDH deficiency, Alport’s
You should be aware that Down’s is trisomy 21, Edwards’ is trisomy 18 and Patau’s is trisomy 13.

80
Q

For each disease/syndrome, choose the most likely mode of inheritance.

Multiple sclerosis

A.	Autosomal recessive
B.	None/unknown
C.	X-linked recessive
D.	 X-linked dominant
E.	 Mitochondrial inheritance
F.	Chromosomal abnormality
G.	Autosomal dominant
A

B. None/ unknown

Here is a list of the important conditions you should be aware of:
Autosomal dominant: Achrondroplasia, APKD, dystrophia myotonica, familial hypercholesterolaemia, Huntington’s, Marfan’s, NF, tuberous sclerosis, osteogenesis imperfecta
Autosomal recessive: CF, hereditary haemochromatosis, sickle cell disease, Wilson’s, Friedreich’s ataxia, CAH
X-linked recessive: DMD, Fragile X syndrome, Haemophilia A and B, G6PDH deficiency, Alport’s
You should be aware that Down’s is trisomy 21, Edwards’ is trisomy 18 and Patau’s is trisomy 13.

81
Q

A 60 year old man on anti-cholinergic drugs presents with supra-pubic pain. He complains that he has not passed any urine in 3 days. O/E he has a distended bladder.

What is the diagnosis?
A.	Urinary tract infection
B.	Labour
C.	Acute urinary retention
D.	Urinary tract stones: bladder outflow obstruction
E.	 Pyelonephritis
F.	 Polycystic kidney disease
G.	Endometriosis
H.	Bladder cancer
I.	Urinary tract stones: ureteric colic/stricture
J.	Colorectal cancer
K.	Ovarian cyst
A

C. Acute urinary retention

This is acute urinary retention caused by anticholinergics. Unwanted effects of this class of drugs include urinary retention, constipation, erectile dysfunction, CNS disturbance, cycloplegia, dry mouth and decreased sweating. Poisoning in severe cases can be treated with anticholinesterases such as physostigmine. This man will need to be catheterised first to relieve the urinary retention and prevent any further complications from occuring.

82
Q

An overweight 30 year old lady presents to A&E with an intense, colicky abdominal pain and vomiting which started last night. She says she has noticed weight gain and pelvic pain for a few months. She has also had vaginal discharge. On examination there is gross swelling of the abdomen.

What is the diagnosis?
A.	Urinary tract infection
B.	Labour
C.	Acute urinary retention
D.	Urinary tract stones: bladder outflow obstruction
E.	 Pyelonephritis
F.	 Polycystic kidney disease
G.	Endometriosis
H.	Bladder cancer
I.	Urinary tract stones: ureteric colic/stricture
J.	Colorectal cancer
K.	Ovarian cyst
A

B. Labour

Abdominal pain throughout pregnancy is common although reproductive organs share the name visceral innvervations as the lower ileum, sigmoid colon and rectum so it may be difficult to differentiate GI pain from gynaecological pain. This acute abdomen would require some consideration as the clinical picture may be distorted by the fact that pregnancy stretches the anterior abdominal wall so peritoneal signs are often different due to a lack of contact with the underlying inflammation, if an inflammatory process were to be present. The case described here could well be labour and the already overweight woman has simply been unaware of the fact she is pregnant (which although uncommon, can occur). The gross abdominal distension (which she may not attribute to anything abnormal if she is already fat) and symptoms are pointing to labour and there may also be uterine contractions. Blood or mucus on vaginal examination may be seen and you would expect the cervix to be soft, effaced and possibly dilated. However, this could be another complication of pregnancy or even a non-gynaecological condition and these need to also be considered during the diagnostic work-up. An ultrasound scan, which is safe to use in pregnancy, should be done here.

83
Q

A 45 year old smoker presents with painful haematuria. He has a history of recurrent UTI’s. He tells you that he is now a taxi driver but he used to work in the rubber industry.

What is the diagnosis?
A.	Urinary tract infection
B.	Labour
C.	Acute urinary retention
D.	Urinary tract stones: bladder outflow obstruction
E.	 Pyelonephritis
F.	 Polycystic kidney disease
G.	Endometriosis
H.	Bladder cancer
I.	Urinary tract stones: ureteric colic/stricture
J.	Colorectal cancer
K.	Ovarian cyst
A

H. Bladder cancer

Gross haematuria is the primary symptom of bladder cancer. Over 80% present with haematuria which is the primary presenting complaint in this condition. The gross haematuria is classically painless and present throughout the entire urinary stream, however while painful the rest of the symptoms fit. It is also worth noting that carcinoma in situ commonly presents with dysuria and frequency and can easily be confused with prostatitis. Risk factors include smoking, exposure to carcinogens such as the aromatic amines used in rubber and dye industries, age >55, pelvic radiation and Schistosomiasis resulting in SCC (related to chronic inflammation – so other risks also include UTI, stones etc). Bladder cancer is the most common cancer in Egypt, for the latter reason. Cystoscopy and urinary cytology are key in diagnosis. Low grade tumours are papillary and easy to see on cystoscopy whereas high grade tumours and carcinoma in situ are often difficult to visualise. Resection provides diagnosis and primary treatment in one step.

84
Q

A 23 year old woman presents with increasing frequency of passing urine. She complains that it is painful and smelly. O/E she has suprapubic pain and tenderness. She has just recovered from a chest infection which was quickly treated.

What is the diagnosis?
A.	Urinary tract infection
B.	Labour
C.	Acute urinary retention
D.	Urinary tract stones: bladder outflow obstruction
E.	 Pyelonephritis
F.	 Polycystic kidney disease
G.	Endometriosis
H.	Bladder cancer
I.	Urinary tract stones: ureteric colic/stricture
J.	Colorectal cancer
K.	Ovarian cyst
A

A. Urinary tract infection

This young woman has presented with a UTI (dysuria, frequency, and foul smelling urine). Sexual activity the strongest risk factor for UTIs in women. It is diagnosed with a urine dipstick and microscopic analysis for bacteria, WBC and RBC and urine culture and antibiotic sensitivities. Antibiotic selection should be based on local guidelines or known sensitivities. Uncomplicated UTIs with no known antibiotic resistance can be treated with co-trimoxazole or nitrofurantoin. If there is resistance, a quinolone can be considered such as ciprofloxacin.

85
Q

A 50 year old lady presents to A&E in excruciating pain. It radiates from the flank to the iliac fossa and labium. She can not lie still and is pale, sweating and vomiting.

What is the diagnosis?
A.	Urinary tract infection
B.	Labour
C.	Acute urinary retention
D.	Urinary tract stones: bladder outflow obstruction
E.	 Pyelonephritis
F.	 Polycystic kidney disease
G.	Endometriosis
H.	Bladder cancer
I.	Urinary tract stones: ureteric colic/stricture
J.	Colorectal cancer
K.	Ovarian cyst
A

I. Urinary tract stones: ureteric colic/ stricture

This patient has renal colic which classically presents with severe flank pain radiating to the groin. Microscopic haematuria is present in up to 90% of cases. Up to 85% of stones are visible on a plain KUB although urate stones are radiolucent. If the stone is radio-opaque, calcification will be seen within the urinary tract. In pregnancy, a renal USS is first line. The IVP has now been replaced by the CT scan which is the new diagnostic standard otherwise. A non-contrast helical (or spiral) CT is preferred due to high sensitivity and specificity and acurately determines presence, site and size of stones. Stones are analysed after they are extracted or when they are expelled to check their composition. It is worth noting that in all females of child bearing age, a urine pregnancy test is necessary to exclude an ectopic pregnancy.

86
Q

A 46 year old diabetic man collapses while painting his house after taking some tablets for painful indigestion and is brought into A&E sweating and in distress. His wife had to stop the car en route to allow him to vomit. Examination shows him to be sweating & tachycardic but is otherwise unremarkable.

What is the diagnosis?
A.	Labyrinthitis
B.	Large bowel obstruction
C.	Gastroenteritis
D.	Meningitis
E.	Small bowel obstruction
F.	Migrane
G.	Pancreatitis
H.	Appendicitis
I.	Gastric ulcer
J.	Myocardial infarction
A

J. Myocardial infarction

This diabetic is having a silent MI without chest pain. All of the other symptoms are indicative such as vomiting, sweating and tachycardia. Silent MIs are more common in the elderly and those with DM probably due to autonomic neuropathy. If ECG shows STEMI, new LBBB or confirmed posterior MI then PCI/thrombolysis is indicated. It is worth noting that RV infarction is present in 40% of inferior infarcts so if ST elevation is seen in II, III and aVF, right sided ECG leads should be obtained. Cardiac biomarkers include CK-MB and troponin. Troponins rise 4-6 hrs after onset of infarction and peak at 18-24 hours and may persist for 7-10 days.

87
Q

A student teacher presents on your take after school feeling drowsy and irritable. She has a splitting headache & has vomited 3 times. There is no blood in the vomit and she denies any relationship to eating. O/E her pulse is slow but regular and her BP is increased.

What is the diagnosis?
A.	Labyrinthitis
B.	Large bowel obstruction
C.	Gastroenteritis
D.	Meningitis
E.	Small bowel obstruction
F.	Migrane
G.	Pancreatitis
H.	Appendicitis
I.	Gastric ulcer
J.	Myocardial infarction
A

D. Meningitis

From the list of options given, the only feasible option is meningitis. This history is not consistent with a migraine. Commonly in meningitis, there will be a headache, fever and nuchal rigidity. There may also be an altered mental status, confusion, photophobia and vomiting. Kernig’s sign is uncommon but is positive when attempts to extend the leg are met with resistance when the patient is supine with the thigh flexed to 90 degrees. Another uncommon sign is Brudzinski’s sign and a petechial/purpuric rash, typically associated with meningococcal meningitis.
CT head should be considered before LP if there is any evidence of raised ICP. An LP will confirm the diagnosis with bacterial meningitis showing a low CSF glucose, elevated CSF protein and positive CSF culture/gram stain or meningococcal antigen.

88
Q

A widowed 55 year old man complaining of a severe pain in his abdomen is admitted while drunk. He finds the pain eases when he sits forward as it radiates to his back. He relates to you that he has been vomiting heavily. O/E he is pyrexial and tachycardic with a rigid abdomen.

What is the diagnosis?
A.	Labyrinthitis
B.	Large bowel obstruction
C.	Gastroenteritis
D.	Meningitis
E.	Small bowel obstruction
F.	Migrane
G.	Pancreatitis
H.	Appendicitis
I.	Gastric ulcer
J.	Myocardial infarction
A

G. Pancreatitis

This patient has pancreatitis. He has vomited heavily and is describing mid-epigastric pain radiating around to the back. This description of the pain itself is almost diagnostic. This pain is classically relieved if the patient curls up and is worse with movement. Complicated haemorrhagic pancreatitis may exhibit Cullen’s sign, Grey-Turner’s sign and Fox’s sign. Make sure you know what these are and you are familiar with the causes of acute pancreatitis – it seems likely to be alcohol in this instance. Those caused by hypocalcaemia may display Chvostek’s sign and Trousseau’s sign.
Key to diagnosis is serum amylase or lipase levels which are massively elevated. Prognostic criteria are outlined in Ranson’s criteria applied on admission and after 48 hours, or the modified Glasgow score which you can find in your Oxford Handbook. An abdominal CT is however the most sensitive and specific study and findings may include enlargement of the pancreas with irregular contours, necrosis, pseudocysts and peripancreatic fat obliteration.

89
Q

A 25 year old man give a history of loss of appetite and nausea over 2 days with profuse vomiting. He also has colicky abdominal pain with increased bowel sounds. PMHx reveals that he had an exploratory laparotomy 2 years ago.

What is the diagnosis?
A.	Labyrinthitis
B.	Large bowel obstruction
C.	Gastroenteritis
D.	Meningitis
E.	Small bowel obstruction
F.	Migrane
G.	Pancreatitis
H.	Appendicitis
I.	Gastric ulcer
J.	Myocardial infarction
A

E. Small bowel obstruction

Bowel obstruction has several causes such as adhesions or cancer in the older patient. In this case the PMH of recent surgery suggests adhesions as a likely aetiology. The profuse vomiting, distended and colicky painful abdomen and increased tinkling high pitched bowel signs are all indicative. The proximal segment of bowel dilates and distal bowel collapses. Completely obstructed patients generally require surgery. If, on AXR, air is seen to be seeping past the obstruction then the obstruction is partial. As a standard, all patients should be made NBM and given supplemental oxygen, IV fluids and NG decompression (to reduce flow/gastric contents/air towards the obstruction), unless they are rushed off for an emergency laparotomy because, for example, they have complete SBO and are peritonitic.

Small bowel vs. large bowel obstruction

Small bowel obstruction tends to be more acute, tinkling bowel sounds can be heard, pain is central and colicky, , and is particularly linked to post-surgical adhesions

Large bowel obstruction is more gradual, bowel sounds are diminished or normal, pain is localised

90
Q

A patient who is unsteady on their feet tells you that they have had 20 minute unsettling spells of nausea and vomiting over the past three days. Their unsteadness worsens with the episodes and they describe a feeling of veering sideways. O/E the patient is pale and sweaty but little else is remarkable.

What is the diagnosis?
A.	Labyrinthitis
B.	Large bowel obstruction
C.	Gastroenteritis
D.	Meningitis
E.	Small bowel obstruction
F.	Migrane
G.	Pancreatitis
H.	Appendicitis
I.	Gastric ulcer
J.	Myocardial infarction
A

A. Labrynthitis

Labrynthitis is an inflammatory condition affecting the labyrinth in the cochlea and the vestibular system of the inner ear. The most common cause is viral in origin although bacterial laryrinthitis occurs as a complication of otitis media or meningitis. The typical presentation is one including vertigo, imbalance and hearing loss. The typical description is of acute rotational vertigo which may last up to 7 hours. This is often associated with N&V. Risk factors include recent URTI or other viral infections such as VZV, CMV, mymps, measles, rubella and HIV. Treatment is typically symptomatic and involves the use of vestibular suppresants (benzodiazepines such as diazepam) and antiemetics (such as promethazine or metaclopramide). Labyrinthitis is the only condition on the list which would cause this constellation of symptoms this patient is describing. In reality, this presentation could of course be vestibular neuritis.

91
Q

A 33 year old lady with no children has been suffering worsening pelvic pain particularly prior to menstruation and is now complaining of deep dysareunia.

What is the diagnosis?
A.	Endometrial cancer
B.	Pelvic inflammatory disease
C.	Endometriosis
D.	Ectopic pregnancy
E.	Fibroids
F.	Adenomyosis
G.	Retroverted uterus
A

A. Endometriosis

Endometriosis is a chronic inflammatory condition defined by endometrial stroma and glands located outside of the uterine cavity – the most common sites being the pelvic peritoneum and ovaries. It may present as an incidental finding in asymptomatic patients but more commonly it presents in women of reproductive age with chronic pelvic pain and/or subfertility. This woman has symptoms which make this diagnosis likely. There is dyspareunia which is pain during sexual intercourse, particularly on deep penetration, and may be caused by a distortion in the pelvic anatomy and rectovaginal involvement. There is also well documented genetic predisposition so a positive FH may be found. Additionally, nulliparous women are more likely to be diagnosed with endometriosis than parous women. The diagnosis can be confirmed on visualising the ectopic tissue directly and focused biopsies during laparoscopy but this is not generally necessary as clinical suspicion is enough to start treatment. Options for treatment include NSAIDs, COCPs, GnRH agonists, danazol or related androgens and surgical destruction of lesions. Those who present with subfertility may be considered for ovarian hyperstimulation and IVF.

92
Q

A 55 year old lady on HRT, complains of non-specific pelvic pain and occasional spotting of blood prioir to her withdrawal bleed on HRT.

What is the diagnosis?
A.	Endometrial cancer
B.	Pelvic inflammatory disease
C.	Endometriosis
D.	Ectopic pregnancy
E.	Fibroids
F.	Adenomyosis
G.	Retroverted uterus
A

A. Endometrial cancer

This is a common malignancy and is usually an adenocarcinoma. Obesity is associated with an increased incidence of endometrial cancer and also poorer outcome. Risk factors to consider aside from obesity include HRT, tamoxifen use, age over 50, unopposed oestrogen and radiotherapy. FH and a history of other cancers are also risks. A good history is important to establish that PV bleeding does not have another obvious cause such as intercourse or that associated with HRT, and that is is unlikely to be related to another malignancy like cervical cancer. Examination tends to be challenging due to the prevalence of obesity in those with endometrial cancer. The presentation is typically with post-menopausal bleeding PV and often the disease is surgically curable. Diagnosis will need to be confirmed by biopsy and histology with histopathology showing adenocarcinoma.

93
Q

A 35-year-old lady suffers of severe menorrhagia and pelvic pain. O/E she has tender, enlarged uterus. She underwent a total hysterectomy, pathology report confirmed diffuse fibromyomatous reaction with endometrial tissue within the myometrium.

What is the diagnosis?
A.	Endometrial cancer
B.	Pelvic inflammatory disease
C.	Endometriosis
D.	Ectopic pregnancy
E.	Fibroids
F.	Adenomyosis
G.	Retroverted uterus
A

F. Adenomyosis

Adenomyosis, as you can probably figure out from the name, is the presence of ectopic glandular tissue in muscle. The first test to order is a pelvic USS which shows a normal or enlarged uterus. It is of minimal diagnostic value, especially if the clinical history and examination findings are suggestive, but can be used to rule out endometriosis, where the USS may show the presence of ovarian endometriomas. Examination may reveal an enlarged globular uterus and uterine tenderness on palpation, particularly during the menses. The history is usually of a parous woman, symptoms commonly occuring after childbirth, with heavy menstrual flow or an abnormal bleeding pattern being seen. Adenomyosis is a condition which can be diagnosed clinically, although if a costly MRI pelvis is done, then abnormal signal intensities within the myometrium can be seen.

94
Q

A 29-year-old lady presents to A&E with severe RIF pain and vaginal bleeding. She says she takes her OCP regulary so there is no chance of her being pregnant. B-HCG test is positive.

What is the diagnosis?
A.	Endometrial cancer
B.	Pelvic inflammatory disease
C.	Endometriosis
D.	Ectopic pregnancy
E.	Fibroids
F.	Adenomyosis
G.	Retroverted uterus
A

D. Ectopic pregnancy

Ectopic pregnancy usually presents between 6-8 weeks after the last normal menstrual period but it can present later on. The risk increases if the woman has had a previous ectopic, surgery on the tubes, genital infections, smokes or uses an IUD. The classic symptoms and signs are pain, vaginal bleeding and amenorrhoea. If the patient is haemodynamically unstable or there is cervical motion tenderness, this may indicate that a rupture has occured or is imminent. Rupture, which is a complication, can present with shock from blood loss and with unusual patterns of referred pain from the presence of intraperitoneal blood. The positive urine pregnancy test here confirms pregnancy. Once the patient is confirmed to be pregnant, a transvaginal USS is used to determine the location of the pregnancy. If an intrauterine gestation is visible on USS regardless of whether it is viable, then the chances of having an ectopic pregnancy are incredibly low. Occasionally, an ectopic pregnancy itself can be seen, either as a ‘doughnut sign’ (adnexal mass separate from two clearly seen ovaries) or ‘ring of fire’ (increased blood flood to the ectopic seen on colour Doppler). A transabdominal ultrasound is less sensitive than a TVUS. Treatment approaches can include expactant, medical (methotrexate) or surgical (salpingectomy, salpingostomy).

95
Q

A 22-year-old woman with Hx of chlamydial urethritis complains of pelvic pain and painful periods. You order an endocervical smear which confirms the presence of Chlamydia trachomatis.

What is the diagnosis?
A.	Endometrial cancer
B.	Pelvic inflammatory disease
C.	Endometriosis
D.	Ectopic pregnancy
E.	Fibroids
F.	Adenomyosis
G.	Retroverted uterus
A

B. Pelvic inflammatory disease

Pelvic inflammatory disease is an acute ascending infection of the female tract that is often associated with Neisseria gonorrhoeae or Chlamydia trachomatis. Key risk factors include prior infection with chlamydia or gonorrhoea or PID, young age of onset of sexual activity, unprotected sex with multiple partners and IUD use. Signs and symptoms vary and can include tenderness of the lower abdomen, adnexal tenderness and cervical motion tenderness. Fever and cervical or vaginal discharge may also be present. Complications include tubo-ovarian abscess and subsequent infertility or ectopic pregnancy due to scarred or obstructed fallopian tubes.

96
Q

A 60 year old woman presents with a poor appetite, weight loss, tiredness & right iliac fossa intermittent discomfort for the last 6 weeks.

What is the diagnosis?
A.	Perianal fissure
B.	Inflammatory bowel disease
C.	Meckel's diverticulitis
D.	Carcinoma of the rectum
E.	Haemorrhoids
F.	Irritable bowel syndrome
G.	Caecal carcinoma
H.	Infective diarrhoea
I.	Anal fissure
J.	Duodenal ulcer
A

G. Caecal carcinoma

The weight loss, fatigue and RIF discomfort point to caecal carcinoma. Right sided colorectal cancer tends to present with anaemic symptoms. Almost 90% are anaemic at diagnosis.

97
Q

A 30 year old man presents with painless fresh rectal bleeding which appears on the stool, on the paper and in the toilet bowl.

What is the diagnosis?
A.	Perianal fissure
B.	Inflammatory bowel disease
C.	Meckel's diverticulitis
D.	Carcinoma of the rectum
E.	Haemorrhoids
F.	Irritable bowel syndrome
G.	Caecal carcinoma
H.	Infective diarrhoea
I.	Anal fissure
J.	Duodenal ulcer
A

E. Haemorrhoids

Haemorrhoids are vascular rich cushions in the anal canal and presents, typically, as painless bright PR bleeding or with sudden onset pain in the area associated with a palpable mass. Pruritus ani is common and there is often perianal pain or discomfort. Diagnosis is made visually. Grade 1 is limited to within the anal canal. Grade 2 protrudes but spontaneously reduces when the patient stops straining. Grade 3 protrudes and reduces fully on manual pressure. Grade 4 is irreducible. Treatment includes fibre, ligation, photocoagulation, sclerotherapy or surgical haemorrhoidectomy. Haemorrhoidectomy is the treatment of choice of choice for patients with grade 4 haemorrhoids or for any patient who has failed with more conservative treatment such as sclerotherapy.

98
Q

A 47 year old man with a 5 year history of dyspepsia, collapse in the pub. He notices that his stools have become black over the last few days.

What is the diagnosis?
A.	Perianal fissure
B.	Inflammatory bowel disease
C.	Meckel's diverticulitis
D.	Carcinoma of the rectum
E.	Haemorrhoids
F.	Irritable bowel syndrome
G.	Caecal carcinoma
H.	Infective diarrhoea
I.	Anal fissure
J.	Duodenal ulcer
A

J. Duodenal ulcer

Haemorrhoids are vascular rich cushions in the anal canal and presents, typically, as painless bright PR bleeding or with sudden onset pain in the area associated with a palpable mass. Pruritus ani is common and there is often perianal pain or discomfort. Diagnosis is made visually. Grade 1 is limited to within the anal canal. Grade 2 protrudes but spontaneously reduces when the patient stops straining. Grade 3 protrudes and reduces fully on manual pressure. Grade 4 is irreducible. Treatment includes fibre, ligation, photocoagulation, sclerotherapy or surgical haemorrhoidectomy. Haemorrhoidectomy is the treatment of choice of choice for patients with grade 4 haemorrhoids or for any patient who has failed with more conservative treatment such as sclerotherapy.

99
Q

A 59 year old man presents with 1 month history of constipation, tenesmus and fresh rectal bleeding. He also notes some weight loss recently.

What is the diagnosis?
A.	Perianal fissure
B.	Inflammatory bowel disease
C.	Meckel's diverticulitis
D.	Carcinoma of the rectum
E.	Haemorrhoids
F.	Irritable bowel syndrome
G.	Caecal carcinoma
H.	Infective diarrhoea
I.	Anal fissure
J.	Duodenal ulcer
A

D. Carcinoma of the rectum

This a rectal carcinoma. Tenesmus, blood PR alongside weight loss are all highly suggestive. Treatment of rectal carcinoma involves surgical excision where possible. This can either be an anterior resection (tumours in the upper 1/3 of the rectum) or an abdominoperineal resection (if the tumour lies lower down). APER involves the formation of a permanent colostomy and has a high incidence of sexual and urinary dysfunction. Anterior resection involves a colo-anal anastamosis.

100
Q

A 25 year old woman presents with a 4 month history of diarrhoea, altered blood and mucus per rectum.

What is the diagnosis?
A.	Perianal fissure
B.	Inflammatory bowel disease
C.	Meckel's diverticulitis
D.	Carcinoma of the rectum
E.	Haemorrhoids
F.	Irritable bowel syndrome
G.	Caecal carcinoma
H.	Infective diarrhoea
I.	Anal fissure
J.	Duodenal ulcer
A

B. Inflammatory bowel disease

This woman has inflammatory bowel disease, which by the history is probably more likely to be UC than CD, whereby the mainstay of treatment is with 5-ASA. A colonoscopy is required to assess the extent of disease and for a definitive diagnosis. Biopsy in CD will show transmural granulomatous inflammation. CD can affect the whole GIT but favours the TI and proximal colon and is macroscopically characterised by skip lesions. UC on the other hand is characterised by the presence of crypt abscesses, which is pathognomic. CD risk is increased 3-4 fold by smoking whereas smoking seems protective in UC. The mainstay of treatment in CD is with steroids and azathioprine to prevent relapses and for those suffering side effects of steroid treatment. TNF-alpha inhibitors also have a role. Surgery in CD is only indicated in a small number of patients who bleed, for bowel perforation and cases of complete obstruction. The aim is to rest distal disease by temporarily diverting faecal flow.

101
Q

A 50 year old man who feels tired all the time, has recently noticed pigmentation of his skin and has lost some weight.

What is the diagnosis?
A.	SIADH
B.	Chronic renal failure
C.	Hyperthyroidism
D.	Hypothyroidism
E.	AIDS
F.	Diabetes mellitus
G.	Addison's disease
H.	Colorectal carcinoma
I.	Psychological distress
J.	Anaemia
K.	Glandular fever
A

G. Addison’s disease

Hyperpigmentation in the palmar creases points towards Addison’s disease. Hyperpigmentation due to excess ACTH production can be mucosal or cutaneous and is more pronounced in the palms, knuckles and around scars. MSH is a byproduct of the production of ACTH from the cleavage of POMC. Anorexia, fatigue and weight loss is observed in all patients. Sodium is low and potassium elevated. Vomiting is present in 75% of patients and nausea is a common finding. Additionally, postural hypotension may be present. The presence of other autoimmune diseases is a risk factor for the development of Addison’s. Diagnosis of Addison’s can be made on an ACTH stimulation test (synacthen test) whereby serum cortisol remains low despite the administration of synthetic ACTH. In an emergency, treatment should not be delayed by diagnostic testing.

102
Q

A 16 year old girl who feels tired all the time. She had a sore throat prior to this. She is concerned because she is unable to study for her exams. Her boyfriend has had similar symptoms.

What is the diagnosis?
A.	SIADH
B.	Chronic renal failure
C.	Hyperthyroidism
D.	Hypothyroidism
E.	AIDS
F.	Diabetes mellitus
G.	Addison's disease
H.	Colorectal carcinoma
I.	Psychological distress
J.	Anaemia
K.	Glandular fever
A

K. Glandular fever

This is infectious mononucleosis or glandular fever and is caused by EBV. It is characterised by fever, pharyngitis and lymphadenopathy. Enlargement of the spleen begins in the first week and lasts 3-4 weeks, occuring in half of all cases. Risk factors for EBV transmission include kissing and sex. The fact her boyfriend has had similar symptoms should raise your suspicion. A FBC will show an atypical lymphocytosis. Confirmation of IM involves detection of the existence of heterophile antibodies using the Paul Bunnell monospot. A more accurate test is a serological test detecting EBV specific antibodies. Treatment is usually symptomatic but IM can carry rare but potentially life threatening complications.

103
Q

A 15 year old boy is tired all the time. He is also suffering from polyuria, nocturia and polydipsia.

What is the diagnosis?
A.	SIADH
B.	Chronic renal failure
C.	Hyperthyroidism
D.	Hypothyroidism
E.	AIDS
F.	Diabetes mellitus
G.	Addison's disease
H.	Colorectal carcinoma
I.	Psychological distress
J.	Anaemia
K.	Glandular fever
A

F. Diabetes mellitus

Polyuria, polydipsia, nocturia in a boy who is tired all the time should make you think of T1DM. Insulin is needed alongside dietary changes and exercise. Insulin regimes aim to mimic physiological insulin release with a basal-bolus dosing. There is an option between using a pump and having multiple daily injections. It is worth noting that there is a high incidence of diabulimia among young people with T1DM who give themselves less insulin than they need in order to lose weight (they lose weight, ‘look good’ but trash their bodies).

104
Q

A 32 year old homosexual man is tired all the time. He also complains of weight loss and purple lesions on his skin.

What is the diagnosis?
A.	SIADH
B.	Chronic renal failure
C.	Hyperthyroidism
D.	Hypothyroidism
E.	AIDS
F.	Diabetes mellitus
G.	Addison's disease
H.	Colorectal carcinoma
I.	Psychological distress
J.	Anaemia
K.	Glandular fever
A

E. AIDS

AIDS (acquired immunodeficiency syndrome) is caused by HIV, which is a retrovirus. To give you an indication of risk here, there is a risk of 50 infections per 10,000 exposed to an infected source in unprotected receptive anal intercourse. The risk with receptive vaginal intercourse is 10 infections per 10,000 exposures. Obviously, people have sex more often than the one off, as is human nature, so what seems like a small risk per sexual encounter adds up. IVDU needle sharing has a risk of 67 per 10,000, a needle-stick is 30 per 10,000 (equal to having receptive vaginal intercourse 3 times with an HIV positive man, so be careful on the wards, though this statistic does depend on factors like the size of the needle) and the risk associated with vertical transmission is associated with maternal viral load (the risk goes if you can suppress the viral load with anti-retrovirals). The thing to note is that the association with homosexuality is based on the increased risk of transmission from receptive anal compared to receptive vaginal, and if you happen to be a homosexual male and contract HIV, you’re unlikely to pass it on with vaginal intercourse. There should not be negative stigma attached to HIV.
There are two types, HIV 1 which is the main virus responsible and HIV 2 which is restricted to parts of West Africa. Weight loss is common in HIV and if more than 10% body weight is lost or BMI reduces to 18.5, this is an indication of more severe immunocompromise. Weight loss in HIV may result from malnutrition, co-existent TB infection or HIV wasting syndrome, the latter being an AIDS defining illness. The purple lesions seen here are due to Kaposi’s sarcoma, which is a neoplasm derived from mesenchymal tissue, associated with HHV-8 infection. This is an AIDS defining infection.
There are WHO (stage 1-4) and CDC criteria used in clinical staging of HIV. This patient needs to have a CD4 count, HBV and HCV screen, VDRL (syphilis), tuberculin skin test (TB) and CXR. HIV viral load will also be assessed. Prophylaxis and immunisations should be considered against infections such as hepatitis, influenza, PCP and TB. HAART needds to be initiated as he has developed AIDS. Classes of antiretrovirals include NRTIs, NNRTIs, protease inhibitors, fusion inhibitors and integrase inhibitors.

105
Q

A 22 year old lady complains of being tired all the time. She has also gained a large amount of weight, feels constipated and cold all the time. Her HR ~ 45 bpm.

What is the diagnosis?
A.	SIADH
B.	Chronic renal failure
C.	Hyperthyroidism
D.	Hypothyroidism
E.	AIDS
F.	Diabetes mellitus
G.	Addison's disease
H.	Colorectal carcinoma
I.	Psychological distress
J.	Anaemia
K.	Glandular fever
A

D. Hypothyroidism

There is weight gain, cold intolerance, fatigue and constipation as well as bradycardia which all point to hypothyroidism. Worldwide, the most common cause is iodine deficiency. Other causes include Hashimoto’s or secondary and tertiary hypothyroidism. It can also result from viral de Quervain’s thyroiditis or postpartum thyroiditis. Symptoms include those mentioned as well as depression, bradycardia, sluggish reflexes, constipation, cold intolerance and muscle cramps. Diagnosis is based on measurement of TSH and thyroid hormones. Treatment is by replacement of T4 with or without T3 in combination. If the patient has normal T3 and T4 but mildly elevated TSH, this is described as subclinical hypothyroidism.

106
Q

A 10 year old African boy who has been in the UK since birth, presents with mild jaundice. O/E his spleen is slightly enlarged. Blood investigation show raised levels of unconjugated bilirubin. Other liver function tests are unremarkable.

What is the diagnosis?
A.	Primary biliary cirrhosis
B.	Crigler-Najjar syndrome
C.	Dubin-Johnson syndrome
D.	Gilbert's syndrome
E.	Primary sclerosing cholangitis
F.	Hepatitis C
G.	Malaria
H.	Hepatitis B
I.	Cholangiocarcinoma
J.	Hepatitis A
K.	Gall stones
L.	Sickle cell anaemia
M.	Carcinoma of head of the pancreas
A

L. Sickle cell anaemia

Africans have higher incidence of sickle cell anaemia. The jaundice here is due to haemolysis and accounts for the raised unconjugated bilirubin. The splenomegaly here is further suggestive. About 8% of black people carry the gene and the prevalence is high in sub-Saharan Africa. The condition is autosomal recessive and therefore occurs in 1 in 4 pregnancies where both parents carry the sickle gene. Sickling occurs when RBCs containing HbS become distorted into a crescent shape. Patients with sickle cell anaemia have no HbA at all. If both parents carry the sickle cell gene, there is a 1 in 4 chance of giving birth to a child with sickle cell anaemia. Sickle cell disease also includes other conditions such as HbS from one parent with another abnormal Hb or beta thalassaemia from the other parent such as HbS-Beta thal and HbSC. Treatment goals include fluid replacement therapy, pain management where necessary and symptomatic control.

107
Q

A patient presents with a cold and mild jaundice. Blood tests show a raised unconjugated bilirubin, normal ALP and ALT. His brother has had episodes of jaundice too.

What is the diagnosis?
A.	Primary biliary cirrhosis
B.	Crigler-Najjar syndrome
C.	Dubin-Johnson syndrome
D.	Gilbert's syndrome
E.	Primary sclerosing cholangitis
F.	Hepatitis C
G.	Malaria
H.	Hepatitis B
I.	Cholangiocarcinoma
J.	Hepatitis A
K.	Gall stones
L.	Sickle cell anaemia
M.	Carcinoma of head of the pancreas
A

D. Gilbert’s syndrome

Gilbert’s occurs in an asymptomatic patient, often as an incidental finding or mild jaundice occuring in adolescence/young adult age, or during times of physiological stress (this patient has a cold). There is elevated unconjugated BR with other liver tests being normal. The blood smear is also normal with normal reticulocyte count, and normal Hb indicating that this is not due to haemolysis. It is a common syndrome and is not really a disease, more a physiological variant. No treatment is needed and this condition is due to decreased UDPGT activity leading to decreased conjugation of unconjugated bilirubin, leading to elevated levels. Positive FH is common as this condition is most likely transmitted in an autosomal recessive pattern.

108
Q

An 18 year old student has just returned from holiday in Africa. He is jaundiced and has moderate hepatomegaly. His blood tests reveal increased serum transaminases and elevated bilirubin. He also has specific IgM antibodies.

What is the diagnosis?
A.	Primary biliary cirrhosis
B.	Crigler-Najjar syndrome
C.	Dubin-Johnson syndrome
D.	Gilbert's syndrome
E.	Primary sclerosing cholangitis
F.	Hepatitis C
G.	Malaria
H.	Hepatitis B
I.	Cholangiocarcinoma
J.	Hepatitis A
K.	Gall stones
L.	Sickle cell anaemia
M.	Carcinoma of head of the pancreas
A

J. Hepatitis A

IgM anti-hepatitis A virus is positive here which is highly sensitive and specific combined with the typical symptoms this student displays. IgM antibodies are detectable typically 5-10 days before symptom onset and remain raised for 4-6 months. It can be ordered alongside IgG anti-HAV and is a cheap and simple test. IgG rises soon after IgM and stays elevated for life so a positive IgG can mean prior infection or recent disease. Again, a cheap and simple to carry out test.
Hepatitis A is primarily transmitted via the faecal-oral route. After the virus is consumed and absorbed, it replicates in the liver and is excreted in the bile (to be re-transmitted). Transmission usually precedes symptoms by about 2 weeks and patients are non-infectious 1 week after onset of jaundice. The history can reveal risk factors such as living in an endemic area, contact with an infected person, homosexual sex or a known food-borne outbreak. This is classically, in EMQs, associated with shellfish which is harvested from sewage contaminated water. If the patient has other liver diseases such as HBV or HCV or cirrhosis then there is a higher risk of fulminant HAV infection. The clinical course of HAV consists of a pre-icteric phase, lasting 5-7 days, consisting characteristically of N&V, abdominal pain, fever, malaise and headache. Rarer symptoms may be present such as arthralgias and even severe thrombocytopenia and signs that may be found include splenomegaly, RUQ tenderness and tender hepatomegaly as well as bradycardia. The icteric phase is characterised by dark urine, pale stools, jaundice and pruritis. When jaundice comes on, the pre-icteric phase symptoms usually diminish, and jaundice typically peaks at 2 weeks. However, a fulminant course runs in <1% of patients with worsenining jaundice and encephalopathy. Serum transaminases may reach in excess of 10,000 units, although there is little correlation between the level and disease severity. ALT is typically higher than AST.

109
Q

A 32 year old woman with ulcerative colitis presents with jaundice, pruritis, RUQ pain and splenomegaly. On direct questioning she admits to having dark urine and pale stools. Her ALP and her conjugated bilirubin is raised.

What is the diagnosis?
A.	Primary biliary cirrhosis
B.	Crigler-Najjar syndrome
C.	Dubin-Johnson syndrome
D.	Gilbert's syndrome
E.	Primary sclerosing cholangitis
F.	Hepatitis C
G.	Malaria
H.	Hepatitis B
I.	Cholangiocarcinoma
J.	Hepatitis A
K.	Gall stones
L.	Sickle cell anaemia
M.	Carcinoma of head of the pancreas
A

E. Primary sclerosing cholangitis

Primary sclerosing cholangitis is a cholestatic liver disease which causes bile duct destruction, cirrhosis and end-stage liver disease. It predominantly affects young and middle-aged men, often with underlying inflammatory bowel disease. There is an association of PSC with UC (typically) and IBD diagnosis tends to precede that of PSC with a mean time from onset of IBD to PSC of 9 years. This patient has evidence of obstructive jaundice and pruritis. The diagnosis involves laboratory tests in combination with cholangiography. There are no specific auto-antibodies specific to or diagnostic of PSC but a number of serum autoantibodies such as ANCA are often present in PSC patients. There is currently no effectively medical therapy and the only treatment option in those with advanced disease is to have a liver transplant. The leading cause of death in these patients is liver failure and cholangiocarcinoma. The latter is a relatively common complication and should be suspected in patients who present with rapidly progressing jaundice, weight loss or abdominal pain on a background of PSC.

110
Q

A 60 year old woman presents with intractable gnawing epigastric pain, weight loss and dyspepsia. O/E she is jaundiced, cachectic, feverish and has an enlarged gall bladder. Blood tests show an increased conjugated bilirubin and increased alkaline phosphatase.

What is the diagnosis?
A.	Primary biliary cirrhosis
B.	Crigler-Najjar syndrome
C.	Dubin-Johnson syndrome
D.	Gilbert's syndrome
E.	Primary sclerosing cholangitis
F.	Hepatitis C
G.	Malaria
H.	Hepatitis B
I.	Cholangiocarcinoma
J.	Hepatitis A
K.	Gall stones
L.	Sickle cell anaemia
M.	Carcinoma of head of the pancreas
A

M. Carcinoma of the head of the pancreas

Pancreatic cancer (of the head) typically presents with painless obstructive jaundice and weight loss and generally presents late. Whipple’s procedure or Traverso-Longmire procedure (pancreaticoduodenectomy) offers the only hope of a cure but only a small minority are elegible for these procedures. The first tests to order are an abdominal USS and LFTs. Note Courvoisier’s law: Jaundice and a palpable painless gallbladder is unlikely to be caused by gallstones. The tumour marker for pancreatic cancer is CA19-9 which is useful in preoperative staging.

111
Q

A 23 year old woman presents with a cough and SOB for 24 hours. O/E she is distressed tahycardicc and resp raet of 25/min. She has a widespread bilateral expiratory wheeze.

What is the most appropriate treatment?
A.	Oral clarithromycin
B.	Nebulised salbutamol
C.	IM adrenaline
D.	Opiate
E.	IV cefuroxime
F.	Salbutamol inhaler
G.	Diuretic
H.	Oral penicillin
I.	Fluids, bed rest
J.	Cough syrup
A

B. Nebulised salbutamol

This patient is having an asthma attack. The best option here is nebulised salbutamol initially. Early systemic corticosteroids and supplemental oxygen should be considered and the patient’s status needs to be monitored regularly.

112
Q

A 30 year old teacher presents to her GP with a couch and SOB worsening over 24 hours. O/E she is pyrexial with no abnormal breath sounds. Her CXr shows bilateral shadowing. Her WCC is normal but she has abnormal liver function tests.

What is the most appropriate treatment?
A.	Oral clarithromycin
B.	Nebulised salbutamol
C.	IM adrenaline
D.	Opiate
E.	IV cefuroxime
F.	Salbutamol inhaler
G.	Diuretic
H.	Oral penicillin
I.	Fluids, bed rest
J.	Cough syrup
A

A. Oral clarithromycin

This is a CAP which is confirmed by CXR shadowing, which would show airspace shadowing with air bronchograms. The history is also consistent with respiratory symptoms and pyrexia. The most sensitive test for CAP is a CXR. The treatment is initially empirical with antibiotics and management would be guided by this patient’s CURB-65 score. A macrolide is the first choice in adults with no contra-indications without a recent course of antibiotics or risk of drug resistance, but always check local prescribing policies. Azithromycin, clarithromycin or erythromycin are all valid choices here.

113
Q

A 39 year old male smoker has had a cough productive of A 40-year-old male smoker with COPD is in hospital with a 2 week cough productive of green sputum with SOB for 24 hours. Temperature is 39.3ºC, RR 35/min, and HR 120/min. There is dullness to percussion and reduced breath sounds at the left base. He is given nebulised salbutamol and systemic corticosteroids.

What is the most appropriate treatment?
A.	Oral clarithromycin
B.	Nebulised salbutamol
C.	IM adrenaline
D.	Opiate
E.	IV cefuroxime
F.	Salbutamol inhaler
G.	Diuretic
H.	Oral penicillin
I.	Fluids, bed rest
J.	Cough syrup
A

E. IV cefuroxime

This is an acute infective exacerbation of COPD and the patient has already received nebulised salbutamol and corticosteroids. The next important thing to do is to give IV antibiotics. Inpatient therapy for severe infections can be with IV ceftriaxone with azithromycin. High risk patients should receive piperacillin and tazobactam, or meropenem, to cover for pseudomonas.

114
Q

An 80 year old man presents with a nocturnal cough & white sputum for 2 weeks. There are bilateral basal crepitations on chest examination. The CXR shows an enlarged heart and a small right pleural effusion.

What is the most appropriate treatment?
A.	Oral clarithromycin
B.	Nebulised salbutamol
C.	IM adrenaline
D.	Opiate
E.	IV cefuroxime
F.	Salbutamol inhaler
G.	Diuretic
H.	Oral penicillin
I.	Fluids, bed rest
J.	Cough syrup
A

G. Diuretic

This patient has developed pulmonary oedema which accounts for the history and examination findings. The CXR indicates the cause is heart failure. He will require diuretics and fluid restriction to deal with his overloaded state. Patients need to be sat upright to improve the SOB and IV access needs to be established. Oxygen, morphine, diuretics (frusemide or another loop diuretic) and nitrates will be given. Once stable, medical treatment of heart failure should be started which involves in the first instance, an ACE inhibitor followed by beta blockade.

115
Q

A 60 year old man has been diagnosed as having bronchial carcinoma with secondaries. He is distressed by chronic coughing.

What is the most appropriate treatment?
A.	Oral clarithromycin
B.	Nebulised salbutamol
C.	IM adrenaline
D.	Opiate
E.	IV cefuroxime
F.	Salbutamol inhaler
G.	Diuretic
H.	Oral penicillin
I.	Fluids, bed rest
J.	Cough syrup
A

D. Opiate

Opiates are commonly prescribed for analgesia and can also be used as an anti-tussive in this case (suppresses ACh and NK release needed for cough activation and inteferes with serotonin receptors in the DRN). They are also abused for their euphoric effects mediated by their action on dopamine release at the nucleus accumbens. They have a host of side effects: GI effects (receptors present on the myenteric and submucosal plexus) of decreased gastric motility, emptying and increased gut water reabsorption… in short, constipation; Respiratory depression (desensitises central chemoreceptors to lessen the response to PaCO2); N&V by stimulating the CTZ; Pupillary constriction via the Edinger-Westphal nucleus; And causes symptoms like pruritis, urticaria, hypotension through histamine release via a direct effect on mast cells.

116
Q

A 32 year old man with a 10 year history of heroin addiction and a string of convictions for theft. He is at risk of HIV and other transmissible diseases due to needle sharing. He wants to stabilise his lifestyle but doesn’t feel ready to give up opiates.

What is the most appropriate treatment?
A.	Motivational interviewing
B.	Physician advice
C.	Alcoholics anonymous
D.	 Naloxone
E.	Controlled drinking
F.	Methadone programme
G.	Treatment under Section 3, Mental Health Act 1984
H.	Aversion treatment
I.	Inpatient detoxification with chlordiazepoxide
J.	Disulfiram
K.	Antipsychotic medication
L.	Token economy
A

F. Methadone program

This patient is a regular recreational opiate user and should be educated about the dangers, particularly of overdose. Needle sharing puts the patient at risk of infections such as HIV and HCV and advice should also be given on the use of clean needles or techniques on how to clean needles. This patient should be commended on wanting to take part in a detoxification programme for his chronic problem. Methadone is a drug which is used in opioid dependence as substitution therapy. Buprenorphine is another drug which can be used for this purpose and both are recommended in NICE guidelines. This medication can be started with a short period of stabilisation followed by either a withdrawal regimen or maintenance treatment, which reduces drug use and crime and improves health.
Methadone is a long-acting opioid agonist, usually given as a single daily oral solution. It has a long half life and takes 3-10 days for plasma levels to reach a steady state. Buprenorphine is a partial opioid receptor agonist (i.e. it has both opioid agonist and antagonist properties) and some patients may prefer this drug as it is less sedating than methadone so may be more suitable for people who drive around, for instance. Adjunctive therapy may be needed to manage withdrawal symptoms. Loperamide can be used for diarrhoea, mebeverine for stomach cramps, paracetamol and NSAIDs for muscular pains and headaches, and metaclopramide or prochlorperazine for N&V. Lofexidine is an alpha-2 agonist which can be used to alleviate physical symptoms of opioid withdrawal. Remember that naloxone is an opioid-receptor antagonist used to reverse opioid overdose (those dependent can be given a supply in case they ‘accidentally’ overdose) and naltrexone is prescribed to prevent relapse in formerly dependent patients.

117
Q

A 43 year old businessman, who has a history of alcohol dependence but has managed to stop drinking. He is afraid of relapsing during a forthcoming business trip and wants help to remain abstinent from alcohol.

What is the most appropriate treatment?
A.	Motivational interviewing
B.	Physician advice
C.	Alcoholics anonymous
D.	 Naloxone
E.	Controlled drinking
F.	Methadone programme
G.	Treatment under Section 3, Mental Health Act 1984
H.	Aversion treatment
I.	Inpatient detoxification with chlordiazepoxide
J.	Disulfiram
K.	Antipsychotic medication
L.	Token economy
A

J. Dilsulfiram

Disulfiram is used as an adjunct in the treatment of alcohol dependence. It leads to an unpleasant systemic reaction after taking even the smallest amount of alcohol as it causes acetaldehyde to accumulate in the body. Disulfiram blocks the metabolism of alcohol. It is only effective if taken daily and symptoms can occur within 10 minutes of ingesting any alcohol and include facial flushing, throbbing headache, palpitations, tachycardia, N&V and possible arrhythmias, hypotension and collapse with large doses of alcohol. These reactions can last several hours. Even the smallest amounts of alcohol in medications and even mouthwash can lead to a reaction. After stopping treatment, alcohol still should be avoided for at least a week. Other medications that can be used to prevent relapse and support abstinence include acamprosate (which stabilises glutamate and GABA systems) and naltrexone which is an opioid antagonist.

118
Q

A 25 year old male student drinks about 4 pints of beer a day, every day. He has no symptoms of alcohol dependency or physical problems. He is concerned his level of drinking may be harmful.

What is the most appropriate treatment?
A.	Motivational interviewing
B.	Physician advice
C.	Alcoholics anonymous
D.	 Naloxone
E.	Controlled drinking
F.	Methadone programme
G.	Treatment under Section 3, Mental Health Act 1984
H.	Aversion treatment
I.	Inpatient detoxification with chlordiazepoxide
J.	Disulfiram
K.	Antipsychotic medication
L.	Token economy
A

B. Physician advice

This (likely medical) student is not dependent nor does he have any physical symptoms but is concerned and aware that his drinking is harmful. At this stage he needs advice on the dangers of excessive alcohol consumption which is the first line intervention for those with patterns of problematic alcohol use but are not dependent (as well as those with mild dependence). The consultations can consist of one or more sessions with a doctor during which feedback can be given about the patient’s alcohol use and the consequences in a supportive and empathic manner. The doctor and patient can work together to develop an action plan for cutting down or eliminating alcohol use and then assessing how effective this is with subsequent visits. Motivational interviewing is a technique which can also be deployed in this situation.

119
Q

A 33 year old homeless man drinks a bottle of whisky per day. He has begun to have episodes of amnesia. He wants to stop drinking. When he last tried to give up drinking, he suffered a grand mal convulsion.

What is the most appropriate treatment?
A.	Motivational interviewing
B.	Physician advice
C.	Alcoholics anonymous
D.	 Naloxone
E.	Controlled drinking
F.	Methadone programme
G.	Treatment under Section 3, Mental Health Act 1984
H.	Aversion treatment
I.	Inpatient detoxification with chlordiazepoxide
J.	Disulfiram
K.	Antipsychotic medication
L.	Token economy
A

I. Inpatient detoxification with chlorodiazepoxide

Chlordiazepoxide is a long-acting benzodiazepine which is used to attenuate alcohol withdrawal symptoms. People with severe dependence should undergo withdrawal in an inpatient setting – withdrawal in those who are severely dependent without medical support can lead to seizures, DT and death. A symptom triggered flexible regimen is typically used in hospital and continued assessment and monitoring is carried out for 24-48 hours, usually followed by a 5 day reducing dose schedule. When BZDs are contraindicated, carbamazepine can be used as an alternative. Clomethiazole can also be used in the acute withdrawal setting although BZDs are preferred. Patients with marked agitation or hallucinations, and those at risk of DT can be given antipsychotic drugs such as haloperidol as an adjunct. This patient should also be given parenteral thiamine (as Pabrinex) due to the risk of developing Wernicke’s encephalopathy from chronic alcohol abuse.

120
Q

A 45 year old man would like to have support to give up drinking.

What is the most appropriate treatment?
A.	Motivational interviewing
B.	Physician advice
C.	Alcoholics anonymous
D.	 Naloxone
E.	Controlled drinking
F.	Methadone programme
G.	Treatment under Section 3, Mental Health Act 1984
H.	Aversion treatment
I.	Inpatient detoxification with chlordiazepoxide
J.	Disulfiram
K.	Antipsychotic medication
L.	Token economy
A

C. Alcoholics anonymous

Alcoholics Anonymous is a help group where people share their experiences and help each other recover from alcoholism.

121
Q

A 19 year old student complaining of amenorrhoea for 9 months. Weight loss, generalised weakness and depression. She has started a vegan diet a year ago. Her BMI is 16.

What is the diagnosis?
A.	Cardiac failure
B.	Renal failure
C.	Anorexia nervosa
D.	Tuberculosis
E.	Malabsorption
F.	Addison's disease
G.	Depression
H.	Hyperthyroidism
I.	Diabetes mellitus
J.	HIV
K.	Malignancy
L.	Liver failure
M.	EBV
A

C. Anorexia nervosa

Anorexia nervosa often occurs in women which is a key risk factor. Puberty/adolescence and obsessive personality traits are additional risks. There is a higher incidence reported in western cultures (presumably skinny models on TV, adverts etc…) and studies on immigrants moving to a western culture exhibited a higher incidence. There is also postulated to be a genetic contribution from twin studies.Those who in higher socioeconomic classes are more affected. BMI is usually <17.5 and the patient often has a fear of weight gain and will refuse intervention to gain weight. The patient often fails to acknowledge how thin they are and can indicate how some parts of her body are ‘big’. Amenorrhoea is also a common complaint. Often anorexics eat vegetarian or vegan diets and you should inquire about the patient’s eating habits.There are two subtypes of AN. Restrictive where the patient will diet and exercise or the bingeing/purging types where there is also calorie restriction but ocassional binges and purging afterwards. There is also a tendency to laxative, diuretic and enema abuse.

122
Q

A 45 year old woman complains of hand tremors, loose stools and is very anxious.

What is the diagnosis?
A.	Cardiac failure
B.	Renal failure
C.	Anorexia nervosa
D.	Tuberculosis
E.	Malabsorption
F.	Addison's disease
G.	Depression
H.	Hyperthyroidism
I.	Diabetes mellitus
J.	HIV
K.	Malignancy
L.	Liver failure
M.	EBV
A

H. Hyperthyroidism

TSH is the initial screening test and if suppressed, T4/T3 levels are measured. Treatment aims to normalise thyroid function and is achieved by radioactive iodine, anti-thyroid medications or with surgery. They are all effective and relatively safe options. Symptomatic therapy is given with beta blockers such as propranolol.

123
Q

A 40 year old unmarried actor has noticed recent weight loss. Although he attributed this to stress you are concerned when you detect generalised lymphandenopathy. Blood count shows neutropenia and thrombocytopenia.

What is the diagnosis?
A.	Cardiac failure
B.	Renal failure
C.	Anorexia nervosa
D.	Tuberculosis
E.	Malabsorption
F.	Addison's disease
G.	Depression
H.	Hyperthyroidism
I.	Diabetes mellitus
J.	HIV
K.	Malignancy
L.	Liver failure
M.	EBV
A

J. HIV

HIV is a retrovirus and there are two types, HIV 1 which is the main virus responsible and HIV 2 which is restricted to parts of West Africa. Weight loss is common in HIV and if more than 10% body weight is lost of BMI reduces to 18.5, this is an indication of more severe immunocompromise. Weight loss in HIV may result from malnutrition, co-existent TB infection or HIV wasting syndrome, the latter being an AIDS defining illness. Generalised lymphadenopathy is also common and is characterised by the painless enlargement of 2 more more non-contiguous sites of >1cm for >3 months. Neutropenia is also seen due to CD4 deficiency and thrombocytopenia may also be seen along with an anaemic picture.
There are WHO (stage 1-4) and CDC criteria used in clinical staging. This patient needs to have a CD4 count, HBV and HCV screen, VDRL (syphilis), tuberculin skin test (TB) and CXR. HIV viral load will also be assessed. Prophylaxis and immunisations should be considered against infections such as hepatitis, influenza, PCP and TB. When to initiate HAART depends on the clinical stage, CD4 and co-morbidities. This patient will need to be started on HAART. Classes of antiretrovirals include NRTIs, NNRTIs, protease inhibitors, fusion inhibitors and integrase inhibitors.

124
Q

A 70 year old man with a history of 10kg weight loss over the previous 3 months. More recently he developed acute lower back pain. Presents to A&E with coughing & sputum. CXR shows left lower lobe pneumonia.

What is the diagnosis?
A.	Cardiac failure
B.	Renal failure
C.	Anorexia nervosa
D.	Tuberculosis
E.	Malabsorption
F.	Addison's disease
G.	Depression
H.	Hyperthyroidism
I.	Diabetes mellitus
J.	HIV
K.	Malignancy
L.	Liver failure
M.	EBV
A

K. Malignancy

This person likely has lung cancer which has resulted in the significant weight loss of 10kg. This is post-obstructive pneymonia which is common in lung cancer patients and is caused, most of the time, by a large and centrally obstructing tumour. It is essential to relieve this obstruction in this case and many techniques can be tried. There is also bone pain here in the lower spinal column which is due to metastases. The prognosis here is not good.

125
Q

A 25 year old woman with fatigue & weight loss. She gives a history of frequent loose stools with abdominal pain. Full blood count revealed iron deficiency anaemia.

What is the diagnosis?
A.	Cardiac failure
B.	Renal failure
C.	Anorexia nervosa
D.	Tuberculosis
E.	Malabsorption
F.	Addison's disease
G.	Depression
H.	Hyperthyroidism
I.	Diabetes mellitus
J.	HIV
K.	Malignancy
L.	Liver failure
M.	EBV
A

E. Malabsorption

There is frequent loose stools here and abdominal pain. Combined with the IDA, this points to malabsorption. This could well be a presentation of coeliac disease – IDA is one of the most common clinical presentations and abdominal pain and diarrhoea are common. Coeliac disease is a systemic autoimmune condition triggered by dietary gluten peptides found in grains. It is a relatively common condition. The only treatment is a strict gluten-free diet for life.

126
Q

A 45-year old lady with a tremor of the wrist (when extended). Other findings on examination include jaundice, spider naevi, ascites.

What is the diagnosis?
A.	Brain tumour
B.	MS
C.	Dystonia
D.	Sydenham's chorea
E.	Exaggerated physiological tremor
F.	Asterixis
G.	Alcohol withdrawal
H.	Cerebellar tremor
I.	Parkinson's Disease
A

F. Asterixis

Asterixis is a flapping tremor, or a liver flap, which is a coarse tremor of the hand when the wrist is held extended. It is caused in this case by liver failure and can be a sign of hepatic encephalopathy. The signs found on examination point to decompensated chronic liver disease.

127
Q

A 17-year-old girl with joint pain, fever, skin rash developed bilateral involuntary jerky movements.

What is the diagnosis?
A.	Brain tumour
B.	MS
C.	Dystonia
D.	Sydenham's chorea
E.	Exaggerated physiological tremor
F.	Asterixis
G.	Alcohol withdrawal
H.	Cerebellar tremor
I.	Parkinson's Disease
A

D. Sydenham’s chorea

Chorea features as part of the acute presentation in 5-10% of patients with rheumatic fever. It can also occur as an isolated event up to 6 months after the initial GABHS infection. It is named Sydenham chorea after the doctor who described St Vitus Dance in the 17th century. Choreiform movements can affect the whole body or just one side of the body, in which case it is referred to as hemi-chorea. The head is often involved with erratic facial movements that resemble grimaces, grins and growns, and the tongue may be affected to resemble a bag of worms when protruded, and protrusion cannot be maintained. In severe cases the patient may have an impaired ability to eat. Chorea disappears with sleep and is made worse by purposeful movements. When the patient is asked to grip the doctor’s hand, the patient will be unable to maintain grip and rhythmic squeezing occurs. There are two signs to look out for in these patients. The first is the spooning sign, which is a flexion at the wrist with finger extension when the hand is held extended. The pronator sign is the second which is when the palms turn outwards when held above the head. Both are consistent with chorea.
Remember that the 5 major manifestations of acute rheumatic fever are carditis, polyarthritis, chorea, erythema marginatum and SC nodules – the most common of which are carditis and polyarthritis. Primary episodes occur mainly in children aged 5-14 and are rare in those over 30. The greatest burden of disease remains in the developing countries and in populations of people living in poverty.

128
Q

A young lady with thyrotoxicosis displays a hand tremor.

What is the diagnosis?
A.	Brain tumour
B.	MS
C.	Dystonia
D.	Sydenham's chorea
E.	Exaggerated physiological tremor
F.	Asterixis
G.	Alcohol withdrawal
H.	Cerebellar tremor
I.	Parkinson's Disease
A

E. Exaggerated physiological tremor

The tremor seen in hyperthyroidism is usually fine and is a exaggerated physiological phenomenon. Treatment aims to normalise thyroid function and is achieved by radioactive iodine, antithyroid medications or with surgery. They are all effective and relatively safe options. Symptomatic therapy is given with beta blockers such as propranolol.

129
Q

A 60-year-old with lung cancer finds it difficult to stand upright and walk. He has ataxia.

What is the diagnosis?
A.	Brain tumour
B.	MS
C.	Dystonia
D.	Sydenham's chorea
E.	Exaggerated physiological tremor
F.	Asterixis
G.	Alcohol withdrawal
H.	Cerebellar tremor
I.	Parkinson's Disease
A

H. Cerebellar tremor

A cerebellar tremor can be seen in MS, trauma or stroke. The history may feature complaints of incoordination and imbalance, a possible FH of cerebellar ataxia or PMH of MS, head trauma, stroke or cerebellar haemorrhage. The tremor itself is a coarse and irregular kinetic tremor which is generated proximally. There may also be abnormal finger-to-nose testing and heel-to-shin testing, dysdiadochokinesia, wide-based ataxic gait (like the patient is drunk) as well as dysarthria (speech problems). The first test to order is an MRI of the patient’s head which may show signs of cerebellar atrophy or may suggest changes of demyelination seen in MS or changes of stroke, trauma or haemorrhage. You may also want to perform other tests such as thyroid function tests to exclude hyperthyroidism.

130
Q

A 40-year-old lady presents with parasthaesia in the left lower limb and displays an intention tremor. She recalls that 6 months prior to this, she had an episode of temporary visual loss which she found frightening.

What is the diagnosis?
A.	Brain tumour
B.	MS
C.	Dystonia
D.	Sydenham's chorea
E.	Exaggerated physiological tremor
F.	Asterixis
G.	Alcohol withdrawal
H.	Cerebellar tremor
I.	Parkinson's Disease
A

B. MS

MS is a demyelinating CNS condition which is characterised by 2 or more episodes of neurological dysfunction which are separated in both time and space. MS classically presents in white women aged 20-40 with temporary visual/sensory loss although any presentation can occur. MRI is a sensitive test but less specific than spinal MRI, however, spinal MRI is abnormal in fewer cases. Treatment aims at treating the attack, preventing future attacks and symptomatic treatment of problems like bladder dysfunction, pain and fatigue.

131
Q

The parents noticed that their 7-week baby has a lump in her neck. The swelling seems to come and go. O/E the lump is located in the posterior triangle of the neck, it transilluminates.

What is the diagnosis?
A.	Pharyngeal pouch
B.	Dermoid cyst
C.	Metastatic carcinoma
D.	Salivary gland stone
E.	Carotid body tumour
F.	Thyroglossal cyst
G.	Branchial cyst
H.	Parotitis
I.	Thyroid nodule
J.	Cystic hygroma
A

J. Cystic hygroma

A cystic hygroma is a cystic lymphatic lesion which occurs as a congenital birth defect. It can arise anywhere but is classically found in the left posterior triangle of the neck and is the most common lymphangioma. There are large cyst like cavities in this lesion containing a watery fluid and as such it transilluminates. They are a benign lesion but can be disfiguring and is a condition which very rarely presents in adulthood, and tends to affect children. It can be seen as part of Noonan’s syndrome, which is an autosomal dominant disorder characteristically with short stature, chest deformity, congenital heart defects and unusual facial features.

132
Q

An 10 year old girl is unwell and complains of painful swallowing. She has bilateral parotid enlargement.

What is the diagnosis?
A.	Pharyngeal pouch
B.	Dermoid cyst
C.	Metastatic carcinoma
D.	Salivary gland stone
E.	Carotid body tumour
F.	Thyroglossal cyst
G.	Branchial cyst
H.	Parotitis
I.	Thyroid nodule
J.	Cystic hygroma
A

H. Parotitis

Parotitis is inflammation of one or both parotid glands. This is a form of sialadenitis, with the parotid glands here being enlarged and inflammed. Pain and dysphagia are common. Fever may also be noted in acute bacterial cases. Patients with acute bacterial sialadenitis may give a history of recent surgery or the use of medications such as antihistamines, antidepressants or anticholinergics, which may also lead to xerostomia. Stones in the glandular ducts (sialoliths) can result in obstructive sialadenitis, although there is no evidence from this case that this is the cause. Other causes include Sjogren’s syndrome which can cause an autoimmune sialadenitis. In terms of management, pus swabs should be taken from any exudate and imaging should be done to exclude an obstructing salivary gland stone or evolving abscess. Plain dental radiographs are the investigation of choice with CT imaging done if plain radiographs come back negative and a stone is suspected. USS is useful if ab abscess is suspected. FBC can be done to evaulate WCC and if a fever is present, blood cultures should be done to guide antibiotic therapy.

133
Q

A 18-year-old man is noted to have a swelling in the midline of the neck. It is non-tender and moves up the neck on swallowing and tongue protrusion.

What is the diagnosis?
A.	Pharyngeal pouch
B.	Dermoid cyst
C.	Metastatic carcinoma
D.	Salivary gland stone
E.	Carotid body tumour
F.	Thyroglossal cyst
G.	Branchial cyst
H.	Parotitis
I.	Thyroid nodule
J.	Cystic hygroma
A

F. Thyroglossal cyst

This midline neck swelling moves up on both swallowing and tongue protrusion making this a thyroglossal cyst. It is a cyst that forms from a remnant thyroglossal duct and can hence develop anywhere along the length of this embryological duct, which is a midline structure between the foramen caecum at the back of the tongue and the thyroid gland.

134
Q

A 78 year old presents with dysphagia and regurgitation of food. Recently, he developed a cough. O/E you notice halitosis and a 6 cm lump behind the sternocleidomastoid muscle. The lump is compressible on palpation.

What is the diagnosis?
A.	Pharyngeal pouch
B.	Dermoid cyst
C.	Metastatic carcinoma
D.	Salivary gland stone
E.	Carotid body tumour
F.	Thyroglossal cyst
G.	Branchial cyst
H.	Parotitis
I.	Thyroid nodule
J.	Cystic hygroma
A

A. Pharyngeal pouch

A pharyngeal pouch is also called a Zenker’s diverticulum (it is a false diverticulum – i.e. it does not involve all layers of the oesophageal wall). The dehiscence of Killian lies in the posterior laryngopharynx which is where the wall herniates through giving an outpouching just above the cricopharyngeal muscle. It may help to gave a look at a picture in an anatomy textbook. A pharyngeal pouch can cause symptoms of dysphagia and the sensation of a lump in the neck (which can frequently be misdiagnosed as globus hystericus). There may also be regurgitation of food, cough, halitosis and gurgling noises and the condition is associated with webs. There is usually no pain except in the presence of a carcinoma. Additionally, it may be asymptomatic.
Examination may reveal a swelling in the neck which is usually found on the left side, in the lower part of the anterior triangle, which is soft and may gurgle on palpation (Boyce’s sign). A spasm of coughing may occur on palpation due to spilling of the contents into the larynx. A barium swallow will confirm the dagnosis. Surgical intervetion may be necessary.

135
Q

A 69-year-old lady noticed hard and painless lumps in her neck. She says it is slowly growing. She also mentions that her voice changed recently. The lumps are located mainly in the anterior triangle, deep to upper third of sternocleidomastoid.

What is the diagnosis?
A.	Pharyngeal pouch
B.	Dermoid cyst
C.	Metastatic carcinoma
D.	Salivary gland stone
E.	Carotid body tumour
F.	Thyroglossal cyst
G.	Branchial cyst
H.	Parotitis
I.	Thyroid nodule
J.	Cystic hygroma
A

C. Metastatic carcinoma

The voice change here suggests a primary laryngeal carcinoma which we assume here has metastasized locally and to lymph nodes in the deep cervical chain. The hard nature of the lumps, the fact they are painless and firm suggests cervical metastases which are more common in supraglottic cancer (glottic cancer has a much lower rate of cervical metastases). Cervical lymphadenopathy is common and the size, location, mobility and degree of firmness indicate the degree of progression of laryngeal malignancy. Hoarseness, dysphonia, sore throat, dysphagia, referred otalgia, vocal cord lesions and persistent neck mass/adenopathy for >3 weeks are sentinel signs of laryngeal cancer, which is frequently associated with smoking and alcohol use. An MDT approach is taken with the aim of treatment with organ preservation, with salvage surgical resection offered in advanced stage disease. The rate of organ preservation has significantly improved in the last 30 or so years.

136
Q

A 75-year-old gentleman complains of dyspnoea and orthopnoea. He has been increasingly tired and lethargic past few weeks.

What is the diagnosis?
A.	Left heart failure
B.	Aortic dissection
C.	 Right heart failure
D.	Pulmonar Embolus
E.	Tietze syndrome
F.	Rheumatic fever
G.	Pericarditis
H.	Endocarditis
I.	Dressler's syndrome
J.	MI
A

A. Left heart failure

There are no expressed signs or symptoms of RVF here such as peripheral oedema, ascites, elevated JVP and hepatomegaly. Nocturia may be a symptom as fluid returns from the legs when the patient lies down flat. RVF leads to a backlog of blood and congestion of the systemic capillaries. LVF, on the other hand, causes congestion in the pulmonary circulation so the symptoms are respiratory. As seen in this patient, there is SOB and there may be the classic cough productive of frothy sputum – a sign of pulmonary oedema. On respiratory examination, pulmonary oedema due to LVF may give audible fine late inspiratory crepitations at the bases. There may also be orthopnoea. This is why you can ask patients in a cardiac history how many pillows they sleep with. PND can also occur as well as ‘cardiac asthma’.
Initial investigations should include ECG, CXR, TTE and bloods including BNP levels. First line treatment is with an ACE inhibitor which reduces morbidity and mortality. Salt and fluid restriction is also beneficial. All patients with chronic heart failure will also receive a beta blocker such as carvedilol. Other adjuncts include spironolactone, diuretics, hydralazine and a nitrate, and digoxin. The NYHA classification criteria can be used based on symptoms to describe functional limitations and ranges from Class I to Class IV with symptoms occuring at rest. Many patients are asymptomatic for long periods of time because mild cardiac impairment is balanced by compensation.

137
Q

A 32 year old man presents with severe tearing chest pain which radiates to the back. The doctor notes that he is incredibly tall.

What is the diagnosis?
A.	Left heart failure
B.	Aortic dissection
C.	 Right heart failure
D.	Pulmonar Embolus
E.	Tietze syndrome
F.	Rheumatic fever
G.	Pericarditis
H.	Endocarditis
I.	Dressler's syndrome
J.	MI
A

B. Aortic dissection

The tearing chest pain suggests aortic dissection. There may also be interscapular pain with dissection of the descending aorta. Dissecting aneurysms are either type A, which involves the ascending aorta, or type B. Type A dissections require urgent surgery whereas type B can be managed medically if it is not complicated by end organ ischaemia. BP differential between the 2 arms is a hallmark feature. Pulse differences may also be present in the lower limbs. There may also be the diastolic murmur of AR in proximal dissections. A CT scan is indicated as soon as a diagnosis of aortic dissection is suspected and should be from the chest to the pelvis to see the full extent of the dissecting aneurysm. What you will see is the intimal flap. MRI is more sensitive and specific but is more difficult to obtain acutely.
Features of Marfan’s syndrome or Ehlers-Danlos syndrome may be present. Features include tall stature, arachnodactyly, pectus excavatum, hypermobile joints and narrow face. Marfan’s predisposes to both aneurysms and/or dissections, presumably due to weakness of the aortic wall. Type IV Ehlers-Danlos predisposes to both aneurysms and/or dissections Other strong risk factors include hypertension, atherosclerotic aneurysmal disease, bicuspid aortic valve, coarctation, smoking, FH and annulo-aortic ectasia.

138
Q

A 27-year-old taxi driver presents with sharp, central chest pain, radiating to shoulders. The pain is relieved by sitting forward. Over the last week he has had flu like symptoms.

What is the diagnosis?
A.	Left heart failure
B.	Aortic dissection
C.	 Right heart failure
D.	Pulmonar Embolus
E.	Tietze syndrome
F.	Rheumatic fever
G.	Pericarditis
H.	Endocarditis
I.	Dressler's syndrome
J.	MI
A

G. Pericarditis

Symptoms include a sharp and severe chest pain retrosternally which is worse on inspiration and when supine, relieved by sitting forwards. The classical finding on examination is a friction rub which is said to sound like ‘walking on snow’. There may be diffuse ST elevations on ECG, classically saddle-shaped, an effusion on echocardiography and blood results suggesting inflammation. Complications include tamponade and constrictive pericarditis. Prior viral infection is a risk factor with the most common pericardial infection being viral. Bacterial purulent pericarditis also occurs. The inflammation is due either to direct viral attack or immune mediated damage. Other risk factors include male gender, post-MI (both ‘early’ and Dressler’s), post-pericardiotomy syndrome, neoplasm from local tumour invasion, uraemia and autoimmune conditions such as RA and SLE.

139
Q

A 45-year old gentleman presents to A&E with central crushing chest pain. It started 3 hours ago when the patient was watching TV. The pain is not relieved by his GTN spray.

What is the diagnosis?
A.	Left heart failure
B.	Aortic dissection
C.	 Right heart failure
D.	Pulmonar Embolus
E.	Tietze syndrome
F.	Rheumatic fever
G.	Pericarditis
H.	Endocarditis
I.	Dressler's syndrome
J.	MI
A

J. MI

This patient’s chest pain sounds like it is due to an MI. Chest pain is classically severe and heavy in nature (often described as crushing), located centrally with possible radiation to the left arm or jaw and lasts for >20 minutes. SOB due to pulmonary congestion and sweating due to high sympathetic output are also common symptoms. Risk factors incorporate the standard set of cardiovascular risks such as smoking, high BP, DM, obesity and dyslipidaemia. The fact the patient has GTN spray suggests a medical history of angina and cardiovascular risk factors. An ECG is indicated here. STEMI, new LBBB or confirmed posterior MI is an indication for PCI/thrombolysis. It is worth noting that RV infarction is present in 40% of inferior infarcts so in this case, right sided ECG leads should also be obtained.

140
Q

3 weeks after having an MI, a 65-year-old man presents with sharp chest pain and pyrexia. Blood test reveals anaemia and raised ESR.

What is the diagnosis?
A.	Left heart failure
B.	Aortic dissection
C.	 Right heart failure
D.	Pulmonar Embolus
E.	Tietze syndrome
F.	Rheumatic fever
G.	Pericarditis
H.	Endocarditis
I.	Dressler's syndrome
J.	MI
A

I. Dressler’s syndrome

This is percarditis again, likely to be Dressler’s syndrome. This is believed to be an autoimmune process with myocardial neo-antigens implicated in the aetiology and occurs typically 2-3 weeks post-MI. Typical treatment is with aspirin. It tends to subside in a few days and raised ESR is a lab finding which can be seen. See above for more information on angina.

141
Q

A 79-year-old man complains of increasing breathlessness. On general examination you notice wide pulse pressure and visible carotid pulsation.

What is the diagnosis?
A.	Aortic stenosis
B.	Aortic regurgitation
C.	Innocent murmur
D.	Tricuspid regurgitation
E.	Rheumatic fever
F.	Mitral regurgitation
G.	Mitral stenosis
H.	Infective endocarditis
I.	Atrial septal defect
A

B. Aortic regurgitation

Aortic regurgitation is the leakage of blood back into the LV in diastole. The collapsing pulse is also known as a water hammer or Corrigan’s pulse and describes the rapid rise and quick ‘collapse’ of the arterial pulse resulting in a wide pulse pressure. The murmur in AR is early diastolic in mild cases and increases to pansystolic in severe cases. Risk factors include a bicuspid valve, rheumatic fever, endocarditis, anklylosing spondylitis and Marfan’s. Other commonly seen signs in EMQs, although uncommon in clinical practice include Traube’s (pistol shot sounds over the femoral arteries when compressed), Quincke’s (subungal capillary pulsations), Duroziez’s (systolic and diastolic murmurs over the femoral arteries) and de Musset’s (head bobbing with each heart beat) sign. Other equally uncommon signs (there are many for AR) include Muller’s, Mayen’s, Lighthouse, Hill’s, Becker’s, Landolfi’s, Rosenbach’s, Gerhardt’s, Lincoln’s and Sherman’s sign. The visible carotid pulsation here is Corrigan’s sign, with the pulse characteristic as described previously.
Occasionally although uncommon, an Austin Flint murmur may be heard which is a rumbling mid-diastolic murmur best heard at the apex, produced by the regurgitant jet hitting the LV endocardium. Its presence indicates severe AR and the absence of a loud S1 or an opening snap distinguishes this from the murmur of mitral stenosis.

142
Q

A 33-year-old is admitted with right-sided weakness. He admits to fever, SOB. On auscultation there is an early diastolic murmur.

What is the diagnosis?
A.	Aortic stenosis
B.	Aortic regurgitation
C.	Innocent murmur
D.	Tricuspid regurgitation
E.	Rheumatic fever
F.	Mitral regurgitation
G.	Mitral stenosis
H.	Infective endocarditis
I.	Atrial septal defect
A

H. Infective endocarditis

Any patient presenting with fever and a new murmur should always make you think of bacterial endocarditis. The weakness here may be consistent with a stroke and systemic emboli may be the cause here. The classic new or worsening murmur is actually rare. As are splinter haemorrhages, which EMQs often mention. Other uncommon signs you may find include Janeway lesions (painless macular haemorrhagic plaques on the palms and soles) and Osler nodes (painful nodules on the pads of the fingers and toes). Roth spots may also be seen on fundoscopy. Three sets of blood cultures are required and this patient will have to go for an echocardiogram. The Duke criteria is used for diagnosis.

143
Q

A 73 year old lady presents with 6-month Hx of chest pain on exertion and 2 episodes of collapse in the last month. O/E the pulse is slow rising.

What is the diagnosis?
A.	Aortic stenosis
B.	Aortic regurgitation
C.	Innocent murmur
D.	Tricuspid regurgitation
E.	Rheumatic fever
F.	Mitral regurgitation
G.	Mitral stenosis
H.	Infective endocarditis
I.	Atrial septal defect
A

A. Aortic stenosis

Aortic stenosis can present with chest pain, dyspnoea and syncope. It is characterised by a harsh ejection systolic murmur heard loudest at the right upper sternal edge at end expiration, which radiates up towards the carotids. The pulse pressure is narrow and there may be an associated slow-rising and plateau pulse. Doppler echo is vital for diagnosis and shows a pressure gradient across the narrowed valve orifice. 20% of cases are due to a congenital bicuspid valve. The most common cause of aortic stenosis in adults is calcification of normal trileaflet valves. Clinically stable patients may be considered for surgical repair or TAVR.

144
Q

A pregnant 27-year-old lady is found to have a soft, short systolic murmur at the apex. There are no other signs or symptoms of heart disease.

What is the diagnosis?
A.	Aortic stenosis
B.	Aortic regurgitation
C.	Innocent murmur
D.	Tricuspid regurgitation
E.	Rheumatic fever
F.	Mitral regurgitation
G.	Mitral stenosis
H.	Infective endocarditis
I.	Atrial septal defect
A

C. Innocent murmur

This is a functional murmur which is not caused by a structural cardiac defect. They often reflect hyperdynamic states such as that seen in pregnancy. Functional murmurs tend to be systolic, occuring in an otherwise healthy individual with no symptoms. They are also characteristically position dependent and soft in nature. Ones that occur in children tend to disappear as the child grows. Benign paediatric murmurs include Still’s murmur.

145
Q

A pansystolic murmur in a 53-year-old gentleman with a drinking problem. There is a systolic thrill at the apex.

What is the diagnosis?
A.	Aortic stenosis
B.	Aortic regurgitation
C.	Innocent murmur
D.	Tricuspid regurgitation
E.	Rheumatic fever
F.	Mitral regurgitation
G.	Mitral stenosis
H.	Infective endocarditis
I.	Atrial septal defect
A

F. Mitral regurgitation

MR is loudest at the apex and radiates to the axilla and tends to be around grade 4. It is associated with a systolic thrill at the apex. TTE is the investigation of choice for diagnosis. Chronic MR is associated with a laterally displaced apex beat with LV dilatation. Mitral valve prolapse is a strong risk factor for development of MR. The alcohol history here is hinting at dilated cardiomyopathy as a possible cause.

146
Q

A 38-year-old woman with known polycystic kidney disease suddenly develops a severe occipital headache. She vomited and then lost consciousness.

What is the diagnosis?
A.	Drug overdose
B.	Normal pressure hydrocephalus
C.	Dementia
D.	Wernicke's encephalopathy
E.	Subdural haemorrhage
F.	Extradural haemorrhage
G.	Alcohol withdrawal
H.	Meningitis
I.	Subarachnoid haemorrhage
J.	 Stroke
K.	Hepatic failure
L.	Encephalitis
A

I. Subarachnoid haemorrhage

SAH (bleeding into the subarachnoid space) presents with sudden severe headache patients will often describe as the worst headache of their life, and can often be so bad that they feel like they’ve been kicked in the back of the back. Half of all patients lose consciousness and eye pain with exposure to light can also be seen. Altered mental status is common. SAH occurs most commonly in the 50-55 age group and affects women and black people more than men and white people. The most common cause of non-traumatic SAH is an aneurysm which ruptures. Conditions which predispose to aneurysm formation and SAH include adult PKD, Marfan’s, NF1 and Ehlers-Danlos. Cerebral aneurysms arise around the circle of Willis. A CT scan is indicated, and if unrevealing, this should be followed by an LP. Cerebral angiography can confirm the presence of aneurysms. The patient should be stabilised and this followed by surgical clipping or endovascular coil embolisation, the choice is subject to much current controversy sparked by relatively recent research. Complications can commonly occur and include rebleeding, hydrocephalus and vasospasm.

147
Q

A 22-year-old student is found collapsed in her room. She is unrousable and has a purpuric rash. Her temperature is 39oC.

What is the diagnosis?
A.	Drug overdose
B.	Normal pressure hydrocephalus
C.	Dementia
D.	Wernicke's encephalopathy
E.	Subdural haemorrhage
F.	Extradural haemorrhage
G.	Alcohol withdrawal
H.	Meningitis
I.	Subarachnoid haemorrhage
J.	 Stroke
K.	Hepatic failure
L.	Encephalitis
A

H. Meningitis

This patient has meningitis. Universities are common sites of outbreaks due to crowding. Commonly there will be a headache, fever and nuchal rigidity. There may also be an altered mental status (this patient has LOC), confusion, photophobia and vomiting. Kernig’s sign is uncommon but is positive when attempts to extend the leg are met with resistance when the patient is supine with the thigh flexed to 90 degrees. Another uncommon sign is Brudzinski’s sign and the petechial/purpuric rash seen here, typically associated with meningococcal meningitis.
CT head should be considered before LP if there is any evidence of raised ICP. An LP will confirm the diagnosis with bacterial meningitis showing a low CSF glucose, elevated CSF protein and positive CSF culture/gram stain or meningococcal antigen. Investigations should not delay treatment in this very urgent situation and antibiotics need to be promptly administered. IM benzylpenicillin can be given on the way to the hospital with empirical antibiotic therapy being started on arrival before the causative organism is identified. This can be, for example, with vancomycin and ceftriaxone (or meropenem, cefotaxime, chloramphenicol). Dexamethasone should also be given before the first dose of antibiotics and continued for 4 days as this has been shown to improve outcome and overall mortality in community-acquired bacterial meningitis (though NICE guidelines state it should not be given to those aged <3 months).

148
Q

A 73-year-old presents with increasing confusion and falls over the last couple of months. On further questioning he admits to urinary incontinence. A CT scan of the head shows dilatation of ventricles.

What is the diagnosis?
A.	Drug overdose
B.	Normal pressure hydrocephalus
C.	Dementia
D.	Wernicke's encephalopathy
E.	Subdural haemorrhage
F.	Extradural haemorrhage
G.	Alcohol withdrawal
H.	Meningitis
I.	Subarachnoid haemorrhage
J.	 Stroke
K.	Hepatic failure
L.	Encephalitis
A

B. Normal pressure hydrocephalus

This is normal pressure hydrocephalus – a diagnosis you should suspect in any patient who presents with a gait apraxia (the falls here from loss of balance) and cognitive impairment. There may also be urinary symptoms such as urgency, frequency or urge incontinence. These symptoms tend to be insidious in onset over months or years.There is not significantly raised CSF pressure though perhaps this should not be named ‘normal pressure’ as elevated CSF pressure may be seen but just not significantly so. CT head is the first test to order, although MRI head can also be done, and could be normal but could show mild to moderate ventricular enlargement, periventricular leukomalacia, cerebral infarction, relative preservation of the cortical gyri and sulci and reduced diameter of the corpus callosum and widened callosal angle. This diagnosis can be excluded if an obstructive lesion is seen. If you suspect Parkinson’s then a levodopa challenge should be ordered.

149
Q

A 61 year-old homeless man is confused and drowsy. He is ataxic, complains of blurred vision.

What is the diagnosis?
A.	Drug overdose
B.	Normal pressure hydrocephalus
C.	Dementia
D.	Wernicke's encephalopathy
E.	Subdural haemorrhage
F.	Extradural haemorrhage
G.	Alcohol withdrawal
H.	Meningitis
I.	Subarachnoid haemorrhage
J.	 Stroke
K.	Hepatic failure
L.	Encephalitis
A

D. Wernicke’s encephalopathy

Wernicke’s is due to acute thiamine deficiency, which is a problem in alcoholics. Others at risk include those with AIDS, cancer and treatment with chemotherapy, malnutrition and GIT surgery, especially bariatric procedures. It is a clinically under-diagnosed condition. The classic EMQ triad is of mental change, ophthalmoplegia and gait dysfunction, which is actually only seen in 10% of cases. In reality, the manifestations are varied and a high index of suspicion is needed. Despite there, the manifestiations typically include altered consciousness, gait disorders and eye movement abnormalities. This is an emergency and treatment is with parenteral replacement of thiamine. This avoids permanent neurological damage including later development of Korsakoff’s psychosis, which is irreversible. Note that thiamine should be given before dextrose! Magnesium deficiency also needs to be corrected as it is a co-factor in the functioning of thiamine dependent enzymes.

150
Q

A 55 year old woman is admitted drowsy with slurred speech. You notice yellowing of the sclera and fetor hepaticus.

What is the diagnosis?
A.	Drug overdose
B.	Normal pressure hydrocephalus
C.	Dementia
D.	Wernicke's encephalopathy
E.	Subdural haemorrhage
F.	Extradural haemorrhage
G.	Alcohol withdrawal
H.	Meningitis
I.	Subarachnoid haemorrhage
J.	 Stroke
K.	Hepatic failure
L.	Encephalitis
A

K. Hepatic failure

This patient has decompensated chronic liver disease (he is in liver failure) which has resulted in neurological symptoms associated with hepatic encephalopathy. The brain is exposed to ammonia which bypasses the liver by portosystemic shunting. It is a diagnosis of exclusion and tests will need to be conducted to rule out other potential causes of confusion. The findings of jaundice and fetor hepaticus (liver failure) are signs of liver disease. Think about the other signs you might see like spider naevi and palmar erythema. This patient may also have asterixis which is a coarse flapping tremor. HE is likely caused by a host of factors. This patient’s LFTs will be abnormal and she is likely to have coagulopathy too (PT will be elevated).

151
Q

A 60 year old man presents with bone pain (back, ribs, femur and humerus affected). Investigations show raised ESR, raised calcium, high urea, high creatinine and punched out lesions on his skeletal x-rays.

What is the diagnosis?
A.	Sickle cell anaemia
B.	Thalassaemia
C.	Non Hodgkin’s lymphoma
D.	Iron defeciency anaemia
E.	Acute myeloid leukaemia
F.	Aplastic anaemia
G.	Chronic myeloid leukaemia
H.	Acute lymphoblastic leukaemia
I.	Megaloblastic Anaemia
J.	Chronic lymphocytic leukaemia
K.	Myeloma
A

K. Myeloma

This patient has multiple myeloma. This is characteristed by clonal proliferation of plasma cells in BM and commonly presents with bony pain and symptoms of anaemia. There may also be infections present in 10%. Elevated ESR agrees with this diagnosis. 30% have hypercalcaemia and half have renal impairment, which is associated with a worse prognosis. This explains the elevated urea and creatinine here. The diagnostic test is serum or urine electrophoresis looking for a paraprotein spike of IgG or IgA and light chain urinary excretion (Bence Jones proteins). Bone marrow examination and skeletal survey will also need to be conducted. Bone marrow analysis will help differentiate this from MGUS and solitary plasmacytoma. Bone changes include osteopenia, osteolytic lesions and fractures which this person’s XR indicates with the punched out lytic lesions. Younger patients may be candidates for high-dose chemotherapy and autologous transplantation.

152
Q

A 48 year old male presents with bruising, infections and fatigue. Lab findings indicate a pancytopenia with low reticulocyte count. Bone marrow biopsy is done on which a definitive diagnosis is made.

What is the diagnosis?
A.	Sickle cell anaemia
B.	Thalassaemia
C.	Non Hodgkin’s lymphoma
D.	Iron defeciency anaemia
E.	Acute myeloid leukaemia
F.	Aplastic anaemia
G.	Chronic myeloid leukaemia
H.	Acute lymphoblastic leukaemia
I.	Megaloblastic Anaemia
J.	Chronic lymphocytic leukaemia
K.	Myeloma
A

F. Aplastic anaemia

This is aplastic anaemia characterised here with the pancytopenia (which is common, but diagnosis requires 2 cytopenias out of 3) and the presentation with infections (neutropenia), fatigue (anaemia) and bruising (thrombocytopenia). Risk factors include paroxysmal noctural haemoglobinuria, hepatitis and NSAIDs. If macrocytosis is seen, this may suggest an inherited syndrome such as Fanconi’s anaemia. The reticulocyte count here rules out haemolytic anaemia. The definitive diagnosis is made on biopsy of bone marrow which shows a hypocellular marrow with no abnormal cell populations and no fibrosis. Which conditions would there be abnormal cell populations or fibrosis on bone marrow biopsy?

153
Q

A 50 year old man presents with weight loss, tiredness, fever, night sweats and abdominal pain. On examination his spleen was palpably enlarged and there were multiple bruises on his body. Investigations showed low Hb, WBC 150 x 109/L.

What is the diagnosis?
A.	Sickle cell anaemia
B.	Thalassaemia
C.	Non Hodgkin’s lymphoma
D.	Iron defeciency anaemia
E.	Acute myeloid leukaemia
F.	Aplastic anaemia
G.	Chronic myeloid leukaemia
H.	Acute lymphoblastic leukaemia
I.	Megaloblastic Anaemia
J.	Chronic lymphocytic leukaemia
K.	Myeloma
A

G. Chronic myeloid leukaemia

This is CML which tends to present in the 30-60 age group. At presentation 1/3 may be asymptomatic though if symptomatic, it presents with symptoms including fever, weight loss and night sweats. There is myeloid stem cell proliferation and presents with raised neutrophils, metamyelocytes and basophils. The patient may also describe LUQ discomfort or fullness due to the feeling of a mass due to splenomegaly. There are also symptoms of anaemia here due to BM infiltration of leukaemic cells. Bruises are common and are either spontaneous or from minor trauma. All patients have raised WCC.
CML is associated with the philadelphia chromosome characterised by t(9;22) of bcr-abl. There tends to be massive splenomegaly which is the most common physical finding on examination. This conditon may transform to AML or ALL in what is known as a ‘blast crisis’. CML responds to imatinib, which is an anti-bcr-abl antibody and gives long term remission in most patients.

154
Q

A 20 year old West Indian lady presents with pain in the chest and hip. She has had episodes of this pain in the past. Investigations showed Hb 6g/dl and reticulocytes 15%.

What is the diagnosis?
A.	Sickle cell anaemia
B.	Thalassaemia
C.	Non Hodgkin’s lymphoma
D.	Iron defeciency anaemia
E.	Acute myeloid leukaemia
F.	Aplastic anaemia
G.	Chronic myeloid leukaemia
H.	Acute lymphoblastic leukaemia
I.	Megaloblastic Anaemia
J.	Chronic lymphocytic leukaemia
K.	Myeloma
A

A. Sickle cell anaemia

The reticulocytosis here is as a result of the haemolytic anaemia this patient is suffering due to sickle cell anaemia. The bone pain in the chest and hip is an acute symptom of vaso-occlusive crises. It is generally unusual for someone to reach this age without being aware that they have sickle cell anaemia so in real life, this would be an uncommon way to present. The diagnosis should however be suspected due to the episodes of pain (vaso-occlusive crises) and the findings of anaemia with reticulocytosis.
Sickling occurs when RBCs containing HbS become distorted into a crescent shape. Patients with sickle cell anaemia have no HbA at all. If both parents carry the sickle cell gene, there is a 1 in 4 chance of giving birth to a child with sickle cell anaemia.

155
Q

An 68 year old woman with tierdness, right sided abdominal pain for 2 months. Investigations showed Hb 8 g/dl and MCV 65fl. Colonoscopy shows an ulcerating mass in the ascending colon.

What is the diagnosis?
A.	Sickle cell anaemia
B.	Thalassaemia
C.	Non Hodgkin’s lymphoma
D.	Iron defeciency anaemia
E.	Acute myeloid leukaemia
F.	Aplastic anaemia
G.	Chronic myeloid leukaemia
H.	Acute lymphoblastic leukaemia
I.	Megaloblastic Anaemia
J.	Chronic lymphocytic leukaemia
K.	Myeloma
A

D. Iron deficiency anaemia

IDA is a microcytic hypochromic anaemia characterised by low serum iron, high TIBC and low transferrin saturation and serum ferritin. Bleeding is the principle cause of IDA. IDA is not an end diagnosis and has many causes and this is something that is important to bear in mind at all times. The cause here is from chronic blood loss from what would appear to be possible right sided bowel cancer. Worldwide, the most common cause is hookworm infection, and in the UK, menstrual losses in women. Causes of IDA can be broadly divided into 4 categories: decreased intake, increased loss, increased requirements (such as in pregnancy), and unknown.

156
Q

A 75-year-old gentleman was brought to A&E mildly confused. He has been unwell last couple of days with productive cough, diarrhoea and fever. CXR is shows infiltrates in the RUL and his bloods show hyponatraemia.

What is the diagnosis?
A.	Leptospira interrogans
B.	Streptococcus pneumoniae
C.	Streptococcus viridans
D.	Mycoplasma pneumoniae
E.	Legionella pneumoniae
F.	Streptococcus pyogenes
G.	Shigella flexineri
H.	Staphylococcus aureus
I.	Salmonella enteritidis
J.	Campylobacter jejuni
K.	Pseudomonas aeruginosa
A

E. Legionella pneumoniae

Legionella is a gram negative rod. Legionella infecting the lungs is legionnaires’ disease or Legionella pneumonia whereas non-lung infection is known as Pontiac fever. This bacteria is found in aqueous environments such as lakes and almost all cases are from contaminated water systems, which relates to the risk factors of getting Legionella (recent water exposure like a hot tub). Smoking is also a risk factor. It can cause confusion as well as hyponatraemia, abdominal pain, diarrhoea and bradycardia. Legionella does not grow on routine culture media and diagnosis relies on urine antigen detection, serology or culture on special media.

157
Q

A 65-year-old with CLL develops a respiratory tract infection in hospital after finishing his chemotherapy course. Bacteria grow on MacConkey agar.

What is the diagnosis?
A.	Leptospira interrogans
B.	Streptococcus pneumoniae
C.	Streptococcus viridans
D.	Mycoplasma pneumoniae
E.	Legionella pneumoniae
F.	Streptococcus pyogenes
G.	Shigella flexineri
H.	Staphylococcus aureus
I.	Salmonella enteritidis
J.	Campylobacter jejuni
K.	Pseudomonas aeruginosa
A

K. Pseudomonas aeruginosa

Pseudomonas has virulence factors for lung colonisation and is noted for its type III injection apparatus, which you may remember from your microbiology lectures. This organism is one of the main 5 organisms responsible for hospital acquired (or nosocomial) infections in the UK. Remember that a nosocomial infection is defined as one acquired >48 hours after admission. It is a gram negative bacilli causing hospital acquired pneumonia and UTI. It particularly affects immunocompromised hosts such as those on chemotherapy, those with cystic fibrosis, burns and wounds. Biofilms are antibiotic resistant. Key risks also include urinary catheters and central venous lines etc… I would not worry too much about MacConkey agar as lots of things grow on it. It is a culture medium designed to grow gram negative bacteria, so organisms ranging from E. coli, Klebsiella and Salmonella, to Proteus, Shigella and Enterobacter will all grow on it. This just rules out Streptococcus pneumoniae.
Pseudomonas is also a special little organism which is not sensitive to many antibiotics originally used for gram negatives. You need to prescribe a drug here which has specific anti-pseudomonal activity such as ceftazidime (a third generation cephalosporin), tazocin, ciprofloxacin, imipenem or gentamicin (usually used with one of the others). This organism acquires resistance genes very quickly so two antibiotics are given. It is, as mentioned, inherently resistant to many drug classes. Some are even resistant to all antibiotics.

158
Q

A 15-year-old girl develops high fever with rigors. O/E she has blanching erythematous rash, ‘strawberry’ tongue, and cervical lymphadenopathy.

What is the diagnosis?
A.	Leptospira interrogans
B.	Streptococcus pneumoniae
C.	Streptococcus viridans
D.	Mycoplasma pneumoniae
E.	Legionella pneumoniae
F.	Streptococcus pyogenes
G.	Shigella flexineri
H.	Staphylococcus aureus
I.	Salmonella enteritidis
J.	Campylobacter jejuni
K.	Pseudomonas aeruginosa
A

F. Streptococcus pyogenes

The ‘strawberry’ tongue, or a red swollen tongue, is a sign of Scarlet fever (along with Kawasaki disease and toxic shock syndrome which is caused by bacteria such as staphylococcus aureus). Scarlet fever is caused by an exotoxin released by Streptococcus pyogenes. The history is characteristically a child <10 years old, usually in the autumn, winter seasons, maybe early spring, presenting with sore throat, fever, malaise and GI upset. Examination may reveal a fever, pharyngeal redness with possible exudate, a generalised sandpaper-like erythematous rash, linear petechial streaks (pastia lines) in skin folds and tender cervical lymphadenopathy. Of course, also, the red swollen tongue. You can request a pharyngeal swab but the diagnosis is generally clinical. You can also expect ASO titres, if done, to be positive.

159
Q

A 30-year-old man presents jaundiced. He tells you he has recently completed a triathlon. He has been suffering from flu-like symptoms for the last week, he complains of severe headache, myalgia, anorexia. O/E he has tender hepatosplenomegaly and a rash over the lower limbs.

What is the diagnosis?
A.	Leptospira interrogans
B.	Streptococcus pneumoniae
C.	Streptococcus viridans
D.	Mycoplasma pneumoniae
E.	Legionella pneumoniae
F.	Streptococcus pyogenes
G.	Shigella flexineri
H.	Staphylococcus aureus
I.	Salmonella enteritidis
J.	Campylobacter jejuni
K.	Pseudomonas aeruginosa
A

A. Leptospira interrogans

Leptospirosis is a zoonosis, which is transmitted by contact with urine of infected animals (also possible sources include blood and fluids). The history may reveal someone swimming in rat infested canal water, for instance, or in this case, a triathlon – which involves some swimming in perhaps not too clean water. Outbreaks of this are associated with flooding and natural disasters, as can be expected. Affected patients can present with an extensive spectrum of clinical manifestations ranging from subclinical illness in 90% to renal and hepatic failure and pulmonary haemorrhage. The important factor in diagnosis is a high index of suspicion based on epidemiological exposure. There is an acute phase with fever, headaches, myalgia and then an immune phase with additional pulmonary symptoms and potential organ damage (leading to the symptoms seen here such as jaundice). The rash is maculopapular and non-pruritic, lasting 1 or 2 days, present during the acute phase. It is rarely seen. Treatment is with benzylpenicillin or amoxicillion and/or doxycycline and supportive care. Those with severe disease carry a poor prognosis.

160
Q

A 42-year-old presents with crampy abdominal pain, nausea and profuse diarrhoea a couple of days after eating grilled chicken in a newly opened restaurant. A gram stain of the faeces shows Gram-negative, motile, spiral-shaped rods.

What is the diagnosis?
A.	Leptospira interrogans
B.	Streptococcus pneumoniae
C.	Streptococcus viridans
D.	Mycoplasma pneumoniae
E.	Legionella pneumoniae
F.	Streptococcus pyogenes
G.	Shigella flexineri
H.	Staphylococcus aureus
I.	Salmonella enteritidis
J.	Campylobacter jejuni
K.	Pseudomonas aeruginosa
A

J. Camplyobacter jejuni

This is what we colloquially refer to as ‘food poisoning’ – infective gastroenteritis. The most common bacterial cause in the UK is Campylobactor jejuni. This organism is also a major cause of traveller’s diarrhoea. The second most common cause of acute enteritis is an organism known as Campylobacter coli. There is also Campylobacter fetus which is a rare cause of extra-intestinal infection mainly in immunocompromised patients. Campylobacters are small, gram negative rods with a curved spiral shape. 11 Campylobacters are pathogenic to humans. Most Campylobacter jejuni infections come from infected poultry. The diarrhoea is usually self limiting and resolves in a week – bacteraemia is very rare. Fluid and electrolyte replacement is essential. Only a small number of patients will benefit from antimicrobials. Note that C. jejuni is a commonly identified organism in the aetiology of Guillain-Barre syndrome.

161
Q

A 27-year-old with severe headaches and hypertension is found to have phaeochromocytoma. Further investigations reveal he has hypercalcaemia secondary to hyperparathyroidism. The alarmed clinician orders a thyroid biopsy which confirms his suspicions.

What is the diagnosis?
A.	DiGeorge's syndrome
B.	MEN II
C.	Pituitary apoplexy
D.	MEN I
E.	Kallmann's syndrome
F.	Cushing's disease
G.	Sheehan's syndrome
H.	Pseudo-Cushing's syndrome
I.	 Simmond’s disease
J.	Nelson's syndrome
A

B. MEN II

MEN (Multiple Endocrine Neoplasia) syndromes are hereditary autosomal dominant tumour syndromes with distinct patterns of organ involvement. At this stage, the purely simplified patterns (classification is actually more complicated) you need to be familiar with are: MEN1 consists of parathyroid adenomas, pancreatic tumours and pituitary adenomas. MEN2A consists of parathyroid, medullary thyroid cancer and phaeochromocytoma. And MEN2B or 3 consists of what is seen in 2A plus the addition of a marfanoid phenotype and ganglioneuromas (intestinal and visceral).
Prophylactic thyroidectomy in childhood is indicated in MEN2. Medical management is aimed at controlling hormone hypersecretion. Surgery is often done on tumours. If you’re really interest, you can look MEN syndromes up in more detail. MEN1 is typically caused by mutations in the MEN1 gene whereas MEN2 is typically caused by mutations in the RET proto-oncogene.

162
Q

A 60-year-old gentleman noticed increasing pigmentation of his skin in the past 5 years. He presents to you with progressive headaches and double vision. He says he was well previously. On further questioning he recalls a surgery to remove his adrenal glands around 30 years ago. MRI demonstrates pituitary tumour.

What is the diagnosis?
A.	DiGeorge's syndrome
B.	MEN II
C.	Pituitary apoplexy
D.	MEN I
E.	Kallmann's syndrome
F.	Cushing's disease
G.	Sheehan's syndrome
H.	Pseudo-Cushing's syndrome
I.	 Simmond’s disease
J.	Nelson's syndrome
A

J. Nelson’s syndrome

Nelson’s syndrome is the enlargement of a pituitary adenoma which occurs after bilateral adrenalectomy. Once you know this fact, the diagnosis is clear. Bilateral adrenalectomy is an operation which can be done for Cushing’s syndrome in order to completely eliminate the production of cortisol. However, this removes cortisol’s negative feedback response which allows any pre-existing pituitary adenoma to grow without negative feedback. As a result, this rapid enlargement of the pituitary adenoma has caused this man’s symptoms of increased pigmentation due to raised MSH (a by product of POMC cleavage to give ACTH), headaches and visual disturbances (due to the space-occupying lesion). This is now rare as the operation is now only used in extreme cases. Sometimes pituitary sugery will be performed.

163
Q

A mother of a 6-month-old boy presents with fatigue and cold intolerance. She said she has not had her periods back and has been unable to breastfeed her baby. She also has reduced libido, depression and weight loss. From the hospital notes you learn that her labour was complicated by significant post-partum haemorrhage. Examination reveals slow relaxing reflexes.

What is the diagnosis?
A.	DiGeorge's syndrome
B.	MEN II
C.	Pituitary apoplexy
D.	MEN I
E.	Kallmann's syndrome
F.	Cushing's disease
G.	Sheehan's syndrome
H.	Pseudo-Cushing's syndrome
I.	 Simmond’s disease
J.	Nelson's syndrome
A

G. Sheehan’s syndrome

Sheehan’s syndrome is panhypopituitarism which occurs due to necrosis of the pituitary gland following post-partum haemorrhage and hypovolaemia. Blood loss leads to a vasoconstrictor spasm of the hypophysial arteries leading to ischaemia and necrosis of the pituitary. This is a rare complication of pregnancy. Symptoms include those seen and are due to deficiency of a host of hormones. Treatment is aimed at replacing the missing hormones (hydrocortisone, thyroxine etc).

164
Q

A 55-year-old lady has centripetal obesity, plethoric moon-shaped face, proximal muscle wasting. Her daughter says she is very lethargic and seems depressed lately. Blood show macrocytosis and raised gamma-GT. The serum cortisol is elevated and fails to suppress on low-dose-dexamethasone test. MRI of the head and CT of the body are normal

What is the diagnosis?
A.	DiGeorge's syndrome
B.	MEN II
C.	Pituitary apoplexy
D.	MEN I
E.	Kallmann's syndrome
F.	Cushing's disease
G.	Sheehan's syndrome
H.	Pseudo-Cushing's syndrome
I.	 Simmond’s disease
J.	Nelson's syndrome
A

H. Pseudo-Cushing’s syndrome

Pseudo-Cushing’s syndrome is where a patient has all of the signs and symptoms and even abnormal hormone levels seen in Cushing’s syndrome however there is no problem to be found with the hypothalamo-pituitary-adrenal axis. It is hence idiopathic. Whole body CT here is normal so rules out causes from organs such as the lungs (ectopic ACTH), adrenals and MRI has also ruled out a pituitary cause.

165
Q

A 16-year-old girl was born colour-blinded and hyposmic. Now she presents with primary amennorhoea and delayed puberty. She has low levels of LH, FSH and oestriadol.

What is the diagnosis?
A.	DiGeorge's syndrome
B.	MEN II
C.	Pituitary apoplexy
D.	MEN I
E.	Kallmann's syndrome
F.	Cushing's disease
G.	Sheehan's syndrome
H.	Pseudo-Cushing's syndrome
I.	 Simmond’s disease
J.	Nelson's syndrome
A

E. Kallmann’s syndrome

Kallmann’s syndrome (hypogonadotrophic hypogonadism) is a cause of primary amenorrhoea. The history tends to be of delayed development of secondary sexual characterisitcs with anosmia. There is either a missing olfactory bulb or one which is not fully developed (this may be seen on MRI) so there may be a lack of a sense of smell or a sense of smell which is severely reduced (hyposmia seen here). It may be diagnostically challenging as it is difficult to distinguish pathological developmental delay from constitutional delay. Normal but pre-pubertal external and internal genitalia are seen, and normal final adult height if treated. You would expect serum FSH to be low (though FSH assays have very very very wide ranges) and LH if done. Low oestradiol can also be expected.

166
Q

A previously well, 29-year-old man with a 2-week history of flu-like illness complains of increasing weakness and numbness in his lower limbs. O/E he has grade 4/5 distal weakness, diminished reflexes, plantars down-going. Upper limbs and cranial nerves unaffected.

What is the diagnosis?
A.	Osler-Weber-Rendu Syndrome
B.	Tietze's Syndrome
C.	Bornholm's disease
D.	Brown-Sequard Syndrome
E.	Felty's Syndrome
F.	Alport's syndrome
G.	Meig's syndrome
H.	Gullian-Barre Syndrome
I.	Milroy's disease
J.	Peutz-Jegher's Syndrome
A

H. Guillan-Barre syndrome

This previous history of influenza-like illness weeks before the onset of neurological symptoms indicates Guillain-Barre syndrome which is a demyelinating polyneuropathy. Classic neurology is a progressive symmetrical muscle weakness affecting lower extremities before upper extremities, and proximal muscles before distal muscles, accompanied by paraesthesias in the hands and feet which often precedes onset of weakness. The paralysis is typically flaccid with areflexia and progresses acutely over days, with an ensuing plateau phase followed by recovery. Two thirds of patients have a history of either prior influenza-like illness or gastroenteritis. Weak risks include immunisation, cancer and lymphoma, older age, HIV infection and male gender. Up to 30% will develop respiratory muscle weakness requiring ventilation so spirometry should be carried out at 6 hour intervals initially (and may show reduced vital capacity). AST and ALT may be elevated though the cause is unclear. LP is useful and the classic finding is of elevated CSF protein with normal cell count (known as albuminocytological dissociation). Treatment is with supportive and disease modifying treatment (plasma exchange or high dose Ig).
GBS is classified by symptoms and variants exist such as Miller-Fisher syndrome, which occasionally crop up in EMQ books but will probably never crop up in clinical practice.

167
Q

Previously well, 17-year-old girl presents with 3-months history of swelling of her right leg. There is no pain or other associated features. O/E you confirm pitting oedema of the shin, blood tests normal. Her mother also suffers from swollen legs.

What is the diagnosis?
A.	Osler-Weber-Rendu Syndrome
B.	Tietze's Syndrome
C.	Bornholm's disease
D.	Brown-Sequard Syndrome
E.	Felty's Syndrome
F.	Alport's syndrome
G.	Meig's syndrome
H.	Gullian-Barre Syndrome
I.	Milroy's disease
J.	Peutz-Jegher's Syndrome
A

I. Milroy’s syndrome

Milroy’s is primary lymphoedema with a familial autosomal dominant pattern of inheritance. It often presents in infancy. There is unfortunately no cure and treatment involves compression, with surgery reserved for those refractive to conservative measures and/or with major morbidity.

168
Q

A 22-year-old lady presents with discomfort in her chest, especially while breathing. There is localised area of tenderness around the 2nd costal cartilage on the right. O/E you can fell a firm, tender lump in this area. There is no history of trauma.

What is the diagnosis?
A.	Osler-Weber-Rendu Syndrome
B.	Tietze's Syndrome
C.	Bornholm's disease
D.	Brown-Sequard Syndrome
E.	Felty's Syndrome
F.	Alport's syndrome
G.	Meig's syndrome
H.	Gullian-Barre Syndrome
I.	Milroy's disease
J.	Peutz-Jegher's Syndrome
A

B. Tietze’s syndrome

This is costochondritis, or Tietze’s syndrome (which describes constochondritis accompanied by chest wall swelling), which presents with insidious onset of anterior chest wall pain which is made worse by certain movements of the chest and deep inspiration. The key sign here is that there is pain when palpating the costochondral joints, particularly the 2nd to the 5th and the diagnosis is clinical. Tests are done to exclude other diagnoses here such as breast pathology. First line treatment is with NSAIDs. Oral NSAIDs are preferred in a primary care setting and a beneficial response confirms the diagnosis. If NSAIDs or local corticosteroid injection (usually performed by a specialist) fail to make the symptoms better than you should seek further investigations and consider a wider differential diagnosis which include conditions like pleuritis, ACS, PE, rib fracture and GORD.

169
Q

A 41-year-old lady with long-standing RA presents to his GP with recurrent chest infections. She also lost 4kg in past 3 months. O/E there is splenomegaly. FBC confirms pancytopenia.

What is the diagnosis?
A.	Osler-Weber-Rendu Syndrome
B.	Tietze's Syndrome
C.	Bornholm's disease
D.	Brown-Sequard Syndrome
E.	Felty's Syndrome
F.	Alport's syndrome
G.	Meig's syndrome
H.	Gullian-Barre Syndrome
I.	Milroy's disease
J.	Peutz-Jegher's Syndrome
A

E. Felty’s syndrome

Felty’s syndrome is a rare extra-articular manifestation of rhematoid arthritis characterised by persistent and idiopathic neutropenia and in some cases splenomegaly. It occurs in <1% of RA patients, often in the 5th to 7th decade of life, typically in white patients with long-standing seropositive and erosive disease. Treatment aims at trying to raise the neutrophil count and prevent recurrent infections, while controlling systemic RA. The cause of the neutropenia is multifactorial and due to an imbalance between production in BM and descruction in peripheral blood. Other causes of neutropenia need to be ruled out first so investigations like peripheral blood film are needed.

170
Q

A 27-year-old gentleman has a long history of frequent nose-bleeds. O/E of his nose you notice red spots on muous membrane. on his face and skin you can also notice some red spots.

What is the diagnosis?
A.	Osler-Weber-Rendu Syndrome
B.	Tietze's Syndrome
C.	Bornholm's disease
D.	Brown-Sequard Syndrome
E.	Felty's Syndrome
F.	Alport's syndrome
G.	Meig's syndrome
H.	Gullian-Barre Syndrome
I.	Milroy's disease
J.	Peutz-Jegher's Syndrome
A

A. Osler-Weber-Rendu syndrome

This man has the facial telangiectasia of OWR, also called hereditary haemorrhagic telangiectasia. This causes abnormal blod vessels pretty much everywhere which are prone to bleed. It is an autosomal dominant condition so a positive FH can often be found.

171
Q

A 22-year-old recently started the OCP. She now suffers a pounding left sided headache every 2 weeks or so, which lasts several hours. She feels ‘strange’ about 2 hours before the headache, during which she experiences nausea, vomitting and visual disturbance.

What is the diagnosis?
A.	Trigeminal neuralgia
B.	Cervical spondylosis
C.	Space-occupying lesion
D.	Tension headache
E.	Subarachnoid haemorrhage
F.	Meningitis
G.	Temporal arteritis
H.	Migraine
I.	Cluster headache
A

H. Migraine

Migraine is a chronic condition, with genetic determinants, which usually presents in early to mid life. The typical migraine aura this patient describes (which can be visual, sensory or speech symptoms) which can occur during or before the headache, is pathognomic, but is not seen in the majority of patients. The aura can be positive phenomena (for example a patient seeing flashing lights) or negative phenomena (for example visual loss). Nausea, photophobia and disability (the headache gets in the way with the patient’s ability to function) accompanying a headache also suggest a migraine diagnosis. The headache of a migraine tends to be prolonged if untreated, and tends to be unilateral and pounding (but does not have to be). The OCP may be a trigger here. Tests aim to rule out other differentials, although if the history is compatible and neurological examination is unremarkable, further testing is not needed.

172
Q

Previously well, 27-year-old woman developed ‘the worst headache ever’ around 3 hours ago. It was sudden and felt like a clap of thunder. He vomitted twice since and has photophobia. There is no family history of headaches, her mother and grandmother died of kidney disease.

What is the diagnosis?
A.	Trigeminal neuralgia
B.	Cervical spondylosis
C.	Space-occupying lesion
D.	Tension headache
E.	Subarachnoid haemorrhage
F.	Meningitis
G.	Temporal arteritis
H.	Migraine
I.	Cluster headache
A

E. Subarachnoid haemorrhage

SAH (bleeding into the subarachnoid space) presents with sudden severe headache patients will often describe as the worst headache of their life, and can often be so bad that they feel like they’ve been kicked in the back of the back. Half of all patients lose consciousness and eye pain with exposure to light can also be seen. Altered mental status is common. SAH occurs most commonly in the 50-55 age group and affects women and black people more than men and white people. The most common cause of non-traumatic SAH is an aneurysm which ruptures. Conditions which predispose to aneurysm formation and SAH include adult PKD, Marfan’s, NF1 and Ehlers-Danlos. Cerebral aneurysms arise around the circle of Willis. The family history of kidney disease here points to adult PKD. A CT scan is indicated, and if unrevealing, this should be followed by an LP. Cerebral angiography can confirm the presence of aneurysms. The patient should be stabilised and this followed by surgical clipping or endovascular coil embolisation, the choice is subject to much current controversy sparked by relatively recent research. Complications can commonly occur and include rebleeding, hydrocephalus and vasospasm.

173
Q

A 47-year-old smoker is suffering from early morning headaches, which are generalised and dull. Coughing and laughing makes it worse. O/E you notice papilloedema

What is the diagnosis?
A.	Trigeminal neuralgia
B.	Cervical spondylosis
C.	Space-occupying lesion
D.	Tension headache
E.	Subarachnoid haemorrhage
F.	Meningitis
G.	Temporal arteritis
H.	Migraine
I.	Cluster headache
A

C. Space-occupying lesion

Raised ICP classically presents with symptoms which are worse in the morning. The headache can either awake the patient from sleep or is present on waking up and decreases after being awake for several hours. It is also made worse on exertion/Valsalva such as the coughing and bending over here. This patient will require a CT head to try to find the cause which could be a space-occupying lesion. This is also the cause of the papilloedema seen on fundoscopy. Raised ICP can also manifest with a CNVI palsy, which is a false localising sign.

174
Q

A 39-year-old man gets intense, stabbing pain in his forehead and under left eye. Pain lasts only couple of seconds, but recurs several times during the day (especially when shaving or chewing).

What is the diagnosis?
A.	Trigeminal neuralgia
B.	Cervical spondylosis
C.	Space-occupying lesion
D.	Tension headache
E.	Subarachnoid haemorrhage
F.	Meningitis
G.	Temporal arteritis
H.	Migraine
I.	Cluster headache
A

A. Trigeminal neuralgia

Trigeminal neuralgia occurs as episodes of severe unilateral pain in the distribution of CNV, usually lasting seconds, with no pain occuring between these episodes. Examination is often unremarkable. The pain is described as sharp, intense, stabbing or burning. It can be triggered commonly by actions such as eating, tooth brushing, cold and touch. Shaving and eating seem to be common in EMQs. Most people are asymptomatic between attacks although the severity of the pain makes these patients live in constant fear. TN is more common in MS and incidence increases with age. Post-herpetic TN is also possible. The mainstay of treatment is medical, with antiepileptics such as carbamazepine (which is the only medicine which is proven in RCTs and is therefore typically first line). If medical treatment fails, surgical options do exist such as microvascular decompression.

175
Q

A 31-year-old man presents with 3-month history of several headaches. The headache comes at night around 1am everynight, and lasts around half an hour. It is severe, left-sided, with ipsilateral watering and reddening of the eye.

What is the diagnosis?
A.	Trigeminal neuralgia
B.	Cervical spondylosis
C.	Space-occupying lesion
D.	Tension headache
E.	Subarachnoid haemorrhage
F.	Meningitis
G.	Temporal arteritis
H.	Migraine
I.	Cluster headache
A

I. Cluster headache

Cluster headache is characterised by attacks of severe pain localised to the unilateral orbital, supraorbital and/or temporal areas which lasts from 15 minutes to 3 hours, and occurs with a frequency ranging from once every other day to 8 times a day. These attacks can occur at the same time period of many weeks (known as the cluster period) accompanied by ipsilateral autonomic signs. The cause is hypothalamic activation with secondary trigeminal and autonomic activation (for instance, lacrimation, rhinorrhoea, nasal congestion, conjunctival injection and partial Horner’s i.e. ptosis and miosis). Cluster period attacks can be triggered by things like alcohol. Greater occipital nerve blockade often provides immediate relief until preventative medications take effect.

176
Q

A 29-year-old with ulcerative colitis developed an ulcer on his right shin. He is otherwise well, with ABPI of 1.0.

What is the diagnosis?
A.	Neuropathic ulcer
B.	Venous Ulcer
C.	Gangrene
D.	Arterial Ulcer
E.	Sickle cell anaemia
F.	Lymphoedema
G.	Pyoderma gangerosum
H.	Scleroderma
I.	Squamous cell carcinoma
A

G. Pyoderma gangrenosum

This is pyoderma gangrenosum, which presents with multiple lesions, most commonly affecting the lower extremity and is linked to UC although it can be seen less commonly in Crohn’s, and is also seen in conditions such as RA and the myeloid dyscrasias. These lesions start as tender papules or vesicles which develop into painful ulcers with a dusky purple edge and surrounding induration and erythema. The base may contain granulation tissue and lesions heal with atrophic scars.

177
Q

A 62-year-old diabetic lady presents with recurrent ulceration of the gaiter area of the left leg. The ulcer is well circumscribed, irregular in shape and of partial thickness. There is a brown discolouration and ‘eczema’ over both calves.

What is the diagnosis?
A.	Neuropathic ulcer
B.	Venous Ulcer
C.	Gangrene
D.	Arterial Ulcer
E.	Sickle cell anaemia
F.	Lymphoedema
G.	Pyoderma gangerosum
H.	Scleroderma
I.	Squamous cell carcinoma
A

B. Venous ulcer

Venous ulcers occur on a background of deep venous insufficiency. There is oedema and a brown skin discolouration due to leaching of pigments and haemosiderin deposition. In addition there may be lipodermatosclerosis and an inflammatory response, which is seen as an eczema-like thickening and hardening of the skin. The skin can also be drawn tightly around the ankle. Ulceration usually follows trauma and is usually on the medial gaiter region. The base has granulation tissue and is sloughy in nature and there is a sloping edge to the ulcer. The shape is often irregular. Look up some photos to help you remember. Once significant arterial disease is excluded (ulcers can have mixed components), the mainstay of treatment is with compression bandaging, appropriate dressings and treatment of any infection with antibiotics. Maggots can also be used and varicose veins should be treated where possible to reduce recurrence. If the ulcer is not healing, a biopsy should be considered (Marjolin’s ulcer).

178
Q

A 24-year-old lady from sub-Saharan Africa presents with a painful ulcer over the anterior shin of the left leg. ABPI is normal.

What is the diagnosis?
A.	Neuropathic ulcer
B.	Venous Ulcer
C.	Gangrene
D.	Arterial Ulcer
E.	Sickle cell anaemia
F.	Lymphoedema
G.	Pyoderma gangerosum
H.	Scleroderma
I.	Squamous cell carcinoma
A

E. Sickle cell anaemia

Adults with sickle cell anaemia may present with leg ulcers although it has to be said that it is very unusual for a person with sickle cell disease to reach adulthood without being aware of their diagnosis. The adult patient could present with unexplained haemolysis, possible intermittent episodes of pain due to vaso-occlusive crises, avascular necrosis and retinal haemorrhage too. The doppler derived ankle-brachial pressure index is normal here which points away from peripheral arterial disease, which in this relatively young person is unlikely in any case. The typical constellation of findings of venous ulcers are not seen (oedema, lipodermatosclerosis).
In real medicine, as it is rare for sickle cell anaemia to present this late, I would consider other possibilities like pyoderma gangrenosum (which is arguably more likely in this case, given the location of the ulcer and age of the patient, so if this was your answer then you can give yourself a ‘real life medicine’ mark), but this is an EMQ (key point!) and the fact this lady is from the sub-Saharan Africa is a key fact. From an epidemiological perspective, the prevalence is 10-30% in sub-Saharan Africa. Between 25 and 30% of newborns in western Africa are carriers of sickle cell trait. This patient here will need to have their peripheral blood film reviewed following by haemoglobin electrophoresis and HPLC. Sickle solubility is a rapid test which can also be done but will not differentiate sickle trait from sickle disease.
At the end of the day, remember that to do well in an EMQ, you need to think like an EMQ.

179
Q

A 41-year-old man has a slowly growing circular ulcer over his right shin for a year now. The ulcers edges are raised and there is contact bleeding along one edge.

What is the diagnosis?
A.	Neuropathic ulcer
B.	Venous Ulcer
C.	Gangrene
D.	Arterial Ulcer
E.	Sickle cell anaemia
F.	Lymphoedema
G.	Pyoderma gangerosum
H.	Scleroderma
I.	Squamous cell carcinoma
A

I. Squamous cell carcinoma

SCC is a malignant tumour of keratinocytes arising in the epidermis of the skin. Risk factors include UV exposure, older age, immunosuppression, fair skin, HPV and exposure to ionising radiation. The skin changes here are characteristic. Most frequently however, SCC often presents with multiple actinic keratoses, with a change in the lesion such as bleeding, induration, change in size and shape or inflammation. Aks and SCCs lie on a spectrum so it may be difficult to clinically differentiate the two. Skin biopsy will confirm the diagnosis. This can either be a shave biopsy, punch biopsy, incisional biopsy or excisional biopsy depending on the size and location.

180
Q

A 65-year-old obese smoker is brought to the GP by his wife because of large, deep ulcer over the lateral malleous of his right leg. Neurological examination of the lower limbs is normal. On further questioning patient complains of calves pain on walking and chest pain on exertion. ABPI is 0.6.

What is the diagnosis?
A.	Neuropathic ulcer
B.	Venous Ulcer
C.	Gangrene
D.	Arterial Ulcer
E.	Sickle cell anaemia
F.	Lymphoedema
G.	Pyoderma gangerosum
H.	Scleroderma
I.	Squamous cell carcinoma
A

D. Arterial ulcer

Arterial ulcers are deep and painful with a well defined edge, usually found on the shin or foot. They often have a pale base and a punched out appearance. The low ankle brachial pressure index is indicative too. There may be local changes such as cold peripheries, loss of hair, dusky cyanosis and toenail dystrophy. The surrounding skin is often white and shiny. It is typically most painful in bed and the pain is sometimes relieved by having the legs dependent. On examination, peripheral pulses may be absent or reduced. An angiogram with contrast will define the lesion and determine whether it can be improved by surgical intervention. Pain often increases when your legs are at rest and elevated. They can occur between the webs of toes so it is important to always check these in your peripheral vascular examination.

181
Q

A 17-year-old girl recently started OCP. She presents with abdominal pain and, vomiting and tachycardia. She has developed left foot drop.

What is the diagnosis?
A.	HIV
B.	Lyme disease
C.	Diabetic amyotrophy
D.	Porphyria
E.	Beri-beri
F.	Charcot-Marie-Tooth Syndrome
G.	Alcohol excess
H.	Vitamin B12 deficiency
I.	Lung carcinoma
A

D. Porphyria

There are many types of porphyria. This patient has acute intermittent porphyria, which is characterised by symptoms like the ones this patient describes – abdominal pain, peripheral motor neuropathy, mental symptoms like confusion. These symptoms, certainly in EMQs, can be trigged by the use of certain drugs which are known to provoke AIP attacks. In reality, the list of drugs is pretty vast and include most CYP450 inducers, but in EMQs, alcohol and the OCP are common. It is worth noting that alcohol also induces an enzyme called delta-aminolevulinic acid synthase, which can exacerbate AIP. It is probably not worth learning the pathways unless you plan on sitting USMLE. AIP is a genetic disorder where there is a partial deficiency of PBGD (the third enzyme in the haem biosynthetic pathway). Treatment of acute attacks involves IV haem arginate with adjunctive dextrose IV. The pain is thought to be neuropathic in origin. The patient may complain of red/browny urine due to increased urinary excretion of intermediates in the haem pathway.

182
Q

A disabled 60 year-old lady lives on her own and rarely leaves her flat. She called an ambulance after becoming SOB while walking upstairs to her bedroom. O/E she has peripheral oedema up to the groin. She has painful feet and calves with mixed motor and sensory neuropathy. She is tachycardic, hypotensive, with raised JVP.

What is the diagnosis?
A.	HIV
B.	Lyme disease
C.	Diabetic amyotrophy
D.	Porphyria
E.	Beri-beri
F.	Charcot-Marie-Tooth Syndrome
G.	Alcohol excess
H.	Vitamin B12 deficiency
I.	Lung carcinoma
A

E. Beri-beri

Beriberi is vitamin B1 (thaimine) deficiency. Deficiency is the cause of several clinical syndromes including wet and dry beriberi and Wenicke’s encephalopathy. The presentation depends on the chronicity of B1 deficiency. Dry beriberi is a distal peripheral polyneuropathy characterised by parasthesia, reduced knee jerks and other tendon reflexes, and progressive severe weakness with muscle wasting secondary to chronic deficiency. Wet beriberi, which this person also has is secondary to either acute or chronic deficiency and is characterised by high-output heart failure with peripheral vasodilation, peripheral oedema and orthopnoea or low-output heart failure with lactic acidosis and peripheral cyanosis. The latter is also referred to as Shosin beriberi.
Remember that Wernicke’s is an acute neuropsychiatric syndrome which classically presents with the triad of acute confusion, ataxia and ocular abnormalities (such as nystagmus and strabismus) secondary to acute B1 deficiency.

183
Q

A known diabetic with peripheral sensory neuropathy is complaining of pain and weakness in his left tight. He can hardly walk upstairs.

What is the diagnosis?
A.	HIV
B.	Lyme disease
C.	Diabetic amyotrophy
D.	Porphyria
E.	Beri-beri
F.	Charcot-Marie-Tooth Syndrome
G.	Alcohol excess
H.	Vitamin B12 deficiency
I.	Lung carcinoma
A

C. Diabetic amyotrophy

Diabetic amyotrophy, more common in T2DM, is an uncommon peripheral diabetic neuropathic complaint. It presents with severe muscle weakness and pain with proximal thigh muscle atrophy. Weak knee flexion and quadriceps wasting is typical of diabetic amyotrophy.

184
Q

A 37-year-old man presents with 4-week history of progressive numbness and pain in his hands. O/E you notice multiple violaceous patches and evidence of peripheral sensory neuropathy.

What is the diagnosis?
A.	HIV
B.	Lyme disease
C.	Diabetic amyotrophy
D.	Porphyria
E.	Beri-beri
F.	Charcot-Marie-Tooth Syndrome
G.	Alcohol excess
H.	Vitamin B12 deficiency
I.	Lung carcinoma
A

A. HIV

HIV is a retrovirus and there are two types, HIV 1 which is the main virus responsible and HIV 2 which is restricted to parts of West Africa. Strong risk factors include needle sharing with IVDU, unprotected receptive intercourse, needle stick injury and high maternal viral load (mother to child). Kaposi’s sarcoma may present as a pink or violaceous patch on the skin or in the mouth and it is an AIDS-defining condition. Peripheral neuropathy is common and may be related either to HIV or some other medicine or toxin (some HAART can cause PN).
There are WHO (stage 1-4) and CDC criteria used in clinical staging. This patient needs to have a CD4 count, HBV and HCV screen, VDRL (syphilis), tuberculin skin test (TB) and CXR. HIV viral load will also be assessed. Prophylaxis and immunisations should be considered against infections such as hepatitis, influenza, PCP and TB. When to initiate HAART depends on the clinical stage, CD4 and co-morbidities. This patient will need to be started on HAART if he has not already. Classes of antiretrovirals include NRTIs, NNRTIs, protease inhibitors, fusion inhibitors and integrase inhibitors.

185
Q

An elderly gentleman is lethargic and complains of numbness in his feet. Blood test reveals a Hb 6.0g/dl and MCV 115fl. O/E you observe a peripheral sensory neuropathy in his lower limb. You order a blood film.

What is the diagnosis?
A.	HIV
B.	Lyme disease
C.	Diabetic amyotrophy
D.	Porphyria
E.	Beri-beri
F.	Charcot-Marie-Tooth Syndrome
G.	Alcohol excess
H.	Vitamin B12 deficiency
I.	Lung carcinoma
A

H. Vitamin B12 deficiency

This is a macrocytic anaemia with elevated MCV. The classical peripheral blood smear would show megalocytes (RBC precursors whose numbers increase) and hypersegmented polymorphonuclear cells although the film may be normal in early deficiency. You would also want to order a serum B12 level in this patient and would expect the corrected reticulocyte index to be low due to decreased production (opposed to high in haemolytic anaemia due to increased production). This peripheral sensory neuropathy would likely be a loss of proprioception and vibration indicating degeneration of the dorsal column-medial lemniscus pathway. The patient may also have a positive Romberg as a result. Parasthesias are also common in B12 deficiency indicating peripheral neuropathy. Pallor and signs of frank anaemia are late signs. It is important to remember that hepatic B12 stores last for many years so B12 deficiency depends on chronic deficiency over a long period of time. In general, deficiency is caused by decreased intake, decreased gastric breakdown from food or malabsorption from the GIT. Serum B12 is a useful investigation to initially conduct and will guide further investigations and treatment. You should consider an underlying malabsorptive process such as coeliac or Crohn’s as a possible cause.

186
Q

A 31-year-old stuntman sustains a displaced spinal fracture with cord transaction at T12/L1 while performing a new trick. He also broken his left humerus and radius. He is stable but his BP remains 100/60 despite fluid resuscitation and his pulse is 55bpm.

What is the diagnosis?
A.	Urinary retention
B.	Haemorrhagic shock
C.	Neurogenic shock
D.	Acute renal failure
E.	Percutaneous jejunostomy
F.	Pulmonary oedema
G.	Intravenous nutrition
H.	Basal atelectasis
I.	Spinal shock
J.	Percutaneous gastrostomy
K.	Cardiogenic shock
A

C. Neurogenic shock

This is a thoracolumbar spine fracture. Neurogenic shock is not to be confused with spinal shock which is not circulatory in nature ( and is characterised by hypotonia or flaccidity that resolves within 24 hours). Neurogenic shock is a form of distributive shock due to spine or braintem injury and there is resulting failure of vasoregulation. As a result there is a fall in systemic vascular resistance with vasodilation, leading to low BP as blood pools in the extremeties where sympathetic tone is low. This is occasionally associated with bradycardia which is due to autonomic disruption.

187
Q

A 72-year-old man underwent hip replacement surgery. 8 hours post-op his urine output has been 30ml, 20ml and 5ml over last 3hours. He is now anuric and seems lethargic. Other vital signs remain normal.

What is the diagnosis?
A.	Urinary retention
B.	Haemorrhagic shock
C.	Neurogenic shock
D.	Acute renal failure
E.	Percutaneous jejunostomy
F.	Pulmonary oedema
G.	Intravenous nutrition
H.	Basal atelectasis
I.	Spinal shock
J.	Percutaneous gastrostomy
K.	Cardiogenic shock
A

A. Urinary retention

This is clearly urinary retention. Urine output is gradually deteriorating in a step-wise manner and he is now anuric

188
Q

A 79-year-old frail lady underwent hip replacement surgery. Post-op she has been prescribed 2L saline 8-hourly. On the second day he becomes short of breath, tachypnoeic. O/E bibasal fine crepitations. ABG confirms Type I respiratory failure.

What is the diagnosis?
A.	Urinary retention
B.	Haemorrhagic shock
C.	Neurogenic shock
D.	Acute renal failure
E.	Percutaneous jejunostomy
F.	Pulmonary oedema
G.	Intravenous nutrition
H.	Basal atelectasis
I.	Spinal shock
J.	Percutaneous gastrostomy
K.	Cardiogenic shock
A

F. Pulmonary oedema

This patient has been fluid overloaded and has pulmonary oedema, causing her dyspnoea. She will require diuretics and fluid restriction to deal with his overloaded state. The findings of bibasal fine creptitions, heard at the end of expiration, is characteristic. Infiltrates may also be seen on CXR. Type 1 respiratory failure is hypoxic respiratory failure, which occurs when PaO2 is low. Type 2 respiratory failure is known also as hypercapnic respiratory failure and occurs when there is hypoxia associated with a high PaCO2.
As a general note for your exams. Common causes of type 1 respiratory failure include pulmonary oedema, pneumonia and PE. Common causes of type 2 include COPD and respiratory muscle weakness.

189
Q

A 66-year old obese, smoker with T2DM presents drowsy to A&E. He has central abdominal pain radiating to the back that started around 5 hours before. He is tachycardic, tacypnoeic and his BP is 90/40 mmHg. Femoral pulses are faintly palpable and you can’t feel any pulses distally. His wife says his pulses are not palpable because of his peripheral vascular disease.

What is the diagnosis?
A.	Urinary retention
B.	Haemorrhagic shock
C.	Neurogenic shock
D.	Acute renal failure
E.	Percutaneous jejunostomy
F.	Pulmonary oedema
G.	Intravenous nutrition
H.	Basal atelectasis
I.	Spinal shock
J.	Percutaneous gastrostomy
K.	Cardiogenic shock
A

B. Haemorrhagic shock

This is a history of a ruptured AAA. He has key risk factors of obesity, smoking and T2DM. There is abdominal pain radiating to the back here and the low BP and compensatory tachycardia is due to the blood loss. This patient is in haemorrhagic shock. As this AAA has ruptured, this man will need urgent surgical repair, with of course standard resuscitation measures. The airway will needed to be managed with supplemental oxygen and ET intubation, a central venous catheter will need to be inserted, an arterial catheter and urinary catheter will also be needed for monitoring, and the target systolic BP is 50-70. Infusing too many gluids may increase the risk of death. The most effective form of surgical repair is an EVAR (endovascular AAA repair), anatomy permitting, otherwise traditional open repair is performed. Open repair has a mortality of 48%. Antibiotics will also be needed to cover bacteria to prevent graft infection. This will be prescribed in line with local protocols.

190
Q

Mr A.L is a 69-year-old who had oesophageal cancer underwent oesophagectomy. What kind of feed does he require?

A.	Urinary retention
B.	Haemorrhagic shock
C.	Neurogenic shock
D.	Acute renal failure
E.	Percutaneous jejunostomy
F.	Pulmonary oedema
G.	Intravenous nutrition
H.	Basal atelectasis
I.	Spinal shock
J.	Percutaneous gastrostomy
K.	Cardiogenic shock
A

E. Percutaneous jejunostomy

This patient needs nutritional support to prevent malnutrition and starvation. Enteral nutrition here is not possible as a chunk of the oesophagus has just been removed and we have to wait for an intact anastomosis. During surgery, a percutaneous jejunostomy can be placed to provide a temporary route of nutrition until oral feeding can resume. A surgeon at operation cannot place a percutaneous gastrostomy, and it is more suitable for prolonged feeding. Furthermore, in the immediate post-operative period, there may be gastric stasis so it is preferred to deliver the feed via a post-pyloric placement. If you have seen a PEG tube (percutaneous gastrostomy), you will know why it is not preferred. It is not a pleasant sight (and I don’t mean cosmetically – just have a look at the needle). Some younger people may ask for it to be converted to a button gastrostomy for cosmetic reasons. A PEG tube is placed laparoscopically.
A percutaneous jejunostomy is an alternative to parenteral (IV) nutrition in this post-operative patient. Parenteral nutrition is really a last resort and this is not an indication for it. Only when the GIT is either unavailable or function is indequate should you consider it. This can be delivered via a venflon, PICC line or centrally. There are a host of complications, both nutritionally, related to the catheter e.g. infection, thrombosis and the effect on organ systems e.g. biliary disease. Note that you should also be aware of the phenomenon known as refeeding syndrome.

191
Q

A 29-year-old man presents with frank haematuria. He also had sore throat, myalgia and fever for last 2 days. You notice his ankles are swollen. There is raised IgA titre.

What is the diagnosis?
A.	Thin membrane disease
B.	Post-streptococcal glomerulonephritis
C.	IgA nephropathy
D.	Henoch-Schonlein purpura
E.	Wegener's granulomatosis
F.	Renal stones
G.	Alport's syndrome
H.	Retroperitoneal fibrosis
I.	Goodpasture's Syndrome
A

C. IgA nephropathy

About 50% of those with IgA nephropathy will present with recurrent episodes of macroscopic haematuria after a URTI or gastroenteritis. About a third will have microscopic haematuria and mild proteinuria. Less than 10% present with nephrotic syndrome or acute rapidly progressive GN. Fever and myalgia are systemic findings. The oedema here may indicate nephrotic syndrome. Definitive diagnosis is made on renal biopsy. Light microscopy shows focal or diffuse mesangial proliferation and extracellular expansion, and IF shows diffuse mesangial IgA deposition in a granular pattern. ACE inhibitors are prescribed to reduce proteinuria, particularly in the setting of hypertension but they have not been shown to preserve renal function. Early treatment with corticosteroids has been shown to delay renal decline in those with moderate proteinuria. A urine dipstick would be the first test to order here.

192
Q

A 35-year-old with respiratory tract infection developed haemoptysis. His ankles are swollen. Blood test shows creatinine of 400 µmol/l. Antibody screen is positive for MPO-ANCA and anti-GBM antibodies.

What is the diagnosis?
A.	Thin membrane disease
B.	Post-streptococcal glomerulonephritis
C.	IgA nephropathy
D.	Henoch-Schonlein purpura
E.	Wegener's granulomatosis
F.	Renal stones
G.	Alport's syndrome
H.	Retroperitoneal fibrosis
I.	Goodpasture's Syndrome
A

I. Goodpasture’s syndrome

Goodpasture’s syndrome is defined by autoantibodies to the alpha-3 chain of type IV collagen which leads to progressive renal dysfunction. It is one of the few causes of pulmonary renal syndrome, which is characterised by pulmonary haemorrhage with rapidly progressive GN. Diagnosis is by early renal biopsy and serology. Around 30-50% of patients with anti-GBM disease will have a positive ANCA result, which changes subsequent management of this condition. Positive ANCA usually suggests a diagnosis of Wegener’s granulomatosis, Churg-Strauss or microscopic polyarteritis instead of Goodpasture’s. Note that MPO-ANCA is also known as pANCA and ANCA stands for anti-neutrophil cystoplasmic antibodies. Anti-GBM serology is a confirmatory diagnostic test in addition to renal biopsy and this can also be used to monitor response to treatment and to gauge when to stop plasmapheresis (i.e. when the antibody titre turns negative). Biopsy of the kidneys will show a crescentic GN and characteristic linear IgG staining on immunofluorescence. The renal function tests here are abnormal, which is the hallmark of Goodpasture’s syndrome.
If it is diagnosed before renal dysfunction becomes severe then aggressive treatment can lead to an excellent prognosis. Plasma exchange can be used to remove preformed antibodies.

193
Q

An 8 year old boy presents with haematuria, frothy urine and oliguria. On investigation, proteinuria, and red cell casts in the urine are confirmed. Two weeks before he has had pharyngitis. There is a raised ASOT, IgM and IgG titres.

What is the diagnosis?
A.	Thin membrane disease
B.	Post-streptococcal glomerulonephritis
C.	IgA nephropathy
D.	Henoch-Schonlein purpura
E.	Wegener's granulomatosis
F.	Renal stones
G.	Alport's syndrome
H.	Retroperitoneal fibrosis
I.	Goodpasture's Syndrome
A

B. Post-streptococcal glomerulonephritis

Nephritic syndrome is typically defined by acute kidney injury, hypertension and an active urinary sediment (RBCs and RBC casts). This is post-infectious glomerulonephritis caused by group A beta-haemolytic streptococcus with renal endothelial cell damage. Serological markers would expect to show antibodies to streptococcus and low complement and treatment here is with antibiotics. The high ASOT (antistreptolysin O antibody titres) indicates post-streptococcal GN. There may also be positive anti-Dnase and antihyaluronidase in post-streptococcal GN.
As a note, causes of nephritic syndrome (with sub-nephrotic range proteinuria) include: IgA nephropathy, post-infectious GN and rapidly progressive GN. Nephrotic syndrome causes include minimal change disease, focal and segmental glomerulosclerosis, membranous nephropathy, deposition diseases and membranoproliferative GN.

194
Q

A 10-year-old boy developed a palpable purpura on lower limbs and buttocks. He also complains of colicky abdominal pain and swelling of his ankles. He has raised serum IgA.

What is the diagnosis?
A.	Thin membrane disease
B.	Post-streptococcal glomerulonephritis
C.	IgA nephropathy
D.	Henoch-Schonlein purpura
E.	Wegener's granulomatosis
F.	Renal stones
G.	Alport's syndrome
H.	Retroperitoneal fibrosis
I.	Goodpasture's Syndrome
A

D. Henoch-Schonlein purpura

Henoch-Schonlein purpura is the most common vasculitis in childhood and in all cases there is a rash of palpable purpura which are typically non-blanching. If there is no rash, then it is not HSP. They are normally 2-10mm in diameter and are due to the extravasation of blood into the skin. They can occur anywhere on the body but are usually concentrated on the lower extremities. Half of all patients have abdominal pain and arthralgias are commonly present (found in about 80%) and often associated with oedema. The joints most often affected are the knees and ankles. About half will show signs of renal disease such as proteinuria or haematuria. Risk factors for this condition include being male, age 3-15 and history of prior UTI. Complications can occur and the most common cause of death is renal failure. While serum IgA levels may be elevated, this is not a specific test for HSP.

195
Q

A 46-year-old woman with Crohn’s disease presents with oliguria for 3 days. She has also recently suffered from lower back pain.O/E she is hypertensive, the kidneys are palpable.

What is the diagnosis?
A.	Thin membrane disease
B.	Post-streptococcal glomerulonephritis
C.	IgA nephropathy
D.	Henoch-Schonlein purpura
E.	Wegener's granulomatosis
F.	Renal stones
G.	Alport's syndrome
H.	Retroperitoneal fibrosis
I.	Goodpasture's Syndrome
A

H. Retroperitoneal fibrosis

Retroperitoneal fibrosis is a rare disorder where there is proliferation of fibrous tissue in the retroperitoneum which here has caused the ureters to become obstructed. This has led to oliguria, back pain and the palpable kidneys. The cause could be an effect of antihypertensives, or due to malignancy or some other cause. A CT scan is the test to order here and treatment depends on severity and can be either surgical or medical, the latter generally with glucocorticoids followed by DMARDs.

196
Q

Contraction of the muscles of the face when tapped gently on the cheek. Sign of hypocalcaemia.

What is the name of the sign?
A.	Cullen's sign
B.	Chvostek's sign
C.	Corrigan's sign
D.	Grey-Turner's sign
E.	Raccoon eyes
F.	Quincke's sign
G.	Murphy's sign
H.	Trousseau's sign
I.	Battle's sign
J.	Muller's sign
K.	Traube's sign
A

H. Trousseau’s sign

Trousseau’s sign is carpal spasm when a blood pressure cuff is used for several minutes. Carpopedal spasm that occurs with hypocalcaemia is a painful spasm and could be the presenting sign. Chvostek’s sign is twitching of the perioral muscles in response to tapping over the facial nerve at the ear. If urgent replacement is necessary, calcium gluconate can be given IV. It is preferred over calcium chloride as it causes less tissue necrosis if it leaks out. It is worth noting that digoxin may be ineffective until serum calcium is restored to normal.

197
Q

A child develops a black eye after falling off the horse. Fracture of the base of skull is diagnosed.

What is the name of the sign?
A.	Cullen's sign
B.	Chvostek's sign
C.	Corrigan's sign
D.	Grey-Turner's sign
E.	Raccoon eyes
F.	Quincke's sign
G.	Murphy's sign
H.	Trousseau's sign
I.	Battle's sign
J.	Muller's sign
K.	Traube's sign
A

E. Racoon eyes

Basilar skull fractures have specific clinical features. Blood pooling from these fractures can cause periorbital bruising (raccoon eyes), brusing over the mastoid area (Battle’s sign) and bloody otorrhoea. There may also be CSF leak resulting in CSF otorrhoea or rhinorrhoea. A unilateral raccoon eye has an 85% positive predictive value for this diagnosis.

198
Q

A 42-year-old obese lady with history of gall stones develops epigastric pain radiating to the back. She is tachycardic and hypotensive. There is a large bruise on the left flank. She denies any injuries.

What is the name of the sign?
A.	Cullen's sign
B.	Chvostek's sign
C.	Corrigan's sign
D.	Grey-Turner's sign
E.	Raccoon eyes
F.	Quincke's sign
G.	Murphy's sign
H.	Trousseau's sign
I.	Battle's sign
J.	Muller's sign
K.	Traube's sign
A

D. Grey-Turner’s sign

Complicated haemorrhagic pancreatitis may exhibit Cullen’s sign, Grey-Turner’s sign and Fox’s sign. Grey-Turner’s sign refers to bruising of the flanks and can take 24-48 hours to occur. It is due to retroperitoneal haemorrhage.

199
Q

Positive in cholecystitis.

What is the name of the sign?
A.	Cullen's sign
B.	Chvostek's sign
C.	Corrigan's sign
D.	Grey-Turner's sign
E.	Raccoon eyes
F.	Quincke's sign
G.	Murphy's sign
H.	Trousseau's sign
I.	Battle's sign
J.	Muller's sign
K.	Traube's sign
A

G. Murphy’s sign

A positive Murphy’s sign may commonly be seen in cholecystitis. This is where palpation of the right subcostal region reveals tenderness. During deep inspiration, the tenderness suddenly becomes worse and there is inspiratory arrest as a result. This sign can also be elicited during ultrasound examination when pressure is applied using the ultrasound probe.

200
Q

Nail bed fluctuation in aortic regurgitation.

What is the name of the sign?
A.	Cullen's sign
B.	Chvostek's sign
C.	Corrigan's sign
D.	Grey-Turner's sign
E.	Raccoon eyes
F.	Quincke's sign
G.	Murphy's sign
H.	Trosseau's sign
I.	Battle's sign
J.	Muller's sign
K.	Traube's sign
A

F. Quincke’s sign

Quincke’s sign is an uncommonly seen sign where there is subungal or lip capillary pulsations caused by the large stroke volume seen in AR. This is a peripheral sign associated with a bounding pulse and the systolic hypertension of chronic severe aortic regurgitation.

201
Q

Antidotes to be aware of:

  • Calcium channel blocker OD
  • Sodium channel blocker toxicity
  • Salicylate or phenobarbital OD
  • Cholinesterase toxicity
  • Cyanide toxicity
  • Warfarin toxicity
  • Digoxin toxicity
  • Sulphonylurea OD
  • Heavy metal toxicity
A

Other specific antidotes to be aware of (check out TOXBASE if interested):
• Calcium channel blocker OD can be treated with calcium chloride. Glucagon can be tried if calcium replacement alone is insufficient.
• Sodium channel blocker toxicity can be treated with sodium bicarbonate depending on the QRS length.
• Salicylate or phenobarbital OD can be treated with urine alkalinisation with IV sodium bicarbonate. Phenobarbital responds well to multidose charcoal.
• Cholinesterase toxicity can be treated with IV atropine if there is symptomatic bradycardia. Severe cases can benefit also from IV pralidoxime.
• Cyanide toxicity can be treated with hydroxocobalamin.
• Warfarin toxicity can be treated with FFP/PCC and vitamin K.
• Digoxin toxicity is treated with digoxin-specific antibody fragments (digoxin immune Fab).
• Sulphonylurea OD can be treated with glucose infusions in combination with IV octreotide if hypoglycaemia is problematic.
• Heavy metal toxicity can be treated with the appropriate chelating agent.